Sie sind auf Seite 1von 130

Notes:

1. Click on the green buttons


to view the multimedia learning resources (internet
connection required)
2. E-Reader functions (e.g.: highlight, notes) will not be available in this PDF view.
3. Desktop and laptop users may open the file within Adobe Digital Editions (ADE) to view the
e-book sample chapter with all the e-Reader functions. Please refer to these links for a stepby-step guide to install (ADE): Windows Mac OS

FORM

CHAPTER

Forces and Motion

Forces and Motion


Year

2007
1

Paper
Section
Number of questions

SPM Topical Analysis

2008

2010

2009
3

2011
3

2
A
1

C
1

1
B
1

A
1

B
1

3
C

ONCEPT MAP

FORCES AND MOTION


Dynamics

Kinematics
Linear
motion

Motion graphs
s t graph
v t graph

Inertia

Mass
Distance
Displacement

Acceleration
Deceleration
vu
a= t

Speed
= Distance
Time
Velocity
Displacement
=
Time

Acceleration due
to gravity, g
Equations of
motion
v = u + at
s = 1
(u + v)t
2
s = ut + 1
at 2
2
v2 = u2 + 2as

Newtons first
law of motion

Newtons second
law of motion
F = ma

Hookes
law
F = kx

Conservation
of
momentum

Impulsive force,
mv mu
F=
t

Elastic potential
energy,
1 2
E=
kx
2

Forces in
equilibrium

Newtons third
law of motion

Work,
W = Fs

Energy

Potential
energy,
Ep = mgh
Safety features
in the design of
vehicles

COMPANION WEBSITE

Learning Objectives

Impulse,
Ft = mv mu

Elasticity

Effects of a force

Gravity

Weight,
W = mg

Momentum

37

Kinetic energy,
1
Ek = 2 mv2
Power

2.1

Analysing Linear Motion

2.1

Analysing Linear Motion

Linear Motion
1 Linear motion is motion in a straight line.
2 Examples of linear motion:

3 Examples of non-linear motion:


Earth

Sun

(a) A spinning top


(a)A passenger on a moving escalator

Figure 2.2

4 The study of the motion of an object without


considering the forces acting on it is called
kinematics.
5 The study of the motion of an object and
the forces acting on the object is called
dynamics.

An athlete running a 100 m race


F
Figure 2.1 O
R
M

(b)

(b)The earth orbiting


the sun

CHAPTER

CHAPTER

F
O
R
M

Distance, Displacement, Speed and Velocity


4

1 The physical quantities of motion are distance, displacement, speed, velocity, time and
acceleration.
Motion
An object in motion changes position.

Distance
The total length of the path an object travels from
one location to another.
Distance is a scalar quantity.

Displacement
The distance an object travels in a specific
direction.
The magnitude of displacement is equal to the
shortest distance between two points.
Thus, displacement is a vector quantity.

Speed and velocity both describe how fast an object is moving but
there is an important difference between these two quantities,
i.e, velocity is related to direction.
Velocity
Velocity is the speed of an object in a specified
direction, that is, the rate of change in
displacement.

Distance moved in a specific direction


______________________________________
Velocity, v =

Time taken
____________________
Displacement, s (m)
=
Time taken, t (s)
Velocity is a vector quantity.

Speed
Speed is the distance covered per unit time, that is,
the rate of change in distance.

Total distance travelled, s (m)


_________________________
Speed, v =

Time taken, t (s)
Speed is a scalar quantity.

Forces and Motion

38

2 Both distance and displacement have the same SI units.


They are measured in metres (m).
3 Both speed and velocity have the same SI units of metre per
second (m s1).
4 However, since displacement and velocity are vector quantities, the
direction of motion must be stated together with the magnitude.

An object moves from position


A to B. The figure below
shows five possible paths
taken by the object. Which is
the shortest path?
Q
R

1
A

Figure 2.3 shows the location of two towns, P and Q.

Jamil's
car

Town Q
Raven's
helicopter

Town P
mountain

Figure 2.3

Jamil drives a car along the road from town P to town Q, which is 300 km away.
The journey takes 5 hours. Raven flies a helicopter due east from town P to Q
for a distance of l00 km in half an hour.
Analyse, in terms of speed and velocity, Jamil's and Raven's journeys.
Solution
Jamils journey
Distance travelled = 300 km
Time taken = 5 hours
Distance travelled
________________
Speed, v =

Time taken
_______
300 km
=

5h

= 60 km h1
The speed of the car is 60 km h1.

The path with the shortest


length is APB (the straight line
that joins A and B) and APB is
known as the displacement
from A to B.
The magnitude of the
displacement is the shortest
distance that links the initial
and final positions of an
object that has moved.
A
The other paths, AQB, ARB,
AXB and AYB are known as
the distances travelled from A
to B.

Ravens journey
Displacement = 100 km due east
Time taken = 0.5 hour
Distance in a
Displacement
specific direction.
____________
Velocity, v =
Time taken
_______
100 km
=
0.5 h

= 200 km h1
The velocity of the helicopter is 200 km h1 due east.

Note: SI unit is not used in this example.


Average Speed and Average Velocity

1 Refer to Example 2.
2 The term average velocity is used because the
athlete is not running at a constant velocity of
10 m s1 in the whole race.
3 Similarly, the speed of the car (in Example 1) is
an average speed since the speed of the car is
not the same throughout the whole journey.
4 Table 2.1 shows the difference between average
speed/average velocity and constant speed/
constant velocity.

An athlete runs 100 m in 10 s.


What is his average velocity?
Solution
s

Average velocity, v = __

t
______
100 m
=
10 s
=
10 m s1
39

Forces and Motion

CHAPTER

lake

F
O
R
M
4
Q
R

Table 2.1

Average speed / Average velocity

Constant speed / Constant velocity

A car moves at an average speed/velocity of


20 m s1 (equivalent to 72 km h1).
This means the car may move 15 m in the first
second, 25 m in the next second and 20 m in the
third second.
On average, the car moves a distance/displacement
of 20 m in 1 second for the whole journey.
15 m

t=2s

t=3s

10 m

t=1s

20 m

t=0

10 m
t=1s

10 m
t=2s

t=3s

An object is said to be moving at a constant or uniform speed if it moves


equal distances in equal successive time intervals, no matter how small the
F
time interval is.
O
For example, an object moving at Ra constant speed of 10 m s1 covers a
distance of 10 m every second, or 1 M
m every 0.1 s.
4

CHAPTER

t=0

25 m

CHAPTER

F
O
R
M

A car moves at a constant or uniform speed / velocity


of 10 m s1.
This means the car moves 10 m in the first second,
10 m in the next second, 10 m in the third second,
and so on.
The car always covers a distance or displacement
of 10 m in 1 second for the whole journey.
The magnitude of speed/velocity remains the same.

SPM
07/P1

3
In an activity for a Physics lesson, a student was
instructed by his teacher to run due north for
a distance of 12 m before moving east for another
16m. The time taken was 20 s.
What was the students
(a) average speed, and
(b) average velocity?

(a) Total distance travelled = AB + BC



= 12 + 16

= 28 m
_____________________
Total distance travelled
Average speed, v =

Time taken

28 m
_____
=
20 s

=
1.4 m s1
(b) Displacement
= The distance travelled in the direction of AC

Solution
16 m
B

=
122 + 162
Apply Pythagoras theorem
= 20 m
__
To determine the
tan = 16
direction of AC
12

= 1.333
tan = 53.1
Average velocity, v

Displacement
=
Time taken

20 m
=
20 s
= 1 m s1 in the direction N53.1 E

12 m
N

Forces and Motion

40

VP
P

Q
VQ

Note:
The speed remains constant throughout while the
V
velocity is changing (i.e. change in direction, but not
P
in magnitude, bearing in mind that velocity is a vector
quantity which has both magnitude and direction.)

Figure 2.4

If the car covers a distance of 3 m each second, find


(a) the speed of the car, F4/2/3
(b) the velocity of the car at
(i)
P(ii)Q(iii)R

Acceleration and Deceleration

VR

Q
VQ

1 When the velocity of an object changes with


time, the object is said to be accelerating.
2 Acceleration is defined as the rate of change
in velocity with time.
Acceleration, a
Initial velocity = u
Final velocity = v
Change in velocity
=
Time taken = t
Time
taken

Final velocity Initial velocity
=
Time taken

3 Since velocity is a vector quantity (i.e., it has


magnitude as well as direction), acceleration
is, thus, a vector quantity.
F4/2/3
4 The SI unit for acceleration is metre per
second per second or m s2 (read as metre per
second squared).


a = v u

t

v>u

Acceleration, a =

vu
t

v<u
Deceleration/Retardation

Acceleration
When v > u, a is positive, the velocity is increasing.
Thus, the speed of the object increases and is said
to be accelerating.
For example,
u=0
20

25

30 35

15
m s1

5
0

u = 30 m s1

v = 30 m s1
40
45

20

27

30

21

50

t=0

10

27

30

40
45

27

30

t=5s

27

30

3
6

24
21

10

9
18

15
20
25

t=0

The driver steps on the accelerator when the traffic


light turns green. The car increases its speed with
F4/2/4
an acceleration of 6 m s2.
Mathematically, we write as: a = 6 m s2

50

12

30

60

55
10

9
15

55
60

5
6

21

40
35

40
45
50

m s1

45

25

30 35

10
5

24

18

15
20
30

25

15

60

55
50

10

9
12

40

v=0
20

55
60

3
6

15

45

35

m s1

21
18

15
20
25

30 35

50

24

12

30

25

15

60

55
10

9
15

40

20

55
60

24

18

m s1

45

35

40
45
50

60

55

30 35

10

55
60

50

25

15

50

10

When v < u, a is negative, the velocity is decreasing.


Thus, the speed of the object slows down and is
said to be decelerating.
For example,

15

12

15

45

20

40
35

30

25

t=5s

The driver applies the brakes when he sees the


traffic lights turn red to reduce its velocity with a
F4/2/5
deceleration or retardation of 6 m s2 until it stops.
Mathematically, we write as: a = 6 m s2
41

Forces and Motion

CHAPTER

VR

Solution
(a) The speed of the car is 3 m s1.
(b) (i) At P:
The velocity, vP = 3 m s1 due east
(ii) At Q:
The velocity, vQ = 3 m s1 due west
(iii) At R:
The velocity, vR = 3 m s1 due N 45 W

Figure 2.4 shows a car moving round a roundabout.

F
O
R
M
4

5
From a constant velocity of 2000 m s1, the velocity of
a rocket increases to 3000 m s1 in 5 s when the rate of
combustion in the combustion chamber is increased.
What is the acceleration of the rocket?

Acceleration, a
vu
=
t
3000 m s1 2000 m s 1
=
5s

1000 m s 1
=
200 metres per second per
5s
second, i.e., a gain in velocity

of 200 m s1 in each second.
200 m s 1
=
1s

Solution

v = 3000 m s1
t=5s

Also read as 200 metres per


second squared.

= 200 m s2

The acceleration of the rocket in Example 5 is 200 m s2. This means that its velocity increases by 200 m s1
for every second, as illustrated in the following diagram.
F
Ot = 2 s
v = 2400 m s1
R
M

t=1s
v = 2200 m s1

t=0
v = 2000 m s1

t=3s
v = 2600 m s1

CHAPTER

u = 2000 m s1
t=0

CHAPTER

F
O
R
M

t=5s
v = 3000 m s1

F4/2/9

Azmi cycles at a uniform speed of 20 m s1. He then


stops pedalling and his bicycle comes to a stop after
8 s. What is his average deceleration?
u = 20 m s1

t=4s
v = 2800 m s1

Solution
u = 20 m s1, v = 0 m s1 and t = 8 s
v u
____
Acceleration,
a =
t
0 20
_____
=
8

= 2.5 m s2

v=0

Negative means
deceleration.

t=8s

Deceleration = 2.5 m s2

bicycle stops

stop pedalling

Figure 2.5

Displacement

change in displacement
causes

Study of Linear Motion

Velocity

change in velocity

Acceleration

causes

SPM
03/P3/(B)

Ticker-timer
1 A ticker-timer is a device used in the laboratory to study the motion of a moving object,
usually a trolley.
Forces and Motion

42

2 The ticker-timer can be used to determine the


following variables.
(a) The time interval of the motion
(b) The displacement of the object
2

(c) The velocity of the object


(d) The acceleration of the object
(e) The type of motion of the object

A vibrating metal strip with a pin is set to vibrate


up and down 50 times per second (i.e., at 50hertz,
which is the frequency of the a.c. supply).

A ticker-timer consists of an electrical


vibrator, which is connected to an alternating
current (a.c.) power supply (12 or 6 V).

1 vibrator
2 metal strip

CHAPTER

50 dots are
punched on the
tape in one second
3
ticker-tape
inclined runway
4 trolley

Figure 2.6

Each time the metal pin moves down,


it makes a dot on the carbonised ticker
tape which passes underneath it.

The ticker tape is attached to a trolley which moves on a


or runway. As the trolley moves, it pulls the tape
through the ticker-timer. A trail of dots is punched on
the ticker tape at equal time intervals. Thus, the dots on
the tape form a complete record of the motion of
the trolley.

F4/2/6
bench

Analysing Motion on a Single Strip


The type of motion of the trolley can be inferred from the distance between the dots on the ticker
tape pulled by the trolley.
direction of motion

The distance between two neighbouring dots is equal.


The object is moving at a constant/uniform velocity.
F4/2/14(a)
direction of motion

direction of motion

The distance F4/2/14(b)


between two neighbouring
dots is small.
The object is moving slowly.

The distance between


F4/2/14(c)two neighbouring dots is
greater.
The object is moving at a greater velocity.
direction of motion

direction of motion

The distance between two neighbouring dots


F4/2/14(e)
decreases.
The velocity decreases.
The object is decelerating.

The distance between two neighbouring


F4/2/14(d)
dots increases.
The velocity increases.
The object is accelerating.
43

Forces and Motion

F
O
R
M
4

To Find the Time Interval of Motion and Velocity of an Object

SPM
05/P3/(B)

1 The time for 1 dot-space, or 1 tick of time is the time interval between one carbon dot and
the next one on the ticker tape.
dot
number 0 1 2 3 4 5 678 910
direction
of
motion

20

30

40

50

one tentick
of time
First 10-tick
strip

Second 10-tick
strip

Third 10-tick
strip

Fourth 10-tick
strip

Fifth 10-tick
strip

4 Therefore, one 10-tick of time


= 10 0.02 s = 0.2 s

10-tick = 0.2 s
5 Similarly, one 5-tick of time and one 2-tick of
time can be calculated as shown in Table 2.2.
6 With the quantities of time interval and
displacement, we can calculate the velocity of
an object.

O
R
M

10-tick strip
A
direction of motion
1 2 3 4
5

4
6

Since the vibrating pin makes 50 dots in 1


second, therefore:
Time for moving 50 dot-spaces = 50 ticks = 1 s
1
s = 0.02 s
1 tick =
50

2 A 10-tick of time is the time interval from dot
number 0 to dot number 10 on the tape.
3 The next 10-tick of time is the time taken to
move from dot number 10 to Fdot number 20.

Table 2.2

CHAPTER

CHAPTER

Figure 2.7
F
O
R
M

B
9

10

8 cm

5-tick strip
P
direction of motion

3
2
1

6 cm

Q
4

2-tick strip
R
direction of motion

1

Time taken to move from A to B


= 10 0.02 s
= 0.2 s
10-tick = 0.2 s

Constant velocity,
s 8 cm
v = =
t 0.2 s
= 40 cm s1

Time taken to move from P to Q


= 5 0.02 s
= 0.1 s
5-tick = 0.1 s

Average velocity,
s 6 cm
v = =
t 0.1 s

Time taken to move from R to S


= 2 0.02 s
= 0.04 s
2-tick = 0.04 s

S
2

5 cm

= 60 cm s1
Average velocity,
s 5 cm
v = =
t 0.04 s
= 125 cm s1

To Find the Uniform Acceleration or Deceleration of an Object

7
A trolley is moving down a runway. A strip with six
dot-spaces as shown in Figure 2.8 is obtained.
1
direction
of motion A

The ticker-timer vibrates at a frequency of 50 Hz.


Find the acceleration of the trolley.
Solution
Frequency, f = 50 Hz
1 tick = 0.02 s

5
P

0.5 cm
= 25 cm s1
0.02 s
1.5 cm
Average velocity from P to Q: v =
= 75 cm s1
0.02 s
Average velocity from A to B: u =

1.5 cm

0.5 cm

Figure 2.8
Forces and Motion

44

Time taken (from the midpoint of AB to the


midpoint of PQ) to produce the change in velocity
= 5 0.02 s
= 0.1 s

Change in velocity
________________
Acceleration,
a =
Time taken
____
v u
=
t
______________
(75 25) cm s1
=
0.1 s
________
50 cm s1
=
0.1 s

= 500 cm s2
=
5 m s2

Watch out! It is 5-tick, not 6-tick. The average


velocity happens somewhere at the midpoints
of AB and PQ respectively. Therefore, the time
taken for the change in velocity is from the
midpoint of AB to the midpoint of PQ.

Alternative method

direction
of motion

0.02 s

0.04 s

0.06 s

0.08 s

0.10 s

CHAPTER

The time taken, t to produce the change in velocity can also be found as shown below.
First, label the time as an interval of 0.02 s at the dots as shown in the figure below.

0.12 s

t
0.01 s

0.11 s

t = 0.11 s 0.01 s = 0.1 s


Making a Tape Chart

2 On the tape, lines are drawn across dots


number 0, 10, 20 and so on, from the start of
the first clear dot, to mark off in sections 10
dot-spaces long.
3 The 10 dot-spaces are labelled in order. The tape
is then cut at the lines to form 10-tick strips.
4 The 10-tick strips are pasted in order side by
side on paper, preferably a graph paper, for
easy measurement, to form a tape chart as
shown in Figure 2.10.

1 Figure 2.9 shows a ticker tape obtained in an


experiment.
dot
number 01 2 3 4 5 6 7 8 9 10
direction
of motion

20

30

one tentick
of time

Figure 2.9

Velocity, v (or distance moved per tentick)

8
7
6
4

3
2
1

0
0

10 20 30 40 50 60 70 80
0.2 0.4 0.6 0.8 1.0 1.2 1.4 1.6

Time in ticks
Time in seconds

Figure 2.10

5 The length of each 10-tick strip is the distance


moved in one 10-tick of time, that is, 0.2 s.
6 The length of the strip represents velocity. The
longer the strip, the greater the velocity since

the time taken for each strip is the same, i.e.


0.2s.
7 As such, the tape chart is practically a velocitytime graph where the vertical axis is the
45

Forces and Motion

F
O
R
M
4

velocity (distance moved per 10-tick), while


the horizontal axis indicates the time since
each strip starts 0.2 s after the one before.
8 Some may prefer to label the vertical axis as
length of the strips, but remember that this

Activ

To determine displacement, velocity and acceleration of a trolley

ity 2.1

Apparatus/Materials
Trolley, runway, ticker-timer, 12 V power supply,
ticker tape, cellophane tape and wooden block.
CHAPTER

F
O
R
M

length actually represents the velocity since the


time taken for each strip is the same.
9 Tape charts can also be made of strips with
5 dot-spaces or 2 dot-spaces.

Calculation
(a) Displacement = x1 + x2 + x3 + x4 + x5 + x6
where x is the length of the strip.
(b) Time taken = 6 10-tick

= 6 0.2 s

= 1.2 s
Displacement
Average velocity = ____________
Time taken

(x1 + x2 + x3 + x4 + x5 + x6) cm

= _________________________
1.2 s

Arrangement of apparatus
ticker tape

ticker-timer
trolley

12 V a.c.
power
supply

inclined
runway

wooden
block

Figure 2.11

F
O
R
The apparatus is set up as shown
M in Figure 2.11.

(c) Average velocity for the 1st strip:


x1
___
u=
cm s1
0.2

Procedure

CHAPTER

1
2 The inclination of the runway is set so that the
4 it is released.
trolley will roll down freely after
3 A length of ticker tape is passed through the
ticker-timer and attached to the trolley.
4 The ticker-timer is switched on and the trolley is
released.
5 The ticker tape obtained is then cut into 6 pieces
of 10-tick strips.
6 The strips are pasted side by side on a graph
paper to form a tape chart.
Results

Average velocity for the 6th strip:


x6
v = ___
cm s1
0.2
ime taken for the change in velocity, t
T
= (6 1) 0.2 s
= 5 0.2 s
= 1.0 s
Acceleration of the trolley is calculated from the
formula:
vu
a=
t

velocity (distance moved per tentick)


5
x6
x5
x4

x2

The length of strip increases uniformly.


Thus, the trolley moves down the runway at a
constant or uniform acceleration.

x3

Discussion

Conclusion

x1

Activity 2.1

The average velocity and acceleration of a trolley are


thus determined.

0
0

10 20 30 40 50 60 Time in ticks
0.2 0.4 0.6 0.8 1.0 1.2 Time in seconds
t

Figure 2.12
F4/2/10
Forces and Motion

46

To prepare a friction-compensated runway


In Activity 2.1, there are two main forces acting on the trolley along the runway, i.e., the component weight of the trolley
down the runway and the frictional force of the runway on the trolley (please refer to Section 2.9 on page 118).
(a) If the slope is not steep enough,
(b) If the slope is too steep, the trolley (c) The slope is adjusted until the trolley
moves down the runway by itself.
goes down at uniform velocity, after
after a slight push, the trolley moves
The dots get farther apart.
a slight push. The dots on the tape
a short distance and then stops.
are equally-spaced. This is a
The dots on the tape get closer.
friction-compensated runway.
direction of motion

stop

Key :
component weight

friction

trolley goes down on its own


velo
city
incr
eas
ing

F4/2/11b

direction of motion
trolley given a slight push

cons

tant v
elocit
y

F4/2/11c

Average velocity
____________________
Total distance travelled
=
Time taken

30 cm
______
=
= 30 cm s1
velocity (distance moved per tentick)
1.0 s
= 0.30 m s2
4
(c) Average velocity for the 1st strip:
10
Key :
3
____
2 cm
component
u =
= 10 cm s1
start
weight
8
0.2
s
2
trolley given a slight push
friction
Average velocity for the 5th strip:
6
stop
1
______
10 cm
v =
= 50 cm s1 Watch out, not 5 10-tick!
4
0.2 s
The time taken for the
change in velocity is from
Time taken, t
2
the midpoint of the 1st strip
=4 10-tick
to the midpoint of the 5th
=4 0.2 s = 0.8 s
0
strip.
0 10 20 30 40 50 Time in ticks
OR:
0 0.2 0.4 0.6 0.8 1.0 Time in seconds
Time taken, t
t
From the time axis.
0.1
0.9
= (0.9 0.1) s = 0.8 s
Figure 2.13
Change in velocity
_________________
Acceleration, a =
Find
F4/2/12
Time taken
(a) the total distance travelled,

v u
____
(b) the average velocity,
=
t
(c) the acceleration of the trolley.

(50 10) cm s1
______________
=
Solution
0.8 s
(a) The total distance travelled

=
50 cm s2 = 0.5 m s2
(from dot number 0 to number 50)
Note:Since the motion is of uniform acceleration, the
= (2 + 4 + 6 + 8 + 10) cm
average velocity in (b) can also be calculated
=
30 cm
u+v

from the formula v =
.
(b) Total time taken
The total time taken
2
= 5 10-tick
is 1 s if you label the
1
= 5 0.2 s
v = (10 + 50) cm s =30cm s1
time axis in seconds.

2
= 1.0 s

A student carried out an experiment using a trolley


and a ticker-timer that vibrates at a frequency of
50Hz. Figure 2.13 shows a tape chart consisting of
10-tick strips that he obtained.

47

Forces and Motion

direction of motion

trolley given a slight push

CHAPTER

Physics Blog

F
O
R
M
4

In the questions, the vertical axis may not be labelled as


velocity or distance moved per 10-tick (or 5-tick or
2-tick). However, the way to solve the problems is still
the same.

As explained earlier, the distance between dots, or the


length of a strip, represents the velocity of a moving
object. Therefore, the increase in distance between dots
and the increase in length between successive strips
represent the increase in velocity. If the increment is
uniform, then the increase in velocity is uniform, that is,
the object is moving with uniform acceleration.
(a) In the figure below, the increase in distance
between the successive dots is equal (i.e., 0.2 cm).
Therefore, the acceleration is uniform.

Figure 2.14 shows a tape chart for a trolley which


moves up on an inclined plane. Determine the
deceleration of the trolley.

direction of motion
1.0 cm 1.2 cm 1.4 cm 1.6 cm

1.8 cm

2.0 cm

Length of strips (cm)


1

(b) In the figure below, the increase in length between


the successive strips is equal (i.e., 1 cm). Therefore,
the acceleration is uniform. (Alternatively, if a
straight line can be drawn across all the top
midpoints of the strips, the acceleration is uniform)

4
5

4
3

F
O
R
0.04 0.08 0.12 0.16 0.20 0.24
M Time (s)

Velocity (cm per tentick)

1
0

t
0.02

0.22

Velocity

8
7
6
5

CHAPTER

CHAPTER

F
O
R
M

4
3
2
1

Figure 2.14

Solution
The time of each 2-tick strip = 2 0.02 s
= 0.04 s
6 cm
u=
= 150 cm s1
0.04 s
1 cm
v =
= 25 cm s1
0.04 s

Time (s)

Time

0 0.2 0.4 0.6 0.8 1.0 1.2

(c) In the figure below, there is no increase in length


between the successive strips. Therefore, the
acceleration is zero (the object is moving with
constant or uniform velocity).
Velocity (cm per tentick)

Time taken to produce the change in velocity, t


= 5 0.04 s
= 5 2-tick
= 0.20 s
OR: t = (0.22 0.02) s
If the time axis is
= 0.20 s
labelled.
Acceleration, a
v u
____
=
t
(25 150) cm s1
=
0.2 s

125 cm s1
=
0.2 s

= 625 cm s2
= 6.25 m s2
Deceleration = 6.25 m s2

Velocity

Time(s)

Time

0 0.2 0.4 0.6 0.8 1.0

(d) In the figure below, the decrease in length between


the successive strips is equalVelocity
(i.e., 2(cm
cm).
Therefore,
per tentick)
the deceleration is uniform.
8

Velocity (cm per 5-tick)


16
14
12
10
8
6
4
2
0

7
Velocity
6

Velocity (cm per tentick) 5

Veloc

4
3

6
Time(s)
0.1 0.2 0.3 0.4

Time

Time (
0
Forces and Motion

48

0.2 0.4 0.6 0.8

1.0

0.2 0.4 0.6 0.8 1.0 1.2


Time
F4/2/34

The Equations of Linear Motion


For linear motion with uniform acceleration, a, the displacement or
distance travelled, s, in a particular direction is given by:

s = Average velocity Time taken
s = 1 (u + v)t
2

Summary of the equations for


linear motion with uniform
acceleration:
v = u + at
1
s = (u + v) t
2
s = ut +

substitute

1
s = (u + v)t
2
1

= (u + u + at)t
2
1

= (2ut + at 2)
2
s = ut +

rearrange

t = v u
a

substitute

( )

1
s = u + v t
2

1
s = (u + v) t
2
(v u)
1
= (u + v) a

2

v 2 = u 2 + 2as

There are 5 physical quantities in the equations of


linear motion with uniform acceleration.
Each of the four equations of motion involves 4 physical
quantities.
To solve numerical problems, you need to know 3
quantities before you can find the value of the fourth
quantity. In the beginning, you may be unsure of which
equation to choose to solve the problem. You may start
by trial and error. After some practice, you will be able to
select the required equation easily.
The table below shows some examples.

(II)For s =

2000 m s1

6000 m s1

80 km

Solution
u = 2000 m s1, v = 6000 m s1,
s = 80 km = 80 000 m, t = ?

Solution

First trial:

1
s = ut + at2
2

u = 20 m s1, a = 2 m s2, t = 8 s, v = ?


?
Second trial: v = u + at

v = 20 + 2(8)
= 36 m s1

1
(u + v) t:
2

The velocity of a rocket travelling at 2000 m s1


increases to 6000 m s1 after moves through a
distance of 80km. Calculate the time for the rocket
to reach this velocity.

(I)For v = u + at:
A car accelerates from 20 m s1 with an acceleration
of 2 m s2.
What is the velocity after 8 seconds?

v 2 = u 2 + 2as

s = displacement
u = initial velocity
v = final velocity
a = uniform acceleration
t = time interval

(v 2 u2)
= 1
a
2
2
2
2as = v u

1 2
at
2


First trial:

v 2 = u2 + 2as

?
1
Second trial: s = (u + v) t
2

Unable to
calculate v,
because only
2 quantities are
known.

3 quantities are
known. Can be
used to find v.

49

(2000 + 6000)
t
80 000 =
2

t = 20 s

Unable to
calculate t
because only
2 quantities
are known.
3 quantities
are known.
Can be used
to find t.

Forces and Motion

a = v u
t

CHAPTER

rearrange

v = u + at

1 2
at
2
F
O
R
M
4

10

12
By applying the brakes, a driver reduces the
velocity of his car from 20 m s1 to 10 m s1 after
a distance of 30 m. Calculate the deceleration of
the car.

a = 2 24

9
= 5.3 m s2

Solution
u = 20 m s1, v = 10 m s1, s = 30 m, a = ?
Applying v 2 = u2 + 2as:
102 = 202 + 2a(30)
100 400
a =

2(30)
= 5 m s2
Deceleration = 5 m s2

11

1
2

F
O
R
M

SPM
Clone

07

The figure below shows the route taken by a van


from town P to town S.

CHAPTER

F
O
R
M

Solution
u = 0, s = 24 m, t = 3 s, a = ?
1
Applying s = ut + at 2:
2
1
24 = 0(3) + a(3)2
2

CHAPTER

Starting from rest, a sprinter reaches his top velocity


in 3 seconds. He runs a distance of 24 m in the
3 seconds. What is his acceleration?
(Assume his acceleration is uniform.)

Salina is driving at a velocity of 10 m s1.


4 to stop her car.
Seeing a cow in front, she brakes
If the deceleration of the car is 2 m s2, what is
the distance the car covers before it comes to a
halt?

3 km

R
1 km
S

5 km

Solution
u = 10 m s1

v=0

What is the displacement of the van?


F4/2/14a
A 4.0 km
C 7.5 km
B 5.0 km
D 9.0 km

s=?

Comments
Displacement is a vector quantity. It is the shortest
distance that links the initial position of an object
to its final position. In the above question, the
displacement is the distance PS. Join P to S, and use
Pythagoras theorem to calculate the displacement.

u = 10 m s1, v = 0, a = 2F4/2/13
m s2, s = ?
2
2
Applying v = u + 2as:

0 = 102 + 2(2)s
4s = 100
100
s =
4

= 25 m

3 km

1 km

3 km

1 km
S

4 km

PS = 3 2 + 4 2
= 5 km

Dont forget to insert the negative sign for deceleration


when doing the calculation.

Forces and Motion

Answer B
50

F4/2/14b

2.1
1 During a class activity, Hashim walks 20 m due east.
He reverses his direction and walks 12 m. After that,
he reverses his direction again and walk for another
10m. If the total time taken is 30 s, what is his
speed and velocity?

6 When a dart is blown from a blowpipe of 1.2m


length, it travels at a speed of 15ms1. Find the
time taken for the dart to travel in the barrel.
7 Daniel drives his car at a constant velocity of
20ms1. He steps on his brakes to reduce the
velocity of the car to 10ms1 after travelling 30 m.
Calculate the deceleration and the further distance
covered before the car stops.

2 A speedboat moves due north for 12 km before


turning east for 8 km. Later, the speedboat moves
south for 6 km. What is the displacement of
the speedboat from its original position? Find its
average velocity, in m s1, if the total time taken is
30 minutes.
3 Find the acceleration or deceleration for the tape
charts below. The ticker-timer in use vibrates at a
frequency of 50 Hz.

CHAPTER

(a)

v = 33 m s1
u=0

Velocity (cm per 10 ticks)


s
10

(b)

If the plane accelerates at 3ms2, calculate the


minimum length of the runway needed by the plane
to reach the speed for take-off.

Time

9 Velocity
A baseball
pitcher
holds a baseball in his hand for
(cm per
10 ticks)
3 m before the baseball is thrown at a velocity of
39ms1.

Length of strips (cm)

10

12

3.0 m

10
8
6
4

39 m s1
Time

Time

4 A sports car accelerates from rest and covers a


distance of 90 m in 6 s. What is its acceleration?
5 A cyclist starts from rest and reaches a velocity of
20ms1 in 8 s. Calculate his acceleration.

2.2

2.2

Find the acceleration of the baseball before it is


released.

Analysing Motion Graphs

Analysing Motion Graphs

Motion Graphs
1 Graphs can be useful in studying motion. They show the changes in
the motion of an object with time.
2 There are two main types of linear motion graphs:
(a) the displacement-time graph
(b) the velocity-time graph
51

8 An aeroplane needs a velocity of 33ms1 for


take-off.

Forces and Motion

F
O
R
M
4

Displacement-time Graphs
1 A displacement-time graph (s-t graph) is a graph that shows how the
displacement of an object varies with time.
2 Figure 2.15 shows a student cycles at a constant velocity from position
A to reach position B, which is 300m away, in 200 seconds. He rests
for 100 seconds at position B and then cycles back to position A using
the same straight path. He reaches position A after another 200
seconds.
zero velocity at rest

displacement (m)

300

negative velocity
moving in opposite direction
and stops at
the starting point

y
I
A
O

II

III
A

x
100

200

300

400

500

time (s)

O
s=0

s = ()
v = ()
(object
on the
left of
O and
moving
due
west)

v=0

s = (+)
(object on the
right with
respect to O)
v = ()
(object moving
due west)

F
Figure
2.15
O
F4/2/15
R
M
4

CHAPTER

positive velocity
moving in a fixed direction

CHAPTER

F
O
R
M

Relationship between the


position of an object,
displacement, s, and velocity, v:
s = (+)
v = (+)
(object on the
right of O and
N
moving
due east)

In Section III of the graph:


From conclusion 1:
Velocity = Gradient
(0 300) m
=
(500 300) s

In Section I of the graph:


To find the velocity, use the formula:
Change in displacement
Velocity =
Time taken
300 m
Velocity, v =
200 s


= 1.5 m s1
The negative sign shows that the direction of
motion is opposite to its original direction.
Take note that velocity is a vector quantity.
At t = 500 s, the graph intersects the t-axis.
The displacement at this moment is zero, that is,
the student has returned to the original position.

= 1.5 m s1
Gradient of the graph
y
=
x
(300 0) m
=
(200 0) s
= 1.5 m s1

Conclusion 1
On a displacement-time
graph, the gradient of the
graph is equal to the
velocity of the object.

Forces and Motion

In Section II of the graph, a horizontal line is drawn from


t = 200 s to t = 300 s. During this period, the student
remained at position B, which is 300 m away from position
A, the origin, with a velocity of zero.

Conclusion 2
On a displacement-time graph,
a horizontal line (gradient = 0)
shows that an object is
stationary, i.e., not in motion.
52

Velocity-time Graphs

SPM
03/P1

SPM
04/P1

SPM
05/P1

SPM
07/P1

SPM
08/P1

SPM
09/P1

1 A velocity-time graph (v-t graph) is a graph that shows the variance in the velocity of an object against
time.
2 A car starts from rest and accelerates for 20 seconds until it reaches a velocity of 30 m s1. The driver
maintains this velocity for 20 seconds. The velocity of the car is then reduced until it stops at t = 60 seconds.
3 The graph in Figure 2.16 shows how the velocity of the car changes against time.

t=0
starts
v (m s-1)

t = 20 s

v = 30 m s-1 v = 0 m s-1
t = 40 s

t = 60 s
stops

constant velocity
30
acceleration
(positive
gradient)

II

20

40

deceleration
(negative
gradient)

III
x

60

t (s)

FigureF4/2/17
2.16
To determine the acceleration of the car
Using the formula:
Change in velocity
Acceleration=
Time taken

The negative
sign indicates
deceleration.

A negative gradient indicates deceleration.

vu
t
(30 0) m s1
=
20 s

Distance travelled from t = 20 s to t = 40 s:


Distance =
 Velocity Time
= 30 m s1 20 s
= 600 m
Area under the graph (for section II)
= 30 20
= 600 unit2

=
1.5 m s2
y
Gradient of the graph =
x
(30 0) m s1
=

(20 0) s

= 1.5 m s2
The deceleration of
the car is 1.5 m s2.

Conclusion 3

Acceleration, a =


To determine the acceleration of the car


From conclusion 1:
Acceleration, a
= Gradient
(0 30) m s1
= (60 40) s

=
1.5 m s2

Conclusion 4

Conclusion 1

On a velocity-time graph, the area under


the graph is numerically equal to the distance
travelled.

On a velocity-time graph, the gradient of the


graph represents the acceleration of the object.

The car travels at a constant velocity of 60 m s1


from t = 20 s to t = 40 s. A horizontal line is
shown in the graph.

This method of calculating the distance travelled


can be applied to any velocity-time graph,
whether the velocity is constant or not.
For example, in Section I of the graph, the area
1
base height)
of the shaded triangle (=
2
equals 300. So, the car travelled a distance of
300 metres in the first 20 second of its motion.

Conclusion 2
On a velocity-time graph, a horizontal line
(gradient = 0) represents a constant velocity.

53

Forces and Motion

v = 30 m s-1

CHAPTER

v = 0 m s-1

F
O
R
M
4

13
Solution
(a) The line OA shows that the motorcycle accelerates
uniformly from 0 m s1 to 12 ms1.
The line BC shows that the motorcycle decelerates
uniformly before coming to a rest.
(b) The motorcycle is moving at a constant velocity
for 10 s (line AB of the graph).
(c) Distance between the two traffic lights
= Area under the graph
1
= (10 + 25) 12 = 210 m

2

Figure 2.17 shows the velocity-time graph of a


motorcycle travelling along a straight road between
two traffic lights.
velocity (m s1)

(d) From 0 s to 5 s:
C
O

10

15

20

time (s)

25

Acceleration =

12 0
= 2.4 m s2
50

From 5 s to 15 s:
because the
Acceleration = 0
object is moving
velocity
s1)
(m From
15 s to 25 s:
at constant
Acceleration
velocity
(a) Explain the motion of the
F motorcycle as
0B 12
represented by the lines OAO and BC on the A

12
=
= 1.2 m s2
R
graph.

25 15
(b) What is the time interval Mduring which the
Thus, the acceleration-time graph is as shown below.
motorcycle is moving at a constant velocity?
C 2
4
(m s time
) (s)
(c) What is the distance between the two traffic O 5 10 acceleration
15 20 25
2.4
lights?
(d) Sketch an acceleration-time graph to represent
time (s)
the motion of the motorcycle between the two
1.2
traffic lights.
2

Figure 2.17

CHAPTER

CHAPTER

F
O
R
M

12

Non-uniform Velocity
displacement, s (m)

60

55
50

30

27

3
6

24
21

10

9
18

15

12

15

45

20

40
35

t=0

25

30

s1
60

55
50

30

27

21

10

9
18

15

12

15
20

40
35

gradient is steeper
greater velocity

24

45

t=1s

gradient
= velocity
(at t = 1 s)

25

30

s2

s
60

55
50

27

30

24
21

10

9
18

15

12

15

45

20

40
35

30

25

t=2s
O

(a)

time, t (s)

(b)

Figure 2.18
F4/2/18
Forces and Motion

54

1 Figures 2.18(a) and (b) show the


positions of a ball falling from rest
and its corresponding displacementtime graph.
2 The ball covers a longer distance in
the second second as compared with
the first second. The ball is moving
with non-uniform velocity.
3 To determine the balls instantaneous
velocity, for example, at t = 1 s or
t = 2 s, a tangent must be drawn at the
respective points on the graph, as
shown in Figure 2.18(b).
4 The gradient of the tangent is equal
to the velocity.
5 However, the stone is falling with
uniform acceleration.

1 Figure 2.19 shows an


athlete running a 100 m
track and the corresponding
velocity-time graph of his
motion.
2 The athlete increases his
velocity (accelerates) until
the maximum velocity.
He maintains the
maximum velocity to
finish the race.
3 The acceleration of the
athlete can be determined
by drawing tangents at the
respective points and
determining the gradients
of the tangents.

constant velocity

velocity, v (m s-1)

constant velocity
smaller gradient
smaller acceleration

gradient = 0
zero acceleration

gradient of tangent
= acceleration at this instant

v
t
O

t1

time, t (s)

t2

Figure 2.19
F4/2/19

A comparison between the displacement-time graph and the velocity-time graph:


Displacement-time graph

Velocity-time graph

displacement

velocity

A
time

time

The object moves at a


constant velocity.
Represents the velocity of
the object.

Non-horizontal
straight line

The object moves at a constant acceleration.

Gradient

Represents the acceleration of the object.

The object is stationary.

Horizontal line

The object moves at a constant velocity.

The object returns to its


original position.

Intersection on
the time-axis

The object stops.

Positive The object moves


in a specific
direction.
Negative The object moves
in the opposite
direction.
No significance

Sign of the
gradient
(positive or
negative)

Positive Acceleration
Negative Deceleration

Numerically equals the distance travelled by the


object

Area under the


graph

55

Forces and Motion

CHAPTER

acceleration

Non-uniform Acceleration

F
O
R
M
4

14
(a) 10 s (from t = 0 s to t = 10 s, the velocity is
positive).
(b) 4 s (from t = 16 s to t = 20 s, the velocity is
negative).
(c) Distance travelled while moving towards the
east, S1 = Area of triangle ABC
1
= 10 20

2

= 100 m
Distance travelled while moving towards the west,
S2 = Area of triangle PQR
1
= 4 10

2

= 20 m

The velocity-time graph in Figure 2.20 shows a particle


starting from rest and travelling east.
v ( m s 1 )
B

20
10

16 18 20

t ( s)

Figure 2.20

(a) How long does the particle travel towards the east?
(b) How long does the particle travel towards the west?
(c) Find the average speed and the average velocity.

Total distance
Average speed =

Time taken
100 + 20
=

20

Solution
To understand the above situation better, please refer
F
to the figure below which demonstrates
the positions
O
of the particle that vary with time. R

100 m
v = 20 m s1
B

v=0
A

CHAPTER

10

10

CHAPTER

F
O
R
M

t=5s

t=0s
N

v=0
C

v = 10

v=0

R
Q
P
t = 20 s t = 18 s t = 16 s

20 m

Note: The particle is at rest from t = 10 s to t = 16 s.

Final displacement
Time taken
100 20
=
20

Average velocity =

t = 10 s
v=0

=
6 m s1

= 4 m s1 to the east

(Please note that in the situation discussed, the


particle is always to the east of its original
position.)

F4/2/21a

Area under velocity-time graph


v

An object moving at a decreasing acceleration is


moving at a decreasing velocity.
A
B

The velocity of an object with a decreasing acceleration


is always increasing. However, the rate of increase is
getting smaller.

Area A (above the t-axis)


= Distance travelled by the moving object in its original
direction
Area B (below the t-axis)
=
Distance travelled by the moving object in the
opposite direction
Total distance travelled = Area A + Area B
Final displacement = Area A Area B

Forces and Motion

56

SPM
Clone

11

9m
15 m

6
4
2
time (s)

-2
-4
-6

CHAPTER

Comments
Displacement = Area above the time-axis Area below
the timeaxis
1
1
=
(6)(2 + 3) (2)(6)
2
2

= 15 m 6 m

= 9 m

velocity(m s-1)

C
D

Answer: C

What is the displacement of the toy car in 6 s?

2.2
1 The figure
shows the
displacementtime graph
of a moving
particle.

s (m)

3 The figure below shows the velocity-time graph of a


motorcycle starting from rest and travelling north.

20
15

v (m s1)

10
12

5
O

10

15

t (s)

(a) What is the velocity of the particle in the initial


period of 3 seconds?
(b) How long is the particle stationary?
(c) At what point in time does the particle return to
its original position?
(d) Calculate
(i) the average speed, and
(ii) the average velocity of the moving particle.
2 The
velocitytime graph
shows the
movement
of a particle.

10

10

13

20

t (s)

(a) What is the deceleration from t = 10 s to t = 13 s?


(b) What is the displacement of the motorcycle
during the first 13 s?
(c) For how long is the motorcycle travelling towards
the south?
(d) What is the final displacement of the motorcycle
at t = 20 s?
(e) What is the average velocity of the motorcycle for
the whole journey?

12

15
6

10

v (m s1)

4 A sports car starting from rest, accelerates uniformly


to 30 m s1 over a period of 20 s. The car maintains
the velocity for 30 s. The velocity is then reduced
uniformly to 20 m s1 in 10 s and then brought to
rest after another further 10 s.
Draw a velocity-time graph to represent the journey
as described above. From the graph, find
(a) the acceleration of the car for the first 20 s,
(b) the distance travelled, and
(c) the average velocity over the time described.

t (s)

(a) What is the total distance, in m, travelled by the


particle in 10 seconds?
(b) For how long is the particle moving with constant
velocity?
(c) Calculate the ratio of acceleration : deceleration.
(d) Calculate the average velocity of the particle.

57

A 0 m
B 6 m

The diagram shows the velocity-time graph of a toy


car in motion.

Forces and Motion

F
O
R
M
4

2.3

Understanding Inertia

2.3

Understanding Inertia
SPM
05/P1

Concept of Inertia

SPM
09/P1

1 All objects tend to continue with what they are doing.


2 Newton's first law of motion (also known as the Law of Inertia)
states that:

3 The tendency of an object to maintain its state of rest or uniform


motion in a straight line is called inertia.
4 Only an external force (or a non-zero net force) can cause a change to
the state of motion of an object (either at rest or moving at the same
speed in a straight line).
5 The external force is a non-zero net force, if more than one external
force act on the object.
SPM
09/P1

F
O
R
M

Situations Involving Inertia

CHAPTER

CHAPTER

F
O
R
M

Every object continues in its state of rest or uniform speed in


a straight line unless acted upon by an external force.

F4/2/23

2
cardboard
coin

thread X
(can withstand 10.5 N)

weight (10 N)
thread Y snaps

Forces and Motion

F4/2/24

When a boy is riding a bicycle that runs over a stone, he is


thrown forward and off the bicycle.
Explanation
The stone abruptly stops the motion of the bicycle, but the
inertia of the boy keeps him in the forward motion and
throws him forwards.
A cardboard is placed on the rim of a glass and a coin is
placed on top of the centre of the glass. When the
cardboard is pulled away quickly, the coin resting on the
cardboard drops straight into the glass.
Explanation
The inertia of the coin maintains its state of rest. When the
card is pulled away, the coin falls into the glass due to gravity.
Note:
If the card is pulled away slowly, the frictional force
between the coin and the card causes the coin to accelerate
so that it moves together with the card. However, if the card
is pulled quickly, the time is too short for the friction to
cause any appreciable movement of the coin.
If thread Y is pulled slowly, thread X will snap. If thread Y is
pulled suddenly, then thread Y will snap.
Explanation
When thread Y is pulled slowly, the additional force to the
weight causes thread X to snap when the tension exceeds
the breaking force. When thread Y is pulled abruptly, the
weight maintains its state at rest due to inertia. Thread Y
thus stretches and snaps. (If the weight does move, it will
stretch and snap thread X instead.)
58

When a stationary bus starts to move forward with an


acceleration, the passengers are thrown backwards.

When a moving bus stops suddenly, the passengers lurch


forwards.

Explanation
The passengers are in a state of motion when the bus is
moving. When the bus stops suddenly, the inertia of the
passengers keeps them in motion. Thus, the passengers
lurch forwards.

A weight is suspended by a fine thread and rests at point P.


When the weight is raised and then released, the thread
snaps when it passes point P.
Explanation
The fine thread is able to support the weight when it is
stationary. When the weight is allowed to fall, the weight
maintains its inertia of moving downwards when it passes
point P. The inertia of the weight causes an additional force
to exert on the thread, thus causing it to snap.

6
fine
thread

thread
snaps

weight

F4/2/25

OK

BO

OK

BO

OK

BO

F4/2/26

If a book is pulled out very quickly from the middle of a


pile of books, the books above it will drop instead of
moving along with it.
Explanation
The inertia of the books above keeps them in their original
position.
Again, if the book is pulled out slowly, the books above it will
move together with the book as explained in situation 1 .

59

Forces and Motion

CHAPTER

Explanation
The passengers in the bus are originally in a stationary
state. When the bus starts to move forward with an
acceleration, the inertia of the passengers keeps them in
their position. Thus, the passengers are thrown backwards.

F
O
R
M
4

The decorative item hanging from the rear-view mirror


swings backwards when a stationary car starts to move or
accelerates.

Explanation
The inertia of the decorative item keeps it in its original
position while the car moves forwards.
The decorative item swings forwards when the moving car
comes to a sudden halt.

O
R
M
4

F object will not change its state of motion unless forced to do so.
Inertia = Laziness literally, such that an

CHAPTER

Explanation
The inertia of the decorative item keeps it in motion when
the car stops suddenly.

CHAPTER

F
O
R
M

I am in
motion.
I will always
move at
constant
speed in a
straight line.

I am at rest,
I will always
stay at rest.

Force is needed to change its velocity, i.e.,


(a) to get the object to start moving, or
(b) to make the object move faster, slower or change the direction of motion.

Hard-boiled or raw?
On a table are two eggs, one raw and one hard-boiled.
How can the two eggs be distinguished?
Solution
The answer is to spin the egg.
Spin each egg on its side. It is much easier to spin the hard-boiled egg.
A hard-boiled egg is solid whereas a raw yolk floats in the liquid white
in a raw egg. When a hard-boiled egg is spun, the solid contents turn
together with the shell. When a raw egg is spun, the yolk and the liquid
white tend to remain at rest because of inertia. The reluctance to spin
along with the shell makes the raw egg harder to spin.

Forces and Motion

60

SPM
04/P1

Relationship between Mass and Inertia

SPM
07/P1, P2

(a)Empty trolley (b)Full trolley


Figure 2.21

2 Similarly, it is easier to stop an empty trolley than a full one if both are
moving at the same speed towards the shopper.
3 The more mass an object has, the harder it is to change its state of
motion.

For example,
(a) It is more difficult to start moving a bucket filled with sand.
(b) When both the buckets are swinging and an attempt is made to stop
them at the lowest point of the swing, it is more difficult to stop the
bucket filled with sand.
ceiling
rope of
the same
length
sand

empty
bucket

bucket filled
with sand

It is harder
to start it
moving from
rest

Massive oil tankers carry


crude oil ranging from 150 to
300 thousands tons. With full
power on, a tanker with a
very large inertia takes a long
time to accelerate to its
maximum speed. The tanker
might need to travel a
distance of 10km before
coming to a stop even with
the engine set into reverse
condition (propeller in
reverse rotation).
It is not an easy job to turn
the tanker around since the
inertia of the crude oil is
enormous. Supertanker
officers need special training
in the handling of heavily
loaded ships.

It is harder
to stop it
here with
your hand

Figure 2.22

F4/2/28

(c) This shows that the bucket with more mass offers a greater resistance
to change from its state of rest or from its state of motion.
4 By the same reasoning, it is harder to start a bowling ball moving and
harder to stop it than a hollow rubber ball of the same size.
5 Thus, an object with a larger mass has a larger inertia.

Mass is a measure of the inertia of a body.
6 However, inertia is a phenomenon. It has no unit even though it is
closely related to mass.

61

Forces and Motion

CHAPTER

Supertankers: Large crude oil


carriers

1 A shopper in a supermarket observes that it is always easier to start


moving an empty trolley than a full trolley.

F
O
R
M
4

2.1
SPM
04/P3(A)

Relationship between inertia and mass

Procedure

Situation

1 One end of the hacksaw blade is clamped by a


G-clamp to a leg of a table as shown in Figure 2.24.
2 A 50 g plasticine ball is fixed to the free end of
the blade.
3 The free end of the blade is displaced horizontally
and released so that it oscillates. The time for 20
complete oscillations, t20, is measured using a
stopwatch. This step is then repeated. The average
of t20 is calculated. Then, the period of oscillation,
t
T is determined by using T = 20 .
20

4 Steps 2 and 3 are repeated with plasticine balls of
masses 75 g, 100 g, 125 g and 150 g.
5 A graph of T against m is drawn.

CHAPTER

Mrs Tan and her son Siao Yang are sitting on two
similar swings.

F
O
R
M

SPM
06/P3(B)

Figure 2.23

Mr Tan gives each of them a push while they are at


rest and later tries to stop them in the same position.
Mr Tan finds that it is harder to push as well as to
stop Mrs Tans motion.
Inference
F

Tabulation of data

The inertia of an object depends onO its mass.


R
Hypothesis
M
CHAPTER

Table 2.3

An object with a larger mass has a larger inertia


(represented by a longer period of 4oscillation).
Aim

Mass of Time of 20 oscillations, Period of


load,
t20 (s) oscillation,
t
m (g) t t Average
T = 20 (s)
1
2
20

To investigate the relationship between mass and


inertia using an inertia balance
Variables

50
75
100
125
150

(a) Manipulated : mass of plasticine, m


(b) Responding : period, T
(c) Fixed
: the stiffness of a hacksaw blade
and the distance of the centre of
the plasticine from the clamp
Notes

Graph

T(s)

A larger inertia means it is more difficult for the


load to start or stop, thus making the time for one
oscillation, T longer. Therefore, the period of
oscillation represents the inertia.
Apparatus/Materials

Experiment 2.1

Hacksaw blade, G-clamp, stopwatch, and plasticine


balls of mass 50 g, 75 g, 100 g, 125 g and 150 g.
Arrangement of apparatus

m (g)

Figure 2.25

Conclusion

hacksaw blade

The graph of T against m in Figure 2.25 shows that


the period increases with the mass of the load i.e. an
object with a larger mass has a larger inertia.
The hypothesis is valid.

G-clamp

plasticine

Figure 2.24
Forces and Motion

62

63

Forces and Motion

Animals such as dogs and


cats shake their bodies
vigorously to dry their
wet fur. The droplets of
water on the fur tend to
continue in motion when
the fur are reversed in
direction during shaking.
As a result, water droplets
are separated from the fur
and fall off.

Sumo wrestlers are heavy.


This is an advantage because the
larger inertia forms a greater
resistance to his opponent who is
trying to topple him.

When chased by a bull, run in a zigzag pattern.


The larger inertia of the bull makes it more
difficult for the bull to turn around
continuously.

Effects of inertia

CHAPTER

When the branch of an apple tree is shaken, the


apples fall to the ground. The apples which are
stationary tend to remain at rest when the branch
is shaken. As a result, the stalks are strained and
the apples break away from the branch.

Applications
of the
concept of
inertia

If the bottom of the handle of a hammer is knocked


downwards against a hard surface, the loose head of the
hammer tightens in its wooden handle. The hammer head
continues with its downward motion after the handle has
come to a stop. This causes the upper part of the wooden
handle to slot deeper into the
hammer head.

F
O
R
M
4

Droplets of water on
a wet umbrella can be
spun off if the
umbrella is rotated
vigorously and
stopped it abruptly.
Owing to inertia,
water droplets on the
umbrella continue to
move even though the
umbrella has stopped
spinning.

The chilli sauce in a bottle


can be forced out easily by
turning the bottle upside
down and giving it a quick
downward shake, followed
by a sudden stop. The sauce
continues its downward
movement due to its inertia
when the bottle is stopped
causing the sauce to be forced
out of the bottle.

Ways of Reducing the Negative Effect of Inertia

SPM
07/P2

1 If a car crashes while travelling at a fixed velocity of 30 m s1, for


example, the inertia of the passengers causes them to continue moving
at 30 m s1 until a force acts to change this speed. This is a
dangerous situation. Upon impact, the passengers will crash into
the parts of the car immediately in front of them and suffer injuries
(Figure 2.26).
2 Ways to reduce the negative effect of inertia in a car:
(a) Safety belt

Figure 2.26
F4/2/30

F
O
R
M
4

(b) Airbag system

CHAPTER

Seat belts secure the driver


and passengers to their seats.
When the car stops suddenly,
the seat belt provides the
external force that prevents
the driver or passengers from
being thrown forwards.

CHAPTER

F
O
R
M

Airbags mounted on the


dashboard or steering wheel
inflate automatically when a
collision occurs. This prevents
the driver or passengers from
crashing into the dashboard.

3 Lorries that carry heavy loads utilise the following features to reduce the negative effects of inertia:
(a) Strong structure behind the drivers cabin

If a loaded lorry stops abruptly, its heavy load, for


example, timber logs, will continue to move
forward towards the drivers cabin because of its
massive inertia. A strong iron structure between
the drivers cabin and the load ensures the drivers
safety.

strong iron structure

F4/2/31 to reduce its inertia


(b) Subdivision of the mass

The figure shows a liquid product being carried in


three separate compartments instead of one. This
reduces the effect of inertia of the liquid on the
walls of each container if the lorry stops suddenly.

(c) Fastening of the objects to the carrier

Big items such as furniture or large electrical


appliances (the objects) must be securely fastened
to the lorry (the carrier) so that the objects become
part of the carrier. This will ensure that the objects
and the carrier always move and stop together.
The objects will not fall off the lorry when the lorry
starts moving, or move forwards when the lorry
stops suddenly.

Forces and Motion

64

SPM
Clone

10

R
Pushed to the right
Pushed to the right
Pushed to the left
Pushed to the left

S
Pushed to the left
Pushed to the right
Pushed to the right
Pushed to the left

Comments
The collision of the trolleys causes trolley P to slow
down while trolley Q to start moving. Due to
inertia, R continues to move fast to the right, and
appeared to be pushed to the right while S, reluctant
to move along with trolley Q, appeared to be
pushed to the left.

Direction of movement of P
Key :


A
B
C
D

Stationary

Right
Left

What happens to wooden block R and S?

Answer A

2.3
1 In a bus moving with a uniform speed in a straight
line, a boy drops a steel sphere from rest outside the
window. He observes that the steel sphere drops
vertically downwards. Explain.

4 A circus strongman slams a hammer and breaks a


brick over the hand of a clown. The clown feels no
pain. Why?
(Caution: Please do not attempt this at home.)

2 A durian is originally placed in a lorry as shown in


Figure (a). When the lorry starts to move with forward
acceleration, the durian rolls backwards. Later, when
the lorry stops suddenly, the durian rolls forwards as
shown in Figure (b). Explain the observations.
hammer

brick

hand
on floor

(a)
(a)
(a)

5 Suria runs along a track from P to R through Q while


holding a pail full of water. At which points of the
track will more water likely to be spilt? Give your
explanation.

(b)
(b)
(b)

3 The figure below shows two blocks of the same


dimensions but of different materials. One is heavier
F4/2/32
than the other. WithoutF4/2/32
lifting the two blocks, explain
one way to identify the heavier block.

F4/2/34

65

Forces and Motion

The diagram shows two trolleys P and Q with two


identical wooden blocks R and S placed on them.
Trolley P moves and collides with the stationary
trolley Q.

CHAPTER

F
O
R
M
4

2.4

Analysing Momentum

2.4

Analysing Momentum

1 If a loaded lorry and a car are moving at


the same speed, it is more difficult for
the lorry to stop (Figure 2.27).
2 This is because the lorry possesses a
physical quantity, momentum, more
than the car.
3 All moving objects possess momentum.
4 Activity 2.2 will help you to gain the idea
of momentum by comparing the effects
of stopping two objects.

The steel ball


released from a
greater height strikes
the plasticine at a
greater velocity.

Figure 2.27

One steel ball and one wooden ball of the same diameter, 2 slabs of plasticine.

Arrangement of apparatus
Note

To compare the effects of stopping two objects in motion

ity 2.2


Apparatus/Materials

steel

steel
F

O
R
M
4

Activ

CHAPTER

CHAPTER

F
O
R
M

What is Momentum?

Note

steel
100 cm

wood

50 cm

50 cm
plasticine

Although both balls


have the same
diameter, the steel
ball has a greater
mass than the
wooden ball.

(a) (b)
Figure 2.28

Activity 2.2

(b) Two objects of different masses moving at


(a) Two objects of the same mass moving atF4/2/96
the same velocity
different velocities
Procedure
1 A steel ball is first released from a height of
50cm and then from 100 cm above a slab of
plasticine as shown in Figure 2.28(a).
2 The depths and sizes of the cavities caused
by the steel ball on the slab are observed and
compared.

Procedure
1 A steel ball and a wooden ball of the same
diameter are released from a height of 50 cm
above a slab of plasticine as shown in Figure
2.28(b).
2 The depths and sizes of the cavities formed
are observed and compared.

Observations
The depth and size of the cavity caused by the
steel ball released from a greater height is deeper
and larger.

Observations
The depth and size of the cavity formed by the
steel ball is deeper and larger.

Conclusion
The moving balls produce an effect on the plasticine which is there to stop the motion. The greater the mass
or the velocity of the moving object is, the greater is the effect (the depth and size of the cavity), the greater is
the momentum.
Forces and Motion

66

Linear Momentum
1 Activity 2.2 also shows that it is always harder to stop a massive object
moving at a high velocity.
2 The above activity serves to explain a concept in physics called
momentum.
3 The linear momentum, p, of a
m
mass, m, moving at a velocity,
v
v, is defined as the product of
mass and velocity.

Momentum and inertia are


not the same.
Momentum Inertia
Inertia depends solely on
mass while momentum as
mass in motion depends on
both mass and velocity.

Figure 2.29
F4/2/36

Unit=kilogram metre per second


=kg m s1

CHAPTER

Momentum =Mass Velocity



p =mv

4 The unit of momentum is kg m s1.


5 Momentum is a vector quantity with the same direction as velocity.
6 If the direction to the right is denoted as positive, an object moving to
the right possesses a positive momentum while an object moving to
the left will have a negative momentum.

15
A ball of mass 0.8 kg strikes a wall at a velocity of
10 m s1 and rebounds at 6 m s1.
What is its momentum
(a) before it strikes the wall, and
(b) after the rebound?
Solution

(a) Take the direction to the right as positive.


Momentum of the ball before striking the wall, p1
=mv
=0.8 10
=8 kg m s1

10 m s1

6m

(b) Momentum of the ball after rebound, p2


=mv
=0.8 ( 6)
= 4.8 kg m s1
Since direction to the
right is positive, direction
to the left is negative.

s1

4 To be precise, the Principle of conservation of


momentum is true for a closed system.
A closed system is one where the sum of
external forces acting on the system is zero.
5 The principle shall be discussed in two
situations as shown in Table 2.4.

Conservation of Momentum
1 The term conservation is used if the total
amount of matter or quantity remains the same
before and after the occurence of an event.
2 The Principle of conservation of momentum
states that:

Table 2.4


The total momentum of a system
is constant, if no external force
acts on the system.
3 An example of an external force is friction.
67

A collision

An explosion

The total momentum of the


objects before a collision
equals that after the
collision.

The sum of the


momentums
remains as
zero after an
explosion.

Forces and Motion

F
O
R
M
4

SPM
06/P1

Collisions

SPM
08/P1

There are two types of collisions.


Collisions
Elastic collisions

Inelastic collisions

Two objects collide and move apart after a


collision.
2

u1

m2

m1

u2

m1

m2

v1

v2

Momentum is conserved.
Total energy is conserved.
Kinetic energy is conserved.

m1

u2

m2
v

Momentum is conserved.
Total energy is conserved.
Kinetic energy is not conserved:
The total kinetic energy after the collision is less
than the total kinetic energy before the collision.
Formula:

m1 u1 + m2 u2 = m1 v1 +Om2 v2
R
M
4

m1 u1 + m2 u2 = (m1 + m2) v

Formula:

m2

u1

CHAPTER

m1

CHAPTER

F
O
R
M

Two objects combine and stop, or move together


with a common velocity after a collision.

Most collisions are inelastic as a significant amount of kinetic energy is converted to other forms of energy
during the collision. However, the collisions involving two objects that bounce off one another with little
deformation during collision are approximate elastic collisions. Collisions between air molecules are elastic.
Examples of approximate elastic collisions are as follows:
Approximate elastic collisions

Physics

Blog
Collision of steel balls of equal mass

Collision of two snooker balls of equal mass

Before collision

After collision

Before collision

After collision

stationary

QR S T

P QP
RQ
S RS

Ball P is pulled to the side and then released so as to


fall back and strike ball Q. It is observed that ball P
stops, but ball T swings out to the same height from
which the ball P was released.
This shows that ball T possesses the same amount of
momentum and kinetic energy as P before it struck
ball Q.

Forces and Motion

stationary

Ball P is hit and moves with velocity u. It is observed


that ball P stops when it collides with ball Q which
moves away with velocity u.
This shows that ball Q, after the collision, has the
same amount of momentum and kinetic energy as
ball P before the collision.

68

Activ

To verify the principle of conservation of momentum in


(a) elastic collisions, and (b) inelastic collisions

ity 2.3

Apparatus/Materials
Ticker-timer, 12 V a.c. power supply, runway, 4 trolleys, wooden block, ticker tape, cellophane tape,
and plasticine.
(A) Elastic collision
Arrangement of apparatus
spring-loaded piston
ticker-timer trolley A
ticker tape
mA

friction-compensated
runway

mB

CHAPTER

12 V a.c.
power supply

trolley B

wooden
block

F
O
R
M

Figure 2.30

Procedure
1 The apparatus is set up as shown in Figure 2.30.
2 The runway is adjusted so that it is friction-compensated.
3 Two trolleys of equal mass are used. Trolley A with a spring-loaded piston is placed at the higher end of the
runway while trolley B is placed halfway down the runway.
4 A ticker tape is attached to trolley A and another to trolley B. Both ticker tapes are allowed to pass through the
ticker-timer.
5 The ticker-timer is switched on and trolley A is given a slight push so that it moves down the runway at
a uniform velocity and collides with trolley B.
6 After the collision, the two trolleys move separately.
7 From the ticker tapes, the velocities of trolleys A and B before and after the collision are calculated.
8 Assuming that the mass of each trolley is 1 unit, the momentum before and after the collision is calculated
and recorded in a table.
9 The experiment is repeated using:
(a) 1 trolley to collide with 2 stationary stacked trolleys,
(b) 2 stacked trolleys to collide with 1 stationary trolley.
Results

after
collision

during
collision

before
collision

x2
vA =

(b) Trolley B

direction
of motion

x1
x2
0.2

after
collision

uA =

during
collision

x1
0.2
before
collision

Activity 2.3

1 Ticker tapes obtained:


(a) Trolley A

direction
of motion
x3

uB = 0

x
vB = 3
0.2

Figure 2.31

69

Forces and Motion

Tabulation of data
Table 2.5


Before collision
After collision

Initial total
Final total
mA mB uA momentum, vA vB momentum,
mA uA mA vA + mB vB

1
2
1

Disscussion

Conclusion
From Table 2.5, it is found that:

F
O
R
M

1 The spring-loaded piston acts as a springy buffer in the collision in order to make the trolley bounce off the
other one.
2 Strictly speaking, this collision is not a perfect elastic collision as part of the kinetic energy of the colliding
trolley changes to sound or heat energy during the collision.

CHAPTER

CHAPTER

F
O
R
M

1
1
2

Total momentum
before collision = Total momentum after collision
4
The principle of conservation of momentum is verified.

(B) Inelastic collision


Arrangement of apparatus
ticker
tape

ticker-timer

plasticine

trolley A

trolley B
friction-compensated
runway

12 V
a.c. power
supply
wooden block

Figure 2.32

Activity 2.3

Procedure
1 Some plasticine is pasted onto trolleys A and B (both without a spring-loaded piston) as shown in Figure 2.32.
2 A ticker tape is attached to trolley A only.
3 The ticker-timer is switched on. Trolley A is given a gentle push so that it moves down the runway to collide
with trolley B which is stationary halfway down the runway.
4 After the collision, trolley A attaches itself to trolley B and they move together.
5 From the ticker tape obtained, the velocity of trolley A before the collision, and the common velocity of
trolleys A and B after the collision are determined. The initial velocity of trolley B is zero.
6 The experiment is repeated using:
(a) 1 trolley to collide with 2 stationary stacked trolleys,
(b) 2 stacked trolleys to collide with 1 stationary trolley.
Forces and Motion

70

Results
Ticker tape obtained:

during
collision

before
collision

after
collision

direction
of motion
v =

x2
x2
0 .2

u =

x1
x1
0.2

Figure 2.33

Tabulation of data

After collision
Final
Final
Final total
mass,
velocity, momentum,
(mA + mB) v (mA + mB) v

1+1=2
1+2=3
2+1=3

1
1
2

CHAPTER


Before collision
Initial
Initial
Initial total
mass, velocity, momentum,
mA u (mA u)

Table 2.6

F
O
R
M

Conclusion
From Table 2.6, it is found that:
Total momentum before collision = Total momentum after collision
The principle of conservation of momentum is verified.

16
An astronaut of mass 90 kg moves at a velocity of 6 m s1 and bumps into a stationary astronaut
of mass 100kg. How fast do the two astronauts move together after collision?
90 kg

100 kg
6 m s1

(a) Before collision

at rest

90 kg

100 kg

(b) After collision


Figure 2.34

Activity 2.3

Solution
This is an inelastic collision.

Total momentum before collision = Total momentum after collision
m1 u1 + m2 u2 = (m1 + m2)v
(90 6) + (100 0) = (90 + 100)v
v is the common velocity
540
v =
of the two astronauts.

190
= 2.8ms1
The two astronauts move at the speed of 2.8 m s1 after the collision.
71

Forces and Motion

3ms

7ms

17
A 50 kg skater is moving due east at a speed of 3ms1 before colliding into another skater of mass
60kg moving in the opposite direction at a speed of 7 m s1. After the collision, the two skaters hold
on to each other. In which direction will they move? What is the speed of the two skaters?
1

3ms

7ms


(a) Before collision

(b) After collision

v=?

Figure 2.35

Solution

Total momentum before collision = Total momentum after collision
m1 u1 + m2 u2 = (m1 + m2)v
50 3 + 60 (7) = (50 + 60) v
F

150 420 = 110v
1
O
v = 2.5ms
v is the common velocity
R
Momentum to the east takenMas positive, so
momentum to the west is negative.
4

of the two skaters.

CHAPTER

CHAPTER

F
O
R
M

v=?

The two skaters will move to the west at a speed of 2.5m s1.

18
A trolley of mass 3 kg moving at a velocity of 2ms1 collides with another trolley of mass 0.5 kg
which is moving at a velocity of 1 m s1 in the same direction. If the 0.5 kg trolley moves at a
velocity of 2.5 m s1 in the same direction after the collision, what is the velocity of the 3 kg trolley?
Solution
2 m s1
3 kg

3 kg

0.5 kg

0.5 kg


(a) Before collision

(b) After collision

The collision is elastic.



Total momentum before collision = Total momentum after collision
m1 u1 + m2 u2 = m1 v1 + m2 v2
3 2 + 0.5 1 = 3 v + 0.5 2.5

6.5 = 3v + 1.25
3v = 6.5 1.25
5.25
v =

3
= 1.75ms1
The 3 kg trolley moves at a velocity of 1.75ms1 in its original direction.

Forces and Motion

2.5 m s1

v=?

1 m s1

72

SPM
04/P2

Momentum and Explosions


1 Rifle
2

The explosion creates a backward momentum on


the rifle. This causes the rifle to recoil backwards.

Total momentum of the rifle


and the bullet is zero as they
are stationary.

ard
kw tum
bac men
mo riflev 2
on

m1

v1
forward momentum
on bullet

(a) Before explosion

When the rifle is fired, the explosion of


the gunpowder forces the bullet out of
the barrel. A momentum in the forward
direction is created.

(b) After explosion


Figure 2.36

F4/2/38

2 Air escapes from a deflating balloon


upward
momentum
balloon
stationary
(u = 0)

m2

v2

Total momentum
of the balloon is
zero as it is
stationary.

v1

m1
downward
momentum

(a) Before
explosion
(a) Before
explosion

The balloon shoots upwards,


moving with an upward momentum.

Air has mass and moves with a velocity.


This creates a momentum in the
downward direction.

(b) After explosion


(b) After explosion
Figure 2.37

3 An explosion is a closed system which does not involve any external forcethat is, the total
momentum is conserved in an explosion.
Total momentum before explosion = Total momentum after explosion
0 = m1v1 + m2v2
Rearranging the formula:
m1v1 = m2v2
where v1 and v2 are of opposite directions.
If we ignore the direction,
m1v1 = m2v2
as the two momenta have the same magnitude.

Momentum to the left = Momentum to the right


Momentum upwards = Momentum downwards
Momentum forwards = Momentum backwards

Bear in mind that the two velocities


are in opposite directions.

73

Forces and Motion

CHAPTER

m2

F
O
R
M
4

Activ

To verify the principle of conservation of momentum in an


explosion

ity 2.4

Apparatus/Materials
4 trolleys, 2 wooden blocks, a hammer, and a metre rule.
Notes
The positions of the wooden blocks are adjusted so that each trolley collides with the corresponding
wooden blocks at the same time, t. From the equation d = vt, the magnitude of the velocity v is
directly proportional to the distance d, i.e., v d (if t is constant). Thus, the distance d travelled by the
trolley represents the velocity of the trolley.

Procedure

(a) Before explosion


release pin
trolley B

trolley A

mB

wooden
block

mA

dA

dB

(b) After explosion


vB

1 The apparatus is arranged as shown in Figure


2.38 (a).
2 Two trolleys A and B of equal mass are placed in
contact with each other on a smooth surface. The
spring-loaded piston in trolley B is compressed.
3 The release pin on trolley B is given a light tap to
release the spring-loaded piston which then
pushes the trolleys apart. The trolleys collide with
the wooden blocks.
4 The experiment is repeated and the positions
of the wooden blocks are adjusted so that both
trolleys collide with them at the same time.
5 The distances dA and dB are measured and
recorded.
6 The experiment is repeated using
(a) 1 trolley with 2 stacked trolleys,
(b) 3 stacked trolleys with 1 trolley.

F
O
R v
A
M

springloaded
piston

CHAPTER

Arrangement of apparatus

CHAPTER

F
O
R
M

Figure 2.38

Tabulation of data
The results of the experiment are recorded in Table 2.7.
Table 2.7

Before explosion
Initial total
momentum
0
0
0

Mass of
trolley A,
mA

Mass of
trolley B,
mB

1
1
3

1
2
1

After explosion
Velocity of
Velocity of
Final total
trolley A,
trolley B,
momentum,
dA
(dB) mA dA + mB (dB)
0
0
0

Activity 2.4

Discussion
1 Total momentum before explosion = 0 (because both trolleys are stationary)
Total momentum after explosion = mAdA + mB (dB)
2 Table 2.7 shows that mA dA + mB (dB) = 0
Total momentum after explosion = Total momentum before explosion
Conclusion
Momentum is conserved in an explosion.
Forces and Motion

74

as d represents v,
and v is a vector.

19
Jane and John go ice skating. With their skates on,
Jane and John push against each other on level ice.
Jane, of mass 50 kg, moves away at a velocity of
3 m s1 to the right. What is Johns velocity if he is
75 kg?

Figure 2.39

CHAPTER

Solution
This is a closed system since the external force, i.e.,
friction, is negligible.
Let the velocity of John be v.

Total momentum = Total momentum

after explosion before explosion
m1 v1 + m2 v2 = 0
50 3 + 75v = 0
150
v = _____
The minus sign indicates that
75
John moves to the left,
opposite to Janes motion.
= 2 m s1
Alternative Method
omentum to the right = Momentum to the left
M
m1 v1 = m2 v2
Just ignore the sign as the two
50 3 = 75 v
skaters are moving in opposite
v = 2 m s1
directions.
John moves to the left at a velocity of 2m s1.

20
Figure 2.40 shows trolley A, with a weight attached, placed in contact with trolley B on a smooth surface.
weight attached to trolley A
release pin
A

d1 = 1 d2
3

d2

Figure 2.40

Solution
Applying m1 d1 = m2 d2:
1
m1 d2 = m d2
3

When the release pin of trolley B is tapped lightly, the


spring-loaded piston pushes the two trolleys to move
in opposite directions. The two trolleys touch the
wooden blocks simultaneously. Trolley A moves onethird the distance moved by trolley B. Given that the
two trolleys are of equal mass, m kg, find the mass of
the weight, in terms of m, that is attached to trolley A.

m1 is the total
mass of trolley A
and the weight.

m1 = 3m
The mass of the weight, m
= 3m m
= 2m

75

Forces and Motion

F
O
R
M
4

21

v' = ?

Figure 2.41

Solution

m1 v1 + m2 v2 = 0

(m v) + (3m v) = 0
3mv = mv
1
v = v
3

1
1
The speed of the boat = v m s
F
3
O
R
M

v m s1

CHAPTER

CHAPTER

F
O
R
M

Harfeez alights a boat at a


velocity of v m s1 and lands
on the dock.
The boat bounces backwards
with a speed of v.
If the masses of Harfeez and
the boat are m and 3m
respectively, find the speed
of the boat (ignoring friction
due to water).

4
Application of the Conservation
of Momentum

1 Rocket
5

1 A rocket carries liquid


hydrogen and liquid
oxygen.

2 The mixture of hydrogen


fuel and oxygen burns
vigorously in the
combustion chamber.

5 In accordance with the


principle of conservation of
momentum, the rocket gains
a forward momentum and
moves forwards at high
velocity.

1
liquid
hydrogen
(as fuel)
1
liquid
oxygen

3 The gases formed


expand rapidly and are
forced to discharge
through the exhaust
nozzle at a high
velocity.

2
combustion
chamber

4 A backward momentum
is created.

3
hot exhaust
gases at
high velocity
4

Figure 2.42
F4/2/41
Forces and Motion

76

2 Jet engine
2

Air from the atmosphere is drawn into


the engine and compressed by a
compressor before it is forced into the
combustion chamber at high pressure.

The hot gases formed expand rapidly


and are forced out of the nozzle at
high speed through the turbine which
rotates the compressor.

In the combustion chamber,


kerosene fuel burns vigorously
with the compressed air.

4 turbine blade

3 combustion
chamber

2 compressor

jet of
exhaust
gases

1
air
intake

CHAPTER

F
O
R
M
4

nozzle
6 moves forwards

fuel injector

6
5

In accordance with the principle of the conservation of


momentum, a forward momentum for the engine is produced.
The plane thus flies forwards.

The ejected high-speed exhaust gases create


a backward momentum.

Figure 2.43

Demonstrations to Show the Principle of Propulsion in Jet fuel


Engines
injector and Rockets

6
moves
forward

The following demonstrates the principle in rockets and jet engines.


Jet engine
2 5
thread
jet of
exhaust
gases

3
front

elastic
band

Rocket
5 upward

1
cylindrical
weight

air

1 A weight is placed on the trolley.


F4/2/43a
2 As the thread
is cut, the weight is
thrown to the back with a
momentum.
3 This in turn creates a forward
momentum which causes the
trolley to move forwards.

water
plastic tube

3
bicycle
pump

nozzle
back
4 turbine blade

3 combustion
chamber

1
air
intake

1
plastic
bottle

rubber stopper 2
downward
4

tripod stand

2 compressor

1 One-third of a plastic bottle is filled of water.


F4/2/43b
2 The bottle is then plugged
F4/2/42tightly with a rubber stopper with a
plastic tube.
3 Air is pumped into the bottle.
4 When the pressure of the air in the bottle is sufficient, it will
push the stopper out together with the water. A downward
momentum is produced.
5 This in turn creates an upward momentum which moves the
plastic bottle upwards.
77

Forces and Motion

The large volume of water that rushes out from a water hose with a very high speed has a large momentum. In
O
accordance with the principle of theR conservation of momentum, an equal and opposite momentum is created causing
the fireman to fall backwards. Thus,M
several firemen are needed to hold the water hose.
4

CHAPTER

The shower of burning fragments from an exploding fireworks launched into the sky is governed by the principle of the
F4/2/44athat the total momentum is conserved.
conservation of momentum. The symmetrical pattern indicates

CHAPTER

F
O
R
M

The principle of the conservation of momentum occurs in nature the squid uses it to propel itself in the water.
The squid moves forward by discharging a jet of water from its body. An equal and opposite momentum created thus
propels the squid in the opposite direction.

SPM
Clone

11

The diagram shows three identical theme park coins


on a straight line, with Q and R touching each other.
P is given a velocity and collides Q.
Q
P

token

token

token

A Moves

Stationary

Stationary

B Stationary

Stationary

Moves

C Moves

Moves

Stationary

D Moves

Stationary

Moves

Comments
The momentum of P is totally transferred to R
through Q.
Therefore, P stops and R moves with the velocity of
P before collision while Q remains stationary.

What will be observed?

Answer B

Forces and Motion

78

2.4
5 A butterfly rests on a leaf floating on the surface of a
pond.

1 A pigeon of mass 120 g is flying at a velocity of


2ms1. What is its momentum?

5 cm s1

2 A bull of mass 250 kg is moving at a momentum of


750 kg m s1. Find its velocity.
3 During training, Othman fires a pistol of 1.5 kg mass.

stationary

(a)
Before
(a)
Beforeshooting
shooting

6 Boat A and boat B are moving at a speed of 2 m s1


and 1 m s1 respectively before the two collide head
on. The masses of boats A and B (including the
passengers) are 150 kg and 250 kg respectively.

30 g

1.5 kg

(b) After
shooting
(b) After
shooting
(a) Before collision

A bullet with a mass of 30 g is released at a


velocity of 300 m s1. What is the recoil velocity of
the pistol?

0.5 m s1

v=?

4 Hizam and his son Jamal are at an ice rink.


A

(b) After collision


If boat A bounces back with a velocity of 0.5 m s1,
what is the velocity of boat B?
7 Sau Fei and Siew Ling, each with a mass of 60 kg and
49.5kg respectively, are standing at rest on an ice
rink. Sau Fei throws a ball of mass 0.5 kg towards Siew
Ling.

Jamal with a mass of 20 kg is moving at a velocity of


2ms1 while Hizam with a mass of 60 kg, is directly
behind Jamal and moving at 6 m s1. Hizam decides
to pick Jamal up and continues moving without
stopping. Determine the final velocity of Hizam and
Jamal.

What is the recoil velocity of Sau Fei if the velocity of


the ball is 8 m s1? What is the velocity of Siew Ling
after she receives the ball?

79

Forces and Motion

CHAPTER

The butterfly then starts moving to the tip of the leaf


at a speed of 5 cm s1 relative to the water. The leaf,
in accordance with the principle of the conservation
of momentum, moves at 3 cm s1 relative to the
water in the opposite direction. If the mass of the leaf
is 8 g, determine the mass of the butterfly.

3 cm s1

F
O
R
M
4

2.5

Understanding the Effects of a Force

2.5

Understanding the Effects of a Force

What is Force?
F

A force is a push
or a pull.

Pressing a switch
F

F
F

COKE

10

1 When you push or pull on an object, you need to know


(a) the strength or magnitude of your
force, and
F4/2/45
(b) the direction in which you are pushing or pulling.
F
2 Therefore, force is a vector quantity
since it has both magnitude and
O
direction.
R
3 A spring balance which isM used to measure the weight (or the
gravitational pull on the object) can be used to measure the magnitude
of a force. For example, the4 force pulling a wooden block can be
measured as shown in Figure 2.44.

Kicking a football

Stretching a chest expander

Pulling off the ring of a soft drink tin

spring balance

CHAPTER

Figure 2.44

The Effects of a Force


The shape of a tube of toothpaste
changes when you press on it.

A spring lengthens or compresses


when you stretch or compress it.

A plastic ruler can be bent when


a force is exerted on it.



1 A force can change the shape of an object (deformation of an object).
F4/2/46a

F4/2/46c

F4/2/46b

CHAPTER

F
O
R
M

Lifting objects

Effects of force

2 A force can change the original state of motion (either at rest or in motion) of an object.
(a) To move a stationary object
(b) To stop a moving object
(c)To change the direction of
motion of an object


A pushing force is required to
move a F4/2/46d
stalled car.
Forces and Motion


F4/2/46e
Friction will stop
the bicycle
when the boy stops pedalling.
80

A tennis player hits a fast-moving


F4/2/46f
tennis ball to return
it to his opponent.

The effects of force are applicable in the games below.


(a) Football

1
Ben kicks the football to Sani from a corner
during a football match. He needs to apply
a force to make the stationary ball move.
Also, as his boot is in contact with the ball,
the force applied deforms the ball.

3
Hazuri manages to catch the ball.
He needs to apply a force to stop the ball.

Hazuri

CHAPTER

Ben

Sani

2
Sani's header produces a force on the ball to change its direction.
F4/2/47a

(b) Baseball

3
If the ball is not hit, it is
stopped by the catcher.

2
The force from the bat deforms the ball
and reverses the direction of the ball.
F4/2/47b

1
A pitcher applies a force when
he throws the baseball.

Relationship between Acceleration, Mass and Force


1 Consider two identical speedboats A and B at
rest on the sea. Different engine thrusts are
applied to accelerate the boats. However,
speedboat A which applies more engine thrust
increases the velocity at higher rate.
2 Consider two identical speedboats, P and Q
with P heavily loaded. When the same engine

thrust is applied, speedboat P accelerates


slower than Q.
3 Thus, the three quantities: acceleration, mass
and force, are related.
4 The relationship between the three quantities
can be determined in the following experiments.
81

Forces and Motion

F
O
R
M
4

2.2

Arrangement of apparatus

B (greater engine capacity)

B
engine
ticker-timer stretched elastic cord (1 unitgreater
of force)
ticker tape
thrust (force)

CHAPTER

F
O
R
M

Relationship between acceleration and force applied on a constant mass


elastic cord attached to the trolley and stretched to a
Situation

fixed length represents one unit of force acting on the


Figure 2.45(a) shows car A and car B of the same
trolley.
mass at the same starting line. Car B is a sports car.
The engine capacity of sports car B is much bigger
Apparatus/Materials

than car A. (A car with a bigger engine capacity can


Trolley,
3
identical
elastic
cords,
runway, tickerprovide greater engine thrust.)
timer,
carbonised
ticker
tape,
cellophane
tape, 12V
Figure 2.45(b) shows that sports car B has built up a
a.c.
power
supply
and
a
wooden
block.
higher velocity than car A after 3 seconds.

A
a.c. power
supply

CHAPTER

trolley
wooden block

F
O
B
R
M

(a) t = 0

friction-compensated
runway

(a)

F4/2/48
greater
engine
thrust
(force)

two stretched elastic cords (2 units of force)


[same extension as (a)]

(b) t = 3 s
Figure 2.45
(b)
Figure 2.46

Can you make an inference about this situation?


Inference

The acceleration of an object depends on the force


which acts on it.

Procedure

1 A friction-compensated inclined runway is


prepared.
2 The apparatus is then set up as shown in Figure
2.46(a).
3 The ticker-timer is switched on and the trolley is
pulled down the runway by an elastic cord
attached to the hind post of the trolley.
4 The elastic cord is stretched until the other end
is level with the front end of the trolley. The
length is maintained as the trolley runs down the
runway.
5 The ticker tape obtained is cut into strips of
10-tick. A tape chart is constructed and the
acceleration, a, is determined.
6 The experiment is repeated with 2, and 3 elastic
cords to double and triple the pulling force to the
same constant extension as when one elastic cord
is stretched.

Hypothesis

When the mass of an object is constant, the greater


the force applied, the greater its acceleration.
Aim

To investigate the relationship between acceleration


and the force applied on a constant mass.

Experiment 2.2

Variables

(a) Manipulated : force applied, F


(b) Responding : acceleration of trolley, a
(c) Fixed
: 
mass of trolley, m, and the
stretching force of an elastic cord
Notes

The force in this experiment is the stretching force in


an elastic cord used to pull the trolley. A length of
Forces and Motion

82

Results

Ticker tape chart:


Velocity (cm per tentick)

Velocity (cm per tentick)

the gradient
is doubled

(a) One elastic cord


(b) Two elastic cords
(c) Three elastic cords
(a) (1 unit of force) (b) (2 units of force) (c) (3 units of force)
Figure 2.47

1 The gradient of the line passing through the dot at the top of each successive strip increases with the number
of cords.
2 This indicates an increase in acceleration when the force is increased.
Tabulation of data

Graph

F
O
R
M
4

1 The graph of a against F is a straight line passing


through the origin as shown in Figure 2.48.
This shows that: a F (if m is constant).

Table 2.8

Units of force

applied, F
1
2
3

the gradient
is tripled

CHAPTER

Velocity (cm per tentick)

Acceleration,
a (m s2)
a1
a2
a3

acceleration, a

force, F

Figure 2.48

Conclusion

The acceleration of an object is directly proportional to the force applied if the mass is constant.
The hypothesis is valid.

2.3

acceleratio

Relationship between acceleration and the mass of an object under a constant force
Situation

B
full-loaded
lorry
empty
lorry

B
B

F
A

A
F



(a)
t = 0
Figure 2.49
F4/2/49

83

Experiments 2.2 & 2.3

Figure 2.49(a) shows two similar lorries, A and B in front of a traffic light. When the light turns green, both drivers
step on the accelerator simultaneously with the same pressure to provide the same engine thrust, F.
Figure 2.49(b) shows that within 3 seconds, the empty lorry has built up a higher velocity than the heavy one.

(b) t = 3 s

Forces and Motion

5 The elastic cord is stretched until the other end


is level with the front end of the trolley. The
length is maintained as the trolley runs down the
runway.
6 The ticker tape obtained is cut into strips of
10-tick. A tape chart is constructed and the
acceleration, a, is determined.
7 The experiment is repeated using 2 trolleys (with
a second trolley stacked on the first trolley) and
3 trolleys. The elastic cord is stretched to the
same fixed length as in the first experiment.
Results

Inference

The acceleration of an object depends on its mass.


Hypothesis

(a) Manipulated : mass of trolley, m


(b) Responding : acceleration of trolley, a
(c) Fixed
: force applied by an elastic cord, F
Notes

Ticker tape chart:


Velocity (cm per tentick)

(a) The mass in this experiment is represented by the


number of identical trolleys used.
(b) The constant force is applied by stretching the
elastic cord with the same extension for each
repetition of the experiment. F
O
Apparatus/Materials
R

(a) One trolley (1 unit of mass)


Velocity (cm per tentick)

CHAPTER

CHAPTER

F
O
R
M

When the force applied on an object is constant, the


greater the mass of the object, the smaller its
acceleration.
Aim

To investigate the relationship between acceleration


and the mass of an object under a constant force.
Variables

Ticker-timer, 12 V a.c. power M


supply, 3 trolleys,
elastic cord, runway, wooden block, ticker tape and
4
cellophane tape.
Arrangement of apparatus
ticker tape

the gradient
is halved

(b) Two trolleys (2 units of mass)

ticker-timer stretched elastic cord

Velocity (cm per tentick)

friction-compensated runway

a.c. power
supply

the gradient is
one third of (a)

trolley
wooden block

(a)
F

(c) Three trolleys (3 units of mass)

stretched elastic cord


[same extension as in (a)]

Figure 2.51

The gradients of the lines joining the uppermost dots


for successive strips decrease as the number of
trolleys used increases. This indicates that
acceleration decreases as mass increases.

2 units of mass

(b)
Figure 2.50

Tabulation of data

Experiment 2.3

Procedure

1 A friction-compensated inclined runway is


prepared.
2 The apparatus is then set up as shown in Figure 2.50.
3 A ticker tape is attached to the trolley and passed
through the ticker-timer.
4 The ticker-timer is switched on and the trolley is
pulled down the inclined runway by an elastic
cord attached to the hind post of the trolley.
Forces and Motion

Table 2.9

Mass of
trolley, m
(Number of
trolleys)
1
2
3

84

Inverse
of mass,
1
m

Acceleration,
a
(m s2)

1.00
0.50
0.33

a1
a2
a3

Graph

The graph of a against

acceleration, a

1
shows a straight line
m

passing through the origin as shown in Figure 2.52.


1
This shows that: a (if F is constant).
m
Conclusion

The acceleration of an object is inversely


proportional to its mass when the force acting
on it is constant.
The hypothesis is valid.

CHAPTER

Figure 2.52

Newton's Second Law of Motion

22
A force of 10 N acts on an object of mass 5 kg on a
smooth floor. Find its acceleration.

1 From Experiment 2.2: a F


1
From Experiment 2.3: a
m

a=?
10 N

The two results are combined.


F
a
m

k is a constant.

orF ma
F = kma

F
O
R
M

Figure 2.54

Solution
F = 10 N, m = 5 kg

F = ma

10 = 5a

a = 2 m s2

2 The unit of force is Newton, N.


3 In order to make the formula as simple as possible,
we make k = 1 by defining a force of 1N as:
1 N is the force which gives a mass of 1 kg
an acceleration of 1 m s2.

1 kg

a = 1 m s-2

Fnet = ma is the mathematical expression of Newton's


second law of motion.

1N

This law states:


The net force on an object is proportional to the rate of
change of momentum.

Force
(net)

F = kma
1 N = k 1 kg 1 m s2
k = 1

Fnet

Change in momentum
Time

(v u)
Fnet m
t

Fnet ma
Fnet = kma

F = ma
Newton's second
law of motion

85

The net force on an


object causes the
object with a mass
of m to change its
velocity from u to v.

mv mu
t

a=

vu
t

Forces and Motion

Experiment 2.3

Figure 2.53

23
152
a = 2 30

A car of mass 1200 kg travelling at 15 m s1 comes


to rest over a distance of 30 m. Find
(a) the average retardation, and
(b) the average braking force.

SPM
03/P1

Balanced Forces and Unbalanced Forces


1 In general, there may be several forces acting
on the mass, whether parallel or anti-parallel,
or in different directions.
2 Thus, the force, F, must be replaced with the
net or resultant force when there are several
forces acting on the mass.
F

SPM
08/P1

1 When the forces acting on an object are


balanced, they cancel each other out (that is,
net force = 0).
2 The object then behaves as if there is no force
acting on it.
3 Since Fnet = 0, the acceleration of the object,
a = 0. Thus, the object remains at rest or moves
at constant velocity when there is no net force
acting on it. This is Newtons first law of motion.
4 Examples of balanced forces:

where a is in the direction of the net or


resultant force.
3 However, for simplicity, F = ma is always used,
bearing in mind that F is the net force acting
(a) Balanced forces on a stationary gymnast

SPM
07/P2(C)

Balanced Forces

O
R to use F = ma
Some prefer
where FMis the sum of the
force vectors.
4

Fnet = ma

SPM
05/P1

on the object (whether a single force or several


forces are acting on it).

CHAPTER

CHAPTER

Solution
u = 15 m s1, v = 0 m s1, s = 30 m
(a) Using the formula v2 = u2 + 2as:
0 = 152 + 2a(30)
F
O
R
M

Retardation is
deceleration.

a = 3.75 m s2
Retardation = 3.75 m s2
(b) F = ma
Negative sign indicates
=
1200 (3.75)
retardation force or
opposing force.
=
4500 N
Average braking force = 4500 N

(b) Balanced forces on a car moving at a constant velocity


R

air resistance, G

thrust, T
T = G + Fr
weight, W

reaction force, R
from beam

The weight of the gymnast,


W, is balanced by the
F4/2/143
reaction force, R, from the beam.
The two forces are of equal magnitude but opposite
in direction.
Without the beam (that is, no reaction force), the
gymnast will fall to the ground because of her
weight.

Forces and Motion

friction, Fr

W=R

W=R

There are 3 horizontal forces acting on a car moving


at a constant velocity. The forward thrust, T,
provided by the car engine is balanced by the
frictional force on the wheels and the air resistance.
T = G + Fr
The weight of the car, W, is vertically balanced by
the reaction force, R, from the road.
W=R

86

Balanced forces (Fnet = 0, a = 0)


F1

F2

F net = 0
(as no force
acting on it)

Object in motion
(v 0, and the object is moving
at constant velocity)

For example,

For example,

5N
2 cm s1
PHYSICS

Ffriction

200 N

Ffriction

F4/2/53c
Linda pushes a book
on a table with a force of 5 N.
The book moves with a uniform velocity of 2 cm s1.
Find the frictional force acting on the book.

Zamhari pushes a heavy


cupboard with a force of
F4/2/53b
200N, but the cupboard does not move.
Find the frictional force acting on the cupboard.

Solution
Using Fnet = ma:
because the book
moves with a
But Fnet = 0 since a = 0
uniform velocity

5 Ffriction = 0
Ffriction = 5 N
(The frictional force here is known as dynamic
friction)

Solution
Using Fnet = ma:
because the
cupboard does
But Fnet = 0 since a = 0
not move
200 Ffriction = 0
Ffriction = 200 N
(The frictional force here is known as static friction)

Note: In both situations, the frictional forces are equal to the forces applied.
Effect of Balanced Forces and Unbalanced Forces on an Object

SPM
08/P2(A)

Balanced forces (Fnet = 0, a = 0)


lift from wings, L

drag or air resistance, G

engine thrust, T
weight, W

T = G
W = L

87

1 Balanced forces on an aircraft allow it to


move at constant velocity at a constant
altitude.
2 The engine thrust is balanced by the drag due to
air resistance while the weight of the aircraft is
balanced by a lift from the wings. The lift from
the wings is discussed in Chapter 3.

Forces and Motion

CHAPTER

Object at rest
(v = 0 m s1)


F1 = F2 F4/2/53a
FromFnet = ma:

0 = ma
a = 0 (since mass, m cannot be zero)

F
O
R
M
4

Unbalanced forces (Fnet 0, a 0)


lift from wings, L
engine thrust, T
drag or air
resistance, G

1 When the forces acting on an object do not cancel out each other, a net force known as unbalanced force is
acting on the object.
2 Unbalanced forces produce an acceleration to the mass on which the forces are acting.
3 However, the object will accelerate in the direction of the net force.
4 When an airplane is moving at a constant velocity, if the pilot increases the engine thrust, the forces acting
horizontally are no longer balanced. There is a net force forwards and the plane will accelerate in the
forward direction.

24
5000 N
upward
F
force from
O
engine

R
M

mass
300 kg

Figure 2.55 shows


a small rocket of
mass 300kg at the
point of take-off.
Find its initial
acceleration.

Solution
Since the upward force is greater than the downward
force by 2000 N, the rocket accelerates upwards.
Neglecting the mass of the exhaust gases, the upward
launching acceleration is given by:

Fnet = 5000 N 3000 N
= 2000 N

Fnet = ma
F
2000
a = mnet =


300

2000 N

CHAPTER

CHAPTER

F
O
R
M

weight, W

resultant
upward
force

weight
3000 N

Figure 2.55

= 6.7 m s2

F4/2/55b

25
John pushes a 12 kg carton with a force of 50 N.

When a floor is smooth, frictional force is nil.

2 m s2

velocity increasing
(a  0)

IL
AG

FR

F1 = 50 N

Ffriction = 0

Fnet = 50 Ffriction

smooth surface

When an object on a rough floor moves at uniform


velocity with a horizontal force acting on it, friction
is equal in magnitude to the applied force (but
acts in the opposite direction).

frictional force, Ffriction

Figure 2.56

F4/2/55a

If the carton moves with an acceleration of 2 m s2,


what is the frictional force acting on the carton?
Solution


Forces and Motion

constant velocity
(a = 0)

E
GIL
RA

Fnet = ma
50 Ffriction = 12 2
Ffriction = 50 24 = 26 N

Ffriction
F = Ffriction
F=F

rough surface

friction

88

26
A shopper pushes a trolley with a force of 20N. The
trolley with a mass of 5 kg, moves at a uniform
velocity of 1 m s1. He then increases his force to
accelerate the trolley. What force should he apply in
order to give the trolley an acceleration of 2ms2?

SPM
Clone

05

Two forces F1 and F2 act on a wooden block which is


placed on a table. The friction between the table and
the block is 3 N.
F2

F1

Solution
table
F = ?
a=2m

(a)

For (a):

Which pair of forces F1 and F2 will accelerate the block?


s2

F1 (N) F2 (N)
A 4 7
B 8 5
C 6 4
D 9 5

(b)

F4/2/57

Ffriction = 20 N

For (b):
Fnet = ma
F Ffriction = ma
F 20 = 5 2
F = 30 N

1m

s1

The trolley is moving


with uniform velocity,
i.e. the force applied
equals friction acting
on the trolley.

CHAPTER

20 N

Comments
The block will move with acceleration if the forces
acting on it are unbalanced.
If the difference between the forces F1 and F2 is
greater than 3 N, then there is an acceleration.

27

AnswerD

Mr Brown whose mass is 70 kg, performs as a human


cannonball at a circus. He is propelled from a 1.6m
long cannon. He is in the barrel of the cannon for 1.2s.

SPM
Clone

07

The figure below shows a car of mass 1200 kg


moving at an acceleration of 2 m s2.

acceleration

Figure 2.57

Find the average net force exerted on him.


Solution
First, we need to find the acceleration of Mr Brown
in the barrel. Then, apply F = ma to find the net force
on him.
u = 0 m s1, s = 1.6m, t = 1.2 s
1
Using s = ut + at 2:
2
1

1.6 = 0 + a (1.2)2
2

If the frictional force acting on the car is 750 N, find


its engine thrust.
A 750 N
C 2400 N
B 1350 N
D 3150 N
Solution
Use Fnet= ma since two forces are acting on the car.
Let T be the engine thrust of the car.

T Friction = ma

T 750 = 1200 2

T = 2400 + 750
= 3150 N

a = 1.6 2 = 2.22 m s2
1.44
Fnet = ma = 70 22.2 = 1554 N

AnswerD
89

Forces and Motion

F
O
R
M
4

Newtons Third Law of Motion


Newtons third law of motion states that:
If object A exerts a force, F on object B, then object B will exert an equal but opposite force, F on
object A. In other words:
To every action there is an equal but opposite reaction.
(b) When a man paddles with a backward force, +F
(action), the reaction force, F, pushes the boat
forwards.

Everyday phenomena that are governed by Newtons


third law of motion:
Blog
(a) When a boy presses on the wall with a force, F, the
wall presses on his hands with a normal reaction
force, F.
action,+F
reactions,
F

action, +F

F4/2/60b

(c) The principle used in rockets and jet engines can also
be explained by Newtons third Iaw of motion. The
action that pushes the exhaust gases out through the
nozzle results in a forward force (reaction force) that
propels the rocket or jet engine forwards.

F
Note:The action and reaction forces
of Newtons third
O
F4/2/60a
law act on different objects whereas two
R
balanced forces act on the same
M object.
4

CHAPTER

reactions, F

CHAPTER

F
O
R
M

Physics

2.5
1 What force is needed so that an object with a mass of
3kg has an acceleration of 2 m s2?

6 (a) When a 2 kg block is pushed with a force of


12N to the right, it accelerates with an
acceleration of 3m s2. In which direction does
friction act? Find its magnitude.

2 A wooden box of mass 2 kg is placed on a smooth


plane. If a force of F is applied to the box, it moves at
an acceleration of 3 m s2.
Find the acceleration of another box with mass of
6kg if the same force is exerted on it.

a = 3 m s2

12 N

3 Puan Zaitun pushes a trolley of mass m kg with a force


of 30 N. The trolley moves with a uniform velocity of
1 m s1. When she doubles her force, the trolley
accelerates at 2 m s2. Find the mass of the trolley.

2 kg


F4/2/62

4 Faizal rides a bicycle at a constant speed of 16ms1.


He stops pedalling and the bicycle stops completely
after 6 s. Given the total mass of Faizal and his
bicycle is 72 kg, find the average opposing force on
Faizal and his bicycle.

(b) An additional force of 20 N to the left then acts


on the block. In which direction does friction
now act? Find the new acceleration of the
block.

5 A race car of mass 1200 kg accelerates from rest to a


1
SPM
in 8 s.
Clone velocity of 72 km h

12 N

07

(a) Find the acceleration of the car.


(b) Find the net force acting on the car.

2 kg


F4/2/63

Forces and Motion

90

20 N

2.6

Analysing Impulse and Impulsive Force

Impulse and Impulsive Force


F

When a tennis racket hits


an oncoming tennis ball,
F4/2/64b
the tennis racket delivers a large force that acts on
the ball for a short time t. The ball bounces off in
the opposite direction. Again, there is a change in
momentum.
The change in momentum is due to the force F
acting on the object for a time t.

F = ma
F=m

Ft

( )
vu
t

= mv mu

substitute

a=

vu
t

Impulse = Change in momentum



unit = N s

or

Impulsive force =

SPM
07/P1

mv mu
t

Change in momentum
Time taken

unit = kg m s1

Impulse is defined as the product of a


force, F and the time interval, t during which
the force acts.
Inpulse = Force Time
=Ft
Impulse is a vector quantity and has the same
direction as the force that causes the change in
momentum.

unit = N

Impulsive force is defined as the rate of change


of momentum during a collision or explosion.
Impulsive force is the large force produced in a
collision or explosion that happens in a short
time of impact.

Both formulae can be used to solve numerical problems.


Both are vector quantities.
91

Forces and Motion

CHAPTER

When a player kicks a F4/2/64a


football, his boot is in
contact with the ball for a time t.
During the time t, an average force F acts on the
ball which makes the ball fly off with a
momentum.
Thus, the force F, acting for a period of time t,
produces a change in momentum to the ball,
since the ball with a mass of m acquires velocity v
after the time t.

F
O
R
M
4

Rebound and Impulse


1 An object might rebound from a wall, or stick to it without rebounding after striking it.
2 In which situation will the wall exert a greater impulse? Look at Example 28.

28
Solution
Take the direction to the right as positive.

A tennis ball and a piece of mud with the same mass


(0.060 kg) which are moving at 9 m s1 strike a wall.
The mud sticks to the wall while the ball rebounds at
6 m s1. Find the impulse on each object.

9 m s-1

mud

tennis ball
6 m s-1

For the tennis ball:


Impulse
= Change in momentum
= mv mu
= 0.06 6 0.06 (9)
= 0.36 + 0.54
= 0.90 N s

Figure 2.58

O
R
M

29

3 Thus, a greater impulse is exerted


on an object if it rebounds after a collision.
F
CHAPTER

9 m s-1

CHAPTER

F
O
R
M

For the mud:


Impulse
= Change in momentum
= mv mu
= 0 0.06 (9)
= 0 + 0.54
= 0.54 N s

Figure 2.59(a) shows a 2 kg wooden block initially at


4
rest on a smooth surface. A force of 8 N is applied on
the wooden block.
8N

u=0
2 kg

8N

t=0s

If the force acts for 3 s,


(a) what is the impulse on the block?
(b) what is the velocity of the wooden block after 3 s?
Solution
(a) Impulse = F t = 8 3 = 24 N s
(b) Impulse = Change in momentum

Ft = mv mu

24 = 2v 0
v = 12 m s1

v
2 kg
t=3s

(a)
(b)
Figure 2.59

30
Solution
(a) Take the direction away from the hand as positive.

Impulse = Change in momentum
=
mv mu

= 0.36 20 0.36 (5)

= 0.36 20 + 0.36 5
=
9Ns
mv mu
(b) F =
t
9
300 =
t
9

t =
300

A player spikes an oncoming volleyball moving


towards him at a speed of 5 m s1 to reverse its direction
at a speed of 20 m s1. The mass of the ball is 0.36 kg.
5 m s-1

20 m s-1

Figure 2.60

(a) Find the impulse on F4/2/164


the ball.
(b) If the average force acting on the ball is 300 N,
how long is the time of contact between the hand
and the ball?
Forces and Motion

= 0.03 s
92

SPM
09/P1

SPM
09/P2/(A)

1 From F = mv mu
t

= Change in momentum
Time of impact

If the change in momentum is constant, then:

From F 1
t
2 A student throws a raw egg at a high speed at a wall, and another egg
against a towel held by his friends. In which case will the egg break?

1
t

t small, F large

t large, F small

towel
egg
egg

egg
egg

CHAPTER

Effects of Time on Impulsive Force

F
O
R
M
4

pile of towel

The egg is stopped by the wall in a very short time


interval, resulting in a large force which causes it
to shatter.

The egg thrown against a towel falls on a few


towels stacked below and does not break.
The movement of the egg is stopped in a longer
time interval, resulting in a small force.

pile of towe

In both cases, the eggs are stopped. Thus, the change in momentum is the same.
3 To understand the effect of time in a collision, look at the following example.

Save your knees!


Joginder (mass, m = 50 kg) jumps down from a wall. He lands on a cement ground at a velocity of 6 m s1.
Joginder bends his knees upon landing.
The time taken to stop his motion is 1.0s.

Joginder didnt bend his knees upon landing.


The time taken to stop his motion is 0.05 s.

u = 6 m s1
u = 6 m s-1

v = 0 m s1

v = 0 m s-1

Solution
Solution

Take the downward direction as positive.


m = 50 kg, u = 6 m s1, v = 0 m s1, t = 1.0 s
mv mu
F =
t
The negative sign indicates the
50 0 50 6
force acts as an opposing force
=
that reduces the momentum
1
of the object.
=
300 N

Take the downward direction as positive.


m = 50 kg, u = 6 m s1, v = 0 m s1, t = 0.05 s
mv mu 50 0 50 6
F=
=
t
0.05
= 6000 N
The impulsive force acting on Joginders leg is 6000N!
This force is great enough to break Joginders legs.

The impulsive force that acts on Joginders leg is 300N.

This example clearly shows that an effective way to reduce the impulsive force is to lengthen the collision time.
u = 6 m s1

93

v = 0 m s1

towel

Forces and Motion

Increasing the Impulsive Force by Reducing the Time of Impact


In all the sports shown below, the time of impact with
the ball is very small and the impulsive force
produced is large.

A footballer kicking
a football

A golfer driving a
golfball with a club

A batter hitting a
baseball

A tennis player hitting a


tennis ball

CHAPTER

FF
OO
RR
M
M

It should be noted that


impulsive forces always
exist in pairs. In the figure
on the right, the change in
momentum of the tennis
ball produces a large
impulsive force on the
racket which reacts to give
rise to an equal but
opposite impulsive force to
the ball (this is in
accordance with Newtons
third law of motion).
Both the racket and the
tennis ball are deformed
temporarily due to the
large force being exerted on
each other.

44

F1

F2

F
O
R
massive
M

CHAPTER

A
hammer head
moving at a fast speed is
brought 4to rest upon hitting
the nail. The large change in
momentum within a short
time interval produces a
large impulsive force which
drives the nail into the wood.

pile driver

In
construction,
tion
the pile driver
is raised to a
certain height
before it is
released. The
momentum
acquired by
the massive
pile driver is greatest just before it
hits the pile. The subsequent
change in momentum that
occurs in a short time interval
results in a large impulsive
force which drives the pile into
the ground.
pile

An expert in karate can


split a thick wooden slab
with his bare hand which
is moving at a very fast
speed. The momentary
contact between the
fast-moving hand and
the wooden slab
produces a large
impulsive force which
splits the wooden slab.

Forces and Motion

94

pestle

mortar

Food such as chillies and


onions can be pounded using a
mortar and pestle (both made
from stone). The pestle is
brought downwards at a fast
speed and stopped by the
mortar in a very short time.
This produces a large
impulsive force which crushes
the food.

Reducing the Impulsive Force by Increasing the


Time of Impact

In sports, the effects of


impulsive forces are reduced
to prevent injuries to
athletes. Thick mattresses
with soft surfaces are used
in events such as the high
jump and pole-vaulting so
that the time of impact
on landing is extended,
thus reducing the
resultant impulsive force.

Polystyrene and cardboard egg


containers are stiff but
compressible. They will
absorb and reduce impulsive
force by lengthening the
time of impact.

F
O
R
M

F
O
R
M

The use of padding in certain sports equipment


like baseball gloves, goalkeeping mitts, polevaulting pits, boxing gloves, and gymnastic mats
is to prevent injuries to players by reducing the
impulsive force.

cardboard egg carton

The effects of
time on the
magnitude of
the impulsive
force

(a) Baseball glove

(b) Boxing glove

In baseball, a player must catch


the ball in the direction of the
motion
of the ball. If the ball is
m
mo
caught
by stopping it in its path,
c
cau
the
t impulsive force acting on the
hand
will be considerable.
h
ha
Moving
his hand backwards
M
Mo
when
catching
the
ball
wh
prolongs
the time for the
pr
momentum change to occur so
mo
the impulsive force is reduced.

Playgrounds are covered with a coarse


fabric material which prolongs thee time
of impact when the children fall,
l, thus
reducing the impulsive force.

When a boxer sees that


his opponents fist is
going to hit his head, he
will move his head
backwards or duck.
This will increase the
stopping time, hence
reducing the average
force on his head since
the momentum change
will be longer.

coarse fabric
material

95

Forces and Motion

CHAPTER

polystyrene

31
Students are always tempted to find the acceleration, a
and then use F = ma to find the impulsive force.
mv mu
, you save the step of finding a,
By using F =
t

A tennis player hits an oncoming 0.060 kg tennis ball


with a velocity of 60 m s1. The ball bounces off in the
opposite direction at 90ms1.
90 m s-1

which is not necessary. Try using the latter formula if the


question mentions the impulsive force, and
acceleration is not stated.

125 N
60 m s-1

SPM
Clone

07

The front and rear sections of a car are designed to


crumple easily. What is the reason?
A To increase the impact time
B To increase the impulse
C To decrease the momentum
D To increase the frictional force

Figure 2.61

Find the time of impact between the racket and the


ball if the impulsive force acting on the ball is 125 N.
Solution
Take the direction to the right as positive.
F = 125 N, m = 0.060 kg, v = 90 m s1, u = 60 m s1
F

mv mu
O
F =
t
R
M

0.06 90 0.06 (60)
125 =

t
4

5.4
+
3.6
t =
125
= 7.2 102 s

Comments

Change in momentum
Time

Impulsive force =

CHAPTER

7
CHAPTER

F
O
R
M

When the time of impact is prolonged, the impulsive


force is reduced.
AnswerA

32

SPM
Clone

09

An athlete bends his legs upon landing in a long


jump event as shown in the Figure 2.63.

Tiger Woods hits a golf ball of mass 0.045 kg at a


velocity of 30 m s1.
m = 0.045 kg

30 m s1

Figure 2.62

If the time of impact is 0.005 s, what is the average


impulsive force applied on the ball by the club?
Solution
m = 0.045 kg, u = 0, v = 30 m s1, t = 5 103 s
Impulsive force, F = mv mu
t

(0.045 30) (0.045 0)
=
5 103
=
270 N

Forces and Motion

Figure 2.63

The purpose of bending his legs is to reduce the


A impulse on his feet
B impulsive force on his legs
C velocity before landing
D time of impact between his legs and the sand.

96

Comments
The impulsive force is inversely proportional to the
time of impact. The athlete bends his legs upon

landing to lengthen the time of impact; reducing the


impulsive force.
AnswerB

2.6
6 A baseball of mass 0.14 kg moving at 40 m s1 is
struck by a bat and rebounds at 60 m s1.

1 A force F acts on a 6 kg object at rest on a smooth


surface. If the velocity of the object increases to
2ms1 in 6 s, what is the value of F ?

60 m s-1

40 m s-1

3 The figure shows a helicopter dropping a box of mass


60kg which touches a sandy ground at a velocity of
12ms1. The box takes 2 seconds to stop after it
touches the ground.

(a)

CHAPTER

2 An object of mass 2 kg is acted on by a force which


causes the velocity of the object to increase from
1ms1 to 9 m s1. What is the impulse on the object?

F
O
R
M
4

(b)

Given that the time of contact is 5 102s, find the


force exerted on the ball.
7 In a crash test, a car of mass 1500 kg crashes into a
wall at 15 m s1. It rebounds at 2 m s1.
40 m s-1
15.0 m s1

(a) Before collision

2 m s-1

What is the magnitude of the impulsive force on the


F4/2/77
box?

(b) After collision

4 Beckham kicks a ball with a force of 1500 N. The


time of contact of his boot with the ball is 0.008 s.
What is the impulse delivered to the ball? If the mass
of the ball is 0.5 kg, what is the velocity of the ball?

If the collision time is 0.18s, find the force exerted


on the car.

5 A 0.045 kg golf ball strikes a wall at a speed of


30ms1 and rebounds at a speed of 20 m s1.

8 A pole-vaulter (mass, m = 50 kg) falls onto a foam


mattress which exerts a force of 250 N on him over a2 m s-1
time interval of 2.0 s.

30 m s-1

foam
mattress

20 m s-1

What is the impulse on the ball? If the force on the


ball is 500 N, find the contact time of the ball with
the wall.

u=?
250 N

Find his velocity just before landing on the mattress.

97

Forces and Motion

fo
ru

Forces and Motion

98

Padded dashboard
Increases the time interval of collision,
thereby reducing the impulsive force
produced during impact.

Bumper
Absorbs the impact
in minor accidents,
thus preventing
damage to the car.

Strong steel struts


Prevents the collapse of the front and back doors
of the car into the passenger compartment.
Also gives good protection from a side-on collision.

Tyres with tread


Drains away water in
its grooves when the
car is on a slippery
road. The car has
more contact on
the road.

Anti-lock braking system (ABS)


Prevents the wheels from locking when the
brakes are applied suddenly.
Allows a car to stop quickly in slippery road
conditions and prevents the car from skidding.

22

Headrest
To push the driver's head so that it moves
together with the rest of his body when a
stationary car is knocked from behind and
pushed forward suddenly. This will prevent
a severe whiplash injury to the neck.

Crumple zone
Increases the time interval of impact so that the resultant impulsive
force is reduced. Therefore, only little energy is transferred to the
inside of the car, and the passengers sustain minimal bodily injuries.
The car is less likely to rebound upon impact. This reduces the
momentum change or impulse.

CHAPTER
CHAPTER

44

Automatic airbag
Acts as a cushion
for the head and
body in an
accident and thus
prevents injuries to
the driver and front
passenger.

Safety seat belts


Prevents the passenger from being
thrown forward or out of the car. Slows
down the forward movement of the
passenger when the car stops abruptly.
FF
OO
RR
M
M

Shatter-proof windscreen
Prevents the windscreen
from shattering and reduce
injuries of passengers caused
by the shards of glass.

1 It is important to increase the time interval


of collision to reduce the impulsive force in
an accident. This concept was explained in
Section 2.6.
2 Figure below explains the safety features in
the design of vehicles.

SPM
07/P1

Being Aware of the Need for Safety Features in Vehicles

lmportance of Safety Features in Vehicles

2.7

CHAPTER

F
O
R
M

Understanding Gravity
SPM
05/P1

Acceleration Due to Gravity

SPM
08/P1

A stroboscopic photograph is
a photograph that shows the
images of an object in motion.
The images are taken at
regular time intervals.

SPM
09/P1

1 Figure 2.64 shows a stroboscopic photograph of two steel spheres


(of different sizes) falling under gravity. The two spheres are dropped
simultaneously from the same height.
2 The time intervals between two successive images are the same.
3 Two inferences can be made from the photograph.

Inference 2

Inference 1
The two spheres are falling
with an acceleration.
The distance between two
successive images of each
sphere increases, showing
that the two spheres are
falling with increasing
velocity, i.e., falling with an
acceleration.

s1

The two spheres are falling


with the same acceleration.
The two spheres are at the
same level at all times.
Thus, a heavy object and a
light object fall with the
same gravitational
acceleration.
In other words, gravitational
acceleration is independent
of mass.

s2

s3

s4

Figure 2.64

Free-falling Object

3 Practically, a free fall can only take place in a


vacuum.
4 The demonstration in Table 2.10 shows the
difference between a fall in air and a free fall in
a vacuum of a coin and a feather. Both the coin
and the feather are released simultaneously
from the same height.

1 A free-falling object is an object falling under


the force of gravity only.
2 A free-falling object does not encounter other
forces like air resistance or friction that would
oppose its motion.

Table 2.10

In air

coin

In vacuum
to vacuum
pump
feather

tap


The air resistance to the feather is greater than the
air resistance to the coin. The feather falls with a
smaller acceleration and takes a longer time to reach
the ground.

In a vacuum, both the coin and the feather would


fall with the same acceleration regardless of their
shapes or masses.

99

Forces and Motion

CHAPTER

2.8

F
O
R
M
4

Figure 2.65Gravitational
force
F4/2/80
is always acting
towards the centre
of the Earth

g = 9.832 m s 2
North Pole (90 N)

rp = distance from the North Pole to


the centre of the Earth
re = distance from the Equator to the
centre of the Earth

rp < re

re F

R
M
4

F4/2/81a

g = 9.780 m s 2

Equator (0) O

CHAPTER

1 Figure 2.65 shows that all objects are pulled towards the centre of the
Earth by the force of gravity.
2 The objects will fall with an acceleration of 9.8 m s2 due to the pull of
this gravitational force.
3 Since this acceleration is due to the force of gravity, it is called the
gravitational acceleration.
4 The value of gravitational acceleration is important in physics and is
specially denoted by the symbol, g.
5 However, the value of g varies slightly according to location.
6 The value of g depends on:
(a) Latitude

Generally, the value of g increases with latitude.

CHAPTER

F
O
R
M

Acceleration of Gravity

Figure 2.66(a)

The elliptical shape of the Earth causes the gravitational acceleration to vary with latitude.
The value of g is smaller at the equator than at the poles because the surface of the Earth is further
from the centre of the Earth at the equator than at the poles.

(b) Height above sea level


The greater the height above the sea level is, the smaller is the value of g.
g1 < g2
g2

Mount Everest

New Delhi (same latitude as


Mount Everest but nearer to
the centre of the earth)

Figure 2.66(b)
F4/2/81b

1 Air resistance depends on the shape or crosssectional area of the falling object.

The figure shows that a paper, repeatedly folded and
sealed tightly with a cellophane tape, falls faster than
a loosely crumpled piece of paper (together with
cellophane tape) of the same mass.
2 Air resistance also increases with the velocity of
the falling object. An objectBlog
moving at a higher
velocity encounters greater air resistance than a
similar object moving at a lower velocity.

crumpled
paper
folded
paper

Physics

F4/2/82
Forces and Motion

100

Activ

To determine the value of gravitational acceleration, g

ity 2.5

Apparatus/Materials
Ticker-timer with carbonised tape, power supply, retort stand, slotted weights with holder, G-clamp, cellophane
tape and soft board.
Arrangement of apparatus
G-clamp

ticker-timer

ticker
tape

stool
weight
G-clamp

ticker tape
folded and
stapled
here

CHAPTER

12 V a.c.
power supply

F
O
R
M

soft board

Figure 2.67

2 Calculation of the value of g:


Initial velocity, u
x cm
x
=
=
cm s1
2

0.02
s
0.04

Procedure
1 A ticker-timer is clamped to a retort stand with a
G-clamp and placed on a tabletop as shown in
Figure 2.67.
2 One end of a carbonised ticker tape
(approximately 1.5m in length) is attached to a
weight holder with a total mass of 200 g.
3 The other end of the ticker tape is passed
through the ticker-timer.
4 A soft board is placed on the floor below the
weight to stop its fall.
5 The ticker-timer is switched on and the weight
is released so that it falls squarely onto the soft
board.
6 Six strips are cut off from the middle section of
the ticker tape with each strip containing 2 dotspaces.
7 A tape chart is constructed. From the chart, the
acceleration of gravity is calculated.
Results

Final velocity, v
y cm
y
=
=
cm s1
2

0.02
s
0.04

Discussion
1 If the activity is repeated with weights of mass
250 g, 300 g, etc, the calculated values of g
would be the same. This shows that gravitational
acceleration, g is not affected by the mass of the
object.
2 It is found that the value of the gravitational
acceleration, g obtained from this activity is less
than the standard value of 9.8 m s2. This is
because the weight (object) is not actually free
falling. It is affected by the following:
(a) Air resistance during its fall.
(b) There is friction between the ticker tape and
the ticker-timer.
Conclusion
A free-falling object falls with gravitational
acceleration, g of 9.8m s2.

1 Ticker tape chart:

y cm

x cm
t

Figure 2.68

101

Forces and Motion

Activity 2.5

Time taken, t = (6 1) strips 0.04 = 0.20 s


Gravitational acceleration, g is calculated by
substituting the values of u, v and t into the
vu
formula g =
.
t

Weight and Gravitational Acceleration

SPM
09/P1

1 The weight of an object is defined as the force


of gravity which is exerted on it by Earth.
2 From the formula:

F = m a


Unit is the same


W = m g
as force, N.




Weight of object

= Mass of object Acceleration due to gravity
FF

O
Gravitational Acceleration Oand
Gravitational Field Strength
RR

22

It is true that the acceleration of an object is directly


proportional to the force acting on it. However, at the
same time, acceleration is also inversely proportional
to mass. Therefore, the effect of greater force is
cancelled by the larger mass of the heavy sphere.
So all objects in free fall have the same acceleration
regardless of their masses.

1 A gravitational field is theMM


region in which an object experiences a
force due to gravitational attraction.
44
2 The gravitational field strength
is defined as the ratio of the weight
to the mass of the object or weight per unit mass.
3 Gravitational field strength =

CHAPTER
CHAPTER

F = force and
a = acceleration

F = ma

Since weight, W, is the force of gravity acting


on an object of mass, m that makes it fall with
an acceleration, g, therefore, using the
corresponding terms, we get:
CHAPTER

F
O
R
M

Since acceleration is proportional to the force acting,


a student may think that a sphere with a weight 10
times the weight of a smaller sphere would fall with
an acceleration 10 times that of the lighter one.

Weight W
=
Mass
m

10
20
30
40

Unit = N kg1

50
60
70

Rearranging the formula,


W = m Gravitational field strength
Compare this with the formula W = m g:

spring balance
measures the
gravitational
attraction
(weight) on
the object

mg


W=m Gravitational field strength

W = m g

Earth


Gravitational field strength = g

Figure 2.69

33
g=Gravitational
acceleration
g=Gravitational field
strength

An object of 8 kg weighs 120 N on a planet.


Find the gravitational field strength on the planet.
Solution

W = mg

120 = 8g
120 N

g =
8 kg

g = 10 m s2
= 10 N kg1
W = mg holds true when g
is given in ms2 as well as
N kg1.

= 15 N kg1
Forces and Motion

102

34
Calculate the weight of an astronaut with a mass of 100kg
(a) on Earth,
1
(b) on the Moon where the gravitational acceleration is of that on Earth,
6
(c) in outer space.
[Take g = 10 m s2]
Solution
(a) Surface of the Earth

(b) Surface of the Moon

(c) Outer space


Mass, m
Weight, W


100 kg
W=mg
=100 10
=1000 N

CHAPTER

Situation


100 kg

100 kg

Gravitational acceleration, g
1
= g
6
1
= 10
6
= 1.67 m s2
W=mg
=100 1.67
=167 N

In the outer space, there is


no gravitational pull on the
astronaut.
g = 0
W= mg
= 100 0
= 0 N (no weight)

Mass and Weight


Notice that in Example 34, the weight, W varies but the mass, m of an object remains the same.
The differences between mass, m and weight, W:
Mass, m

Weight, W

The mass of an object is the amount of


matter in the object.

Definition

The weight of an object is the force of


gravity on the object.

The mass of an object is constant


everywhere.

Changing of value

The weight of an object varies with the


magnitude of gravitational field strength, g
of the location.

A scalar quantity

Physical quantity

A vector quantity

A base quantity

Type of quantity

A derived quantity

kilogram (kg)

SI unit

103

newton (N)

Forces and Motion

F
O
R
M
4

SPM
03/P1

Some recommend changing the acceleration, a in the


equations to g, but this is not necessary.
Simply bear in mind that while solving problems involving
free fall, the acceleration, a in the equations has a value of:
a = 10 m s2 (for downward motion)
Since the object is accelerating downwards.
a = 10 m s2 (for upward motion)
Since the object is moving with deceleration.

Remember our equations of motion with constant


acceleration:
v = u + at
1
s = ut + at 2
2
v2 = u2 + 2as

20 000 km

mass
weight
1 kg g 1 N
1

15 000 km

mass
1 kg

10 000 km

5 000 km

F
O
mass
1 Rkg
M

g2

Solution
up = 0
ap = 10 m s2

weight
2N

Coconut accelerating
downwards.

g3

35

t = 1.5 s
v=?

Wei Bo is a basketball player. His vertical leap is


0.75m. What is his take-off speed?

( a) v = u + at
= 0 + 10 1.5
= 15 m s1
1
(b) s = ut + at2
2
1
h = 0 + 10 1.5 1.5 = 11.3 m
2

Solution

37

0.75 m

Forces and Motion

Coconut falls from rest.

weight
10 N

Earth
g1 < g2 < g3

s = 0.75 m
a = 10 m s2
v = 0
v2 = u2 + 2as
0 = u2 + 2 (10) 0.75
u2 = 15
u = 3.87 m s1

SPM
04/P1

A coconut takes 1.5 seconds to fall to the ground.


What is
(a) its speed when it strikes the ground?
(b) the height of the coconut tree?

25 000 km

Since W = mg,
the weight of
an object
depends on
the
gravitational
field strength,
g which
decreases as
the distance of
the object
from the
centre of the
Earth
increases.

CHAPTER

36
CHAPTER

F
O
R
M

SPM
07/P1

After winning a game, a pitcher throws a baseball


vertically up with a velocity of 30 m s1.
(a) What is the time taken for the ball to reach the
maximum height?
(b) What is the speed of the baseball when it returns
to his hands?
(c) How long is the ball in the air before it comes
back to his hands?

Wei Bo is moving upwards.


At the highest point,
Wei Bo is at rest
momentarily.

104

A C

The ball takes 3 s


to reach the
maximum height.


v2 = 900

v = 30 m s1+ve
Since the ball is moving downwards (direction of

motion is opposite
to its initial direction),
v = 30 m s1
ve
The speed of the ball is 30 m s1.
(c) When the ball returns to his hands, the
displacement, s, is zero.
u = 30 m s1, a = 10 m s2, s = 0
1
s = ut + at2

2
1
when the ball
0 = 30t + (10)t2
is thrown.
2

5t2 30t = 0

t(t 6) = 0
t = 0
or t = 6

(b)
When the ball returns to his hands, the
displacement, s, is zero.
Consider direction upwards as positive.
u = 30 m s1, a = l0 m s2, s = 0
v2 = u2 + 2as
= 302 + 2(10)(0)

when the ball returns


to his hands.

The time taken for the ball to return to his


hands is 6 s.

Problems Involving F = ma and W = mg


(a) From Example 37, by neglecting air resistance, a ball
thrown at 30 m s1 takes 3 seconds to move up.
This is true as the deceleration is 10 m s2. The
velocity decreases by 10 m s1 for each second.
So after 3 seconds, the velocity is zero.

30 m s1

1 s

20 m s1

1 s

10 m s1

1s

Lift
reading on the scale shows
the normal reaction force

0 m s1

(b) The velocity when the ball returns to its original


position is of the same magnitude as the initial
velocity of the ball, but in the opposite direction.
(c) The time to descend to the original position is the
same as the time to ascend. Therefore, an
alternative method to find the total time in the air is
shown as follows.
Time to reach the highest point = 3 s
The total time in the air = 2 3 = 6 s

weighing machine

Figure 2.70

F4/2/85

1 When a girl stands on the platform of a weighing


scale, there are two forces acting on her:
(a) the girls weight, W (= mg) acting down
wards, and
(b) the upward normal reaction force, R exerted
on her feet by the platform of the scale.
2 The reading of the scale gives the value of the
normal reaction force, R.
3 Table 2.11 shows a different situation in the lift.
[Please take note that when the lift is
accelerating upwards or downwards, the
reading on the scale gives the apparent weight,
which is equal to the normal reaction force on
the feet of the girl.]

at rest
t=3s

0 m s-1

0 m s-1

t=3s

t=2s

10 m s-1

10 m s-1

t=4s

t=1s

20 m s-1

20 m s-1

t=5s

t=0s

30 m s-1

30 m s-1

t=6s

F4/2/84

W = true weight = mg
R = normal reaction force
exerted on the girl by the
platform of the scale

105

Forces and Motion

CHAPTER

Solution
(a) Consider the motion
from A to B (motion
up):
u = 30 m s1
a = 10 m s2
At the
v = 0
highest
v = u + at
point,
0 = 30 + (10)t v = 0
t = 3 s

F
O
R
M
4

Table 2.11

Situation

Explanation

(a)Lift at rest or moves up or down at a constant


velocity
cable
R

acceleration, a = 0
F net = 0

Applying Fnet = ma,



R mg = ma

R mg = 0
R = mg
mg

Reading on the weighing


scale gives the true weight.

The girl feels normal.


F4/2/86b

weighing machine
mg

F4/2/86a

22

(b)

FF
O of a m s2
O
Lift moves up at an acceleration
RR
M
M
cable

R1

44

acceleration
=a

CHAPTER
CHAPTER

CHAPTER

F
O
R
M

reading shows the


normal reaction
force, R

Velocity is zero or constant


a=0

As force vector

weighing machine

Lift accelerates upwards


Resultant force, Fnet on the girl
is upwards
R1 > mg

R1

Applying Fnet = ma,


R1 mg = ma
R1 = mg + ma
Reading on the weighing scale
is increased by ma.

mg

mg

The girl
feels heavier.
F4/2/87b

F4/2/87a

(c) Lift moves down at an acceleration of a m s2


cable

R2

Lift accelerates downwards


Resultant force, Fnet on the girl
is downwards
mg > R2
Applying Fnet = ma,

mg R2 = ma

R2 = mg ma

acceleration
=a
weighing machine

Reading on the weighing scale


is decreased by ma.

mg

F4/2/88a

Forces and Motion

106

R2

mg

The girl feels lighter.


F4/2/88b

SPM
06/P1

38

Solution

A box of mass 1.6 kg is suspended from a spring


balance hanging from the ceiling of a lift.

The reading on the spring balance


= The tension in the spring, T
10

60
70

(b) When the lift is accelerating upwards:



T1 mg = ma
Apparent weight (the

T1 =1.6 10 + 1.6 2
reading on the scale)
=19.2 N

W = mg

increases when object


accelerates upwards.

Figure 2.71
F4/2/203

What is the reading on the spring balance if


(a) the lift is stationary?
(b) the lift moves upwards at an acceleration of
2ms2?
(c) the lift moves downwards at an acceleration of
3ms2?
[Take g = 10 m s2]

(c) When the lift is accelerating downwards:



mgT2 =ma

T2 =1.6 10 1.6 3 Apparent weight (the
reading on the scale)
=11.2 N

decreases when object


accelerates downwards.

Help! I am floating!
What is the apparent weight of a girl on a weighing scale
in a lift if the cable of the lift suddenly breaks?

Solution
The girl, the weighing scale and the lift will free-fall if the
cable suddenly breaks.

a = g

R = mg ma
= mg mg
= 0

The apparent weight is zero and the girl experiences


weightlessness. She feels as if she is floating.

Pulley System
1 A frictionless pulley serves to change the direction of a force.
2 The tension, T that results from pulling at the ends of the string or rope
has the same magnitude along its entire length.
(A) A force pulling a mass over a pulley
In this situation, the tension T, is equal to the pulling force F, even if
the rope is slanting.
107

Forces and Motion

CHAPTER

50

(a) If the lift is stationary:


T=mg
=1.6 10
=16 N

20
30
40

F
O
R
M
4

 boy is pulling a bucket filled with sand. The mass of the bucket with the sand is
A
3 kg. Find the tension in the rope if the bucket is
i(i) stationary, or
(ii) moving up with a constant velocity of 2 m s1.

Solution
Since the bucket is moving upwards, T > mg.
Applying Fnet = ma,

T mg = ma
T (3 10) = 3 2
F
T =
6 + 30
O
= 36 N
R
Applied
force, F = 36 N
M
2

 he boy increases his force to move the bucket upwards with an acceleration of
T
2 m s2. Find the applied force, F.

CHAPTER

CHAPTER

F
O
R
M

Solution
Whether the bucket is stationary or moving up with a constant velocity,
the acceleration, a = 0.
Applying Fnet = ma,

T mg = ma

T (3 10) = 0

T = 30 N

(B) A pulley with two masses 4


1 The heavier mass will accelerate downwards while the lighter
one will accelerate upwards with the same magnitude.
2 The tension is not equal to the weight of either mass.

39
Two masses of 5 kg and 3 kg are connected to a rope
which passes over a frictionless pulley.

Applying Fnet = ma,



Mg T = Ma

50 T = 5a (i)
For the 3 kg mass, it moves upwards (T> weight mg).
Applying Fnet = ma,

T mg = ma
T 30 = 3a (ii)
(i) + (ii): 50 30 = 8a
8a = 20
a = 2.5 m s2
Acceleration, a = 2.5 m s2

Figure 2.72

Substitute the value of a into equation (ii).



T 30 = 3 2.5

T = 30 + 7.5
= 37.5 N

Find the tension in the rope and the acceleration of


the 3 kg mass when the 5 kg mass is released.
Solution
When the 5 kg mass is released, it moves downwards
at an acceleration of a (weight Mg > T).

Forces and Motion

Tension in the rope, T = 37.5 N

108

9
For Example 39, if you need to find the acceleration
only, you can solve the problem by considering the two
masses together.
The weight of 50 N overcomes the weight of 30 N.
Applying Fnet = ma,
50 30 = (5 + 3)a
Since both the
masses are

20 = 8a
moving together.

a = 2.5 m s2

SPM
Clone

07

A stone falls from a height of 20m. What is the time


taken for the stone to reach the ground?
A 1.4 s
C 2.8 s
B 2.0 s
D 3.6 s

40
A 2 kg weight is connected by a rope to a 3 kg
wooden block. The rope passes over a smooth pulley
as shown in Figure 2.73.

Solution
u = 0 m s1, a = 10 m s2, s = 20 m
1
Applying s = ut + at 2,
2

1
20 = 0 + (10)t 2
2

CHAPTER

5t2 = 20

t 2 = 4

t = 4
= 2 s
AnswerB

10

SPM
Clone

09

The diagram shows a


coconut falling from a
tree.
Which of the following
graphs correctly shows
the relationship between
the gravitational force, F
and the mass, m, of the
coconut?

Figure 2.73

The weight is then released. Find the tension in the


rope if a friction of 5 N acts against the wooden block.
Solution

F
A

For the 2 kg weight, motion is downwards.


Applying Fnet = ma,
20 T =
2a (i)
F4/2/209
For the wooden block, motion is to the right.
Applying Fnet = ma,
T 5 = 3a (ii)
(i) + (ii): 20 5 = 5a
5a = 15

a = 3 m s2
Substitute a into equation (ii).

T 5 = 3 3

T = 9 + 5
= 14 N
Tension in the rope = 14 N


F
B


D F

Comments
Gravitational force (weight), w = mg (i.e. F = mg).
Since g is constant, so F m.
The corresponding graph is a straight line passing
through the origin.
Answer C
109

Forces and Motion

F
O
R
M
4

2.8
If the distance between the water surface and his
hands is 9 m, what is
(a) the time, and
(b) the velocity
when the stone strikes the water surface?

1 An object weighs 60 N on the surface of the Earth


where the gravitational field strength is 10 N kg1.
What is the weight of the object on a planet where
the gravitational field strength is 18 N kg1?
2 Mustafa steps off a 3.0 m high diving board and
drops to the water below. Find his velocity when his
leg touches the water.

5 A ball is thrown vertically upwards with a velocity of


20m s1. At what height will the ball be when its
velocity reduces to half?

3 Alvin tosses a coin up. The coin takes 1.2 s to return


to his hand.

7 Find the tension in the rope and the acceleration of


the 2 kg weight in each of the following figures.
(a)

(b)

F
Find
(a) the initial velocity of the coin, O
R
(b) the maximum height of the coin.
M

4 A man releases a stone into a well.

CHAPTER

6 A student of mass 50 kg stands on a weighing scale


in a lift. If the reading on the scale becomes 550 N,
is the lift accelerating upwards or downwards?
Find the magnitude of the acceleration of the lift.

CHAPTER

F
O
R
M

8 A steel sphere is dropped from a height of 9 m on


the surface of the Moon. Given that the gravitational
SPM
Clone
1
03 acceleration on the Moon equals
of Earths
6

gravitational acceleration, find the time taken by the
sphere to reach the surface of the Moon.

2.9

Analysing Forces in Equilibrium


SPM
07/P1

Forces In Equilibrium
1 Forces in equilibrium or balanced forces have
been discussed in Section 2.5.
2 When forces are in equilibrium, the net
force, Fnet or resultant force, FR is zero. The
object will either be
(a) at rest, or
(b) in motion with constant velocity.
3 Examples of two forces in equilibrium are
(a) a skydiver falling at a constant velocity,
(b) a book resting on a table.

SPM
08/P1

air resistance, G

W=G

constant
velocity

weight, W

(a)A skydiver falling at constant velocity

Forces and Motion

110

normal reaction force, R

Addition of Forces and Resultant Force


R=W

1 In Figure 2.76(a), the two forces of 3 N and


4 N, pushing a wooden block of mass 2 kg on
a smooth surface in the same direction cause
the block to accelerate at 3.5 m s2.

weight, W = mg

(a)

a = 3.5 m s-2

(b) Book on a table


Figure 2.74

a = 3.5 m s-2

normal reaction force, R (exerted by


surface on the object)

tio
fric

rough
surface

smooth surface

(b)Same acceleration (effect) when a force of 7 N


replaces the two forces 3 N and 4 N
Figure 2.76

2 Figure 2.76(b) shows that if the two forces are


replaced with a single force of 7 N, it will still
accelerate at 3.5 m s2.
3 Thus, the effect of pushing the block with 7 N
is the same as pushing it with 3 N and 4 N in
the same direction, i.e., the block accelerates at
3.5m s2.
4 For the example discussed, 7 N is the resultant
force of the combined forces of 3 N and 4 N.
5 The resultant force is defined as a single
force that will produce the same effect as the
two or more combined forces that it replaces.

weight, W = mg

Figure 2.75

5 A tilted surface is called an inclined plane.


6 To understand better how three forces work in
equilibrium, we need to understand
(a) the resultant force of two forces, and
(b) the resolution of a force.

To Find the Resultant Force


SPM
09/P1

(I) Two Parallel Forces

The resultant force is obtained by simple arithmetic.


Forces acting in the same direction

Forces acting in opposite directions

Subtract the magnitude of the smaller force by the


larger one (to find the difference in magnitude
between the two forces).
The resultant force is in the direction of the larger
force.

Add the magnitudes of the two forces.


The resultant force is in the same direction as the
two forces.
Resultant force, F=F1 + F2

Resultant force, F=F2 F1

111

Forces and Motion

CHAPTER

4 In this section, we shall study more about the


equilibrium of forces, including an object at
rest on an inclined plane where three forces
are in equilibrium. The resultant force on the
object is zero.

F
O
R
M
4

(II) Two Non-parallel Forces (forces at an angle to each other acting at a point)

Resultant force, FR
the tail of F1 to the

1 Simple arithmetic cannot be applied to find the resultant force of two


non-parallel forces.
For example, how to determine the resultant force of the two forces F1
and F2 acting at an angle to each other as shown in Figure 2.77?
2 Instead, we can determine the resultant force by drawing scaled
diagrams using the two methods below.
SPM
07/P1

The Triangle Method (Tail-to-Tip Method)


Figure 2.78 shows a scaled diagram of the triangle of forces constructed to determine the resultant force of the
two forces, F1 and F2, acting at an angle to each other (Figure 2.77).
First, choose a suitable scale for the two forces, for example, 1 cm : 20 kN.
Then, follow the steps below to determine the resultant force.
Complete the triangle by
Draw F2 according to the
scale, with its tail starting
from the tip of F1.
3

Draw the force F1 according


to the scale.

connecting the tail of F1 to


the tip of F2.
4

FR

F2

F
O
R
M

F1

F1

CHAPTER

CHAPTER

Method (I)

F
O
R
M

Figure 2.77

F2

A
5
To find the resultant force,
measure the length OB and
the angle AOB.
F1

2
With the help of a protractor,
measure at A.

Figure 2.78
F4/2/93

Method (II)
The Parallelogram Rule (Parallelogram of Forces)
Figure 2.79 shows a scaled diagram of the parallelogram of forces constructed to determine the resultant of two
forces, F1 and F2 acting at an angle to each other (Figure 2.77).
First, choose a suitable scale for the two forces, for example, 1 cm : 50 N.
Then, follow the steps below to determine the resultant force.
With the help of a protractor,
measure , then draw F 2 so
that the tails of both forces F 1
and F 2 start at O.
2
C

Complete the parallelogram


using a pair of compasses.

F1

FR

F2

F2

F1

F2

Draw the diagonal of the


parallelogram.

F1

5
This diagonal represents the
magnitude and direction of the
resultant force, F R.

1
Draw the force, F 1 according
to the chosen scale.

Figure 2.79

Note that the tails of both forces F1 and F2, and the tail of the resultant force, FR are all at the same point, O and
FR is in between F1 and F2.
F4/2/94
[Note: More students prefer Method (II) to Method (I).]

Forces and Motion

112

41
An ocean liner is towed into harbour by two tugs, A and B, exerting forces of 12 kN and 10 kN
respectively and the tow-ropes making an angle of 68. Find the resultant force acting on the liner.

A
12 kN
68
B

10 kN

CHAPTER

Figure 2.80

Solution
Use a scale of 1 cm : 2 kN (if a scale of l cm : 1 kN is used, the diagram would be too large).
4

Complete the parallelogram by


using a pair of compasses.

Arc with a radius of 5 cm


and the centre of circle at C.
Arc with a radius of 6 cm
and the centre of circle
at A.

Draw a line, OC, of


6 cm to represent
the 12 kN force
from point O.

5
6 cm

FR

F2

Measure the angle after


drawing diagonal OB.

68

Measure 68 from OA
using a protractor.

38
O

Draw the diagonal OB.

F1
A

5 cm
Draw a line, OA, of 5 cm to
represent the 10 kN force.

The resultant force, FR, is represented in both magnitude and direction by the diagonal OB.
Length of OB = 9.2 cm
Magnitude of the resultant force, FR = 9.2 2 kN
= 18.4 kN
AOB = 38
The ocean liner is pulled forward by a resultant force of 18.4 kN at an angle of 38 to the 10 kN force.
Note: There is no need to draw the scaled diagram following the original directions of F1 and F2 as long as the
angle between them is correctly measured.
[Note:If you prefer to solve the problem by calculation, please refer to Example 48 in page 117.]

113

Forces and Motion

F
O
R
M
4

(III) Two Perpendicular Forces

Figure 2.81 shows the two forces (F1 and F2)


together with the resultant force, FR using the
parallelogram rule.
The resultant force is obtained using Pythagoras
theorem.

This is a special situation for two non-parallel forces


acting on an object at a right angle to each other.

FR

F2

Resultant force, FR = F12 + F22


F
tan = 2
F1

F2

Note: You can also solve the problem by drawing a scale


diagram as discussed in section (II).

F1

42
Solution
A sketch of the forces is drawn.

Heng Gee

F
O
R
M

F2 = 70 N

Samy and Heng Gee pull a crate with forces of 70N


and 90 N respectively.

90 N

Resultant force, FR = 702 + 902


= 114 N
70
tan =
= 0.7778
The incorrect
way to determine the resultant force
90

F1 = 90 N

Samy

using the parallelogram rule:

tan = 37.9
The resultant
which acts in a direction
D force is 114N C
37.9 from the original 90 N force.

Figure 2.82

Find the resultant force on the crate due to these two


forces.

F2

The incorrect way to determine the resultant force


using the parallelogram rule:
D

FR

70 N

CHAPTER

Figure 2.81

CHAPTER

F
O
R
M

FR

BD represents the
resultant force.
B

F1

Remember:
To find the resultant force using the parallelogram
rule, the tails of the three forces FR, F1 and F2 must
originate from the same point.

D
F2

FR

F1

BD represents the
resultant force.

F2

FR

AC represents the
resultant force.

B
A

Forces
and Motion
Remember:

To find the resultant force using the parallelogram


rule, the tails of the three forces FR, F1 and F2 must
originate from the same point.

114

F1

5 The magnitudes of the vertical and horizontal


components can be determined with
knowledge of simple trigonometry. We need to
know the magnitudes only since the direction
is already known.
Refer to Figure 2.83(b).

SPM
03/P2

Resolution of Forces

1 Since two forces can combine to give a single


resultant force, therefore by reversing the
process, a single force can also be resolved or
broken up into two components.
2 This reverse process is known as the
resolution of forces.
3 Usually, a force is resolved into components
which are perpendicular to each other.
4 Figure 2.83(a) shows a single force, F. Applying
the parallelogram rule, the force F is resolved
into a pair of perpendicular components: the
vertical force, Fy and the horizontal force, Fx.

Fy = F sin

(a)

Fx

sin =

Fy
F

F = F sin
y

F = F cos
x

Horizontal component, Fx = F cos , and vertical


component, Fy = F sin , are not fixed formulae.
It depends on the position of .
Generally, you can take the component at the given
angle as F cos and the other component as F sin .
Please study Example 43 carefully.

Fx
F

Fy

cos =

For OBC:

CHAPTER

For OAB:

Fx = F cos

(b)
Figure 2.83

(c)

43
Find the horizontal and vertical components for each
of the following forces.
120 N
80 N

60

To find the vertical component:


Fy = F sin
= 80 sin 60
= 80 0.866
= 69.3 N upwards

(b)

60

120 sin 60

(a)

(b)

120 N

Figure 2.84

60
120 cos 60

Solution
(a)
80 N

80 sin 60

is 60 from vertical line.


To find the horizontal component:
Fx = F sin
= 120 sin 60
= 120 0.866
= 103.9 N to the left

To find the vertical component:


Fy = F cos
= 120 cos 60
= 120 0.5
= 60 N downwards

60
80 cos 60

is 60 from horizontal line.


To find the horizontal component:
Fx = F cos
= 80 cos 60
= 80 0.5
= 40 N to the right

* Please take note of the position of the angle .


115

Forces and Motion

F
O
R
M
4

44
Solution

A boy is pulling a toy car with a force of 6 N as shown


in Figure 2.85.

6 sin 30

6N
30

6 cos 30

45

SPM
03/P1

Santhiran pulls a 5 kg crate on the Ffloor with a force of


O
35 N. The friction against the box is
8N.
R

35 sin 25
35 N

Find the horizontal component of the force. If the toy


car is moving with constant velocity, what is the
friction against the toy car?

M
4

5N

25

CHAPTER

Figure 2.85
CHAPTER

F
O
R
M

The horizontal component, Fx = 6 cos 30


= 6 0.866 = 5.2 N
Since the toy car is in dynamic equilibrium (a = 0), the
horizontal force on the toy car is balanced by the friction.
Friction = 5.2 N

8N

25

8N

35 cos 25

Since the crate moves along the floor, only the


horizontal forces need to be considered.
Applying Fnet = ma,
35 cos 25 8 = 5a

23.7 = 5a

a = 4.7 m s2

8N

Figure 2.86

What is the acceleration ofF4/2/97


the crate?
Solution
First, resolve the pulling force into its horizontal and
vertical components.

46
A gardener pushes a
250 kg concrete roller
with a force of 200 N
directed from the handle.
The handle is at an
angle of 33 to
the horizontal.

(c) If the gardener pulls the handle with the same


force in a reverse direction, what is the total
downward force on the ground then?
(d) In which direction, pushing or pulling, would the
roller be more effective in levelling the grass?

33
200 N

Solution
Resolve the force into horizontal and vertical components.

Figure 2.87
F4/2/98

(a) Find the horizontal and vertical components of the


force.
(b) What is the weight of the concrete roller?
What is the total downward force on the ground?

Forces and Motion

116

Resultant vertical force on the ground


= Weight + Vertical component
= 2500 + 109 = 2609 N
(c) When the gardener pulls the roller:
The vertical component of the force
= 109 N upwards
Resultant vertical force on the ground
= Weight Vertical component
= 2500 109 = 2391 N
(d) When the gardener pushes the roller, the net
downward force is greater. This makes it more
effective in levelling the grass.
2

(a) To find the horizontal component:


Fx = F cos
= 200 cos 33
= 200 0.8387
= 168 N to the right
To find the vertical component:
Fy = F sin
= 200 sin 33
= 200 0.5446
= 109 N downwards
(b) Weight, W= mg = 250 10
= 2500 N

Eric and Darren are pulling a trolley on which their


sister, Tina is sitting. Both Eric and Darren pull the
rope with a force of 50 N, which makes an angle of
30 with the direction of motion.

What is
(a) the total of the forces towards the left?
(b) the acceleration of Tina and the trolley?
Solution
Resolving the tension forces into components.
50 N
30
30

50 sin 30
40 N

50 cos 30
40 N
50 cos 30

50 N

Figure 2.88

The total mass of Tina and the trolley is 30 kg and the


friction between the trolley and the floor is 40 N.

CHAPTER

47

50 sin 30

(a) We need only consider the horizontal components


of the forces.
Total forces to the left
= 2 50 cos 30 40
= 2 50 0.866 40 = 46.6 N
(b) Fnet = ma
46.6 = 30a

a = 1.6 m s2

To Find the Resultant Force of Two Forces at an Angle to Each Other by Calculation

48
Find the resultant force
of the two forces as
shown in Figure 2.89.

8N

First resolve the 8 N force into its vertical and


horizontal components.
Total vertical force = 8 sin 60 = 6.93 N
Total horizontal force to the right = 12 8 cos 60
= 12 4 = 8 N
FR = 82 + 6.932 = 10.6 N

6.93
tan = ____
8

= 41
The resultant force is 10.6 N at an angle of 41 to
12N.

120
12 N

Figure 2.89

Solution

F4/2/239
8 sin 60

8N
60

120
12 N

8 cos 60

12 N

6.93 N

FR
8N

F4/2/240

117

Forces and Motion

F
O
R
M
4

The Effective Components of a Weight on an Inclined Plane


1 Consider a box resting on an inclined plane at an angle to the horizontal.
2 The weight of the box is W = mg. The weight can be resolved into two perpendicular
components:
(a) the component vertical or perpendicular to the plane = mg cos ,
(b) the component down or parallel to the plane = mg sin .

W = mg

M
4

sin

mg co

mOg

CHAPTER

CHAPTER

F
O
R
M

W = mg

Figure 2.90

3 To solve problems involving inclined planes, weight is replaced with its two perpendicular
F4/2/99
components.
Acceleration of an object on a smooth
Object in equilibrium on a rough inclined plane


inclined plane
The figure below shows a box at rest on an
inclined plane.

The figure below shows a box on a smooth


inclined plane.
For an object on a smooth inclined plane, the
vertical Rcomponents of the forces are balanced.
g
vin n
oHowever,
ne the plane is not balanced.
the force down
tm a
pla

normal reaction force, R

in
gs

fr

n,

io
ict

tio

F fric

mg cos

jec ith n
ob wn w ratio
do cele

ac a
sin
mg

Since the box is at rest:


The net force perpendicular to the plane = 0

R mg cos = 0

g
vin
mo h an
tmg
c
t
i cosn
je
ob wn w ratio
do cele

ac a
sin
mg

R
th
oo

ne

pla

sm

mg cos

Applying Fnet = ma,



mg sin = ma

a = g sin
F4/2/100
If = 30,
If = 60,

a = 10 sin 30
a = 10 sin 60
= 5 m s2 = 8.66 m s2
The greater the inclined angle, the greater its
acceleration.

The net force parallel to the plane = 0



Ffriction mg sin = 0
Therefore:

R = mg cos
Ffriction = mg sin

Forces and Motion

th

oo

sm

118

49
A carton of mass 5 kg is at rest on an inclined plane making an angle of 15 with the horizontal.
Find the frictional force and the normal force acting on the carton.
Solution
F nor

g
5k

FR

ion

15

15
in
s
50 15

W = mg

Friction, FR= mg sin


= 5 10 sin 15
= 50 0.2588
= 12.9 N

50

50 cos 15

fric

Fnormal= mg cos
= 5 10 cos 15
= 50 0.9659
= 48.3 N

CHAPTER

15

ma

50
Figure 2.91 shows a horizontal force, F acts on a wooden block of weight 8 N placed on a smooth plane inclined at
30 to the horizontal.
th

o
mo

30

ne

pla

W=8N

Figure 2.91

What must the magnitude of F be to prevent the block from sliding down?
Solution
Resolving the force, F and W(= mg) parallel to the plane.
(The vertical component does not play a role because when the block is in motion, it will only move along the
plane.)
os

F
30
0 30
3
os sin
F c mg
30

Fc

30

in
gs

30

m
W = mg

30
Forces along the plane

F cos = mg sin

F cos 30 = 8 sin 30
8 0.5
F =

0.866
= 4.6 N

119

Forces and Motion

F
O
R
M
4

Three Forces in Equilibrium


Problems involving three forces in equilibrium can be solved either by:
(a) Method A: Resolution of forces
(b) Method B: Drawing a closed triangle of forces

A 6 kg weight is suspended from a hook in the ceiling


as shown in Figure 2.92. A horizontal string pulls the
supporting string so that the latter makes an angle of
50 with the horizontal. Find the tensions in both
strings.

T1
T2

50

6 kg
60 N

Solution

Figure 2.92
F
O
Resolution of forcesR
M

Method B: By drawing a closed triangle of forces


Starting with any one of the three forces, a triangle
of forces is drawn. Lets say you start with OA
which represents the weight, 60 N.
a Draw the line OA with an arrow, to represent
60N.
b Starting from A, draw line AB which represents
T2.
c Then, starting from B, draw the line BO which
represents T1 to complete the triangle. The
directions of the three forces follow an
anticlockwise direction. However, if you draw
in sequence 60N, T1 and T2, youll get a triangle
of forces that show a clockwise direction.

Method A:
The slanting force, T1 is resolved as shown.
4

CHAPTER

CHAPTER

F
O
R
M

51

T1 sin 50

T1
T2

50

T1 cos 50

60 N

T2

60 N

Since the weight is in equilibrium, therefore:


Vertically:
Total force upwards = Total force downwards

T1 sin 50 = 60

60
T1 =
sin
50


60
=

0.7660
= 78.3 N
Horizontally:

60 N

60 N

T1
60 N
T2

50
T2

After drawing the triangle, use simple trigonometry


to solve the problem.
60
60
= sin 50
= tan 50
T1
T
2

60
60
T1 =
T2 =
sin
50
tan



50
60
60
=
= 78.3 N
=
= 50.3 N
0.7660
1.192



Total force to the left = Total force to the right



T1 cos 50 = T2

T2 = 78.3 0.6428
= 50.3 N

Forces and Motion

120

52

25 25
F

F sin 25

F sin 25

56 N

56 N

F cos 25

25
25

We need to consider the vertical components of the


forces only.
Since the pail of water is in equilibrium:
Total force upwards = Total force downwards
2F cos 25 = 56
56
W = mg
F =
= 5.6 10
2
cos
25

W = mg

Figure
2.93
F4/2/102

= 56 N

= 30.9 N
Note:Method B is not suitable for this question as
there is no right angle involved.

Solution
Resolve F into its vertical and horizontal components.

53
A lamp of weight 25 N is supported by two ropes as shown
in Figure 2.94.
Given that the tension in rope A is 20 N, find
(a) the angle ,
(b) the tension in rope B.
Solution
Resolve the forces into their components.
T

30 20 N

T cos

T sin
25 N

25

25

W = mg

20 cos 30

F4/2/102

Figure 2.94

20 sin 30
25 N

(a) For horizontal component:


F4/2/253
T sin = 20 sin 30
T sin = 10 (i)

(b) Substitute = 52.5 into equation (i).



T sin 52.5 = 10
10
T =
sin
52.5

For vertical component:


T cos + 20 cos 30 = 25

T cos = 25 20 cos 30

T cos = 7.68 (ii)
T sin
10
(i) (ii):
T cos = 7.68




= 12.6 N
Tension in rope B = 12.6 N

tan = 1.302
= 52.5
121

Forces and Motion

F cos 25

CHAPTER

Joan and her sister lift a pail of water of total mass


5.6kg. Each of them exerts a force of F at 25 to the
vertical. Find the magnitude of F.

F
O
R
M
4

Each elephant in the diagram below is pulling a rope


attached to a mass of 10 kg. The elephants pull with a
force of 5000 N.

5000 N

= 0.01

= 89.4
The angle between the rope, 2 = 2 89.4 = 178.8

5000 N

It is impossible to make the rope horizontal.


See the explanation below.
Let T = tension in this rope.
2T cos = 100
The rope is
Supposing = 90
horizontal.
100
T =
2 cos

Find the angle between the ropes.


Can the ropes be pulled to become horizontal?
Solution
Resolve the forces into their components:
5000 cos

5000 cos

5000 N

5000 N

F
O
R
M

100 N

F4/2/103

5000 sin

5000 sin

100 N

50
=
cos
90

50
=
0

11

Note:You may try to use = 89.99999 and see what is


the value of tension in the rope, T.

SPM
Clone

04

Comments
First, indicate the directions of tension forces along
the string. Since the poster is in equilibrium, the
three forces should then form a closed triangle of
forces.

The figure below shows a poster hanging on a wall.


nail

tension,T1

Value cannot be
determined.

CHAPTER

CHAPTER

10 kg

F
O
R
M

Vertical component:
2 5000 cos = 100
100
cos = 2 5000

tension,T2

T1

T2
T1

weight, W
T2

Which of the following triangles of forces shows the


forces, T1, T2 and W acting on the poster?
A

B C D
Remember:
F4/2/259
Draw the second force from
the tip of the first force.
Then, draw the third force from the tip of the second
force to complete the triangle.

Forces and Motion

Answer A


122

12

SPM
Clone

06

Solution

A mirror of weight 12 N is hung on the wall using a


string as shown below.

1
T

60

60

30
Draw a vertical
line of 6 cm (to
represent the
weight of 12 N).

Measure 30, same


reasoning as 2 . Then
draw line AC to intersect
with BC.

6 cm
(12 N)

T
2
30

Draw a scale drawing of a triangle of forces to


determine the tension, T in the string.
(Use the scale 1 cm : F4/2/104a
2 N)

Measure 30 using a
protractor (since the string
makes 30 to the vertical).
Draw a line BC.

Measure the length of BC.


BC = 3.4 cm
F4/2/104b
Tension, T = 3.4 2 N = 6.8 N

2.9
4 In a circus, a monkey is pulling his chain with a
forward force of 20 N as
shown in the figure.

1 Two forces of 60 N and 80 N act on an object.


Find
(a) the greatest and the smallest forces that can be
exerted,
(b) the resultant force if the force 80 N is acting to
the right and is at right angles to the 60 N force,
(c) the resultant force if the force 80 N is acting to
the right and is at 120 to the 60 N force.
2 Two astronauts push a
800 kg satellite with
forces of 35 N and 45 N
which make an angle of
60 with each other.
Find the resultant force
and the acceleration
acquired by the satellite.

45 N

What force does the clown need to apply to the


chain to prevent the monkey from moving forward?

35 N

5 A footballer is injured and requires traction on his


foot as shown in the figure below.

3 Aunty Anne is
ironing with a
force of 6 N as
shown in the
figure.

What is the effective force that pushes the iron


forwards?

If the force of the traction is 125 N, what is the mass,


m required to pull the rope?

123

Forces and Motion

CHAPTER

F
O
R
M
4

8 A wooden block is placed on top of a smooth


runway of length 2 m. The angle of inclination with
the floor is 30. Find
(a) the time and
(b) the velocity of the block
when it reaches the bottom of the runway.

6 A boy exerts a force of F to pull a box of mass 2 kg


up an inclined plane which makes an angle of 30
with the floor.

30

7 A 3.5 kg flowerpot is supported by two ropes as


shown in the figure. The rope attached to the wall is
horizontal and the rope attached to the ceiling
makes an angle of 30 with the horizontal.
F

10 A traffic light of mass 15 kg is suspended from


two cables as shown in the figure.
40

50

40

O
R
M

Find the tension in the rope.

CHAPTER

150

Given that the friction acting on the box is 3 N, find


(a) the normal reaction force, R acting on the box,
(b) the component of the weight down the plane,
(c) the value of F if the box is moving up the plane
with an acceleration of I m s2.

CHAPTER

F
O
R
M

9 A bird of mass 0.6 kg stands on the midpoint of a


horizontal rope. The rope sags and an angle of 150
is formed.

tion
Fric N
3
=

50
T2

T1

m = 15 kg

Find the tension in each rope.

Find the tension in each cable.

2.10 Understanding Work, Energy, Power and Efficiency

2.10

Understanding Work, Energy, Power and Efficiency


SPM
03/P1

Work

1 Work is done when a force causes an object to move in the direction of the force.

F
s
F

(a) A manF4/2/105a
pushing a bicycle

(b) A weightlifter lifting a barbell


Figure 2.95

Forces and Motion

124

F4/2/105b

2 The work done, W is defined as the product


of the force, F and the displacement, s in the
direction of the force.
Displacement in the
Work done=Force
direction of the force

4 One joule is the work done when a force of


one newton moves an object over a distance of
one metre, in the direction of the force.
1m
1N

W=F s

F4/2/106a
Figure
2.96

1 J = 1 N 1 m = 1 N m

3 The SI unit of work is the joule, J.


Work is a scalar quantity.

1 Force, F in the same direction as the


displacement, s

2 Force, F at an angle to the displacement, s


F sin
F

F cos

The object does notF4/2/106


move in
c the direction of F.
Instead, the horizontal component of the force,
F cos , moves the object on the surface of the floor.
Work done, W
Horizontal
Displacement in the
= component direction of the force

The object moves over F4/2/106


a distance
b of s, in the
direction of the force.
W=F s

= F cos s
W = Fs cos

54

55

A worker in a hypermarket pushes a trolley full of


goods with a horizontal force of 120 N for a
displacement of 15 m. What is the work done by him?

Man Ling pulls a luggage with a force of 30 N that is


applied along the handle at an angle of 40 to the
horizontal. How much work has she done after
walking a distance of 15 m horizontally?

Solution

Solution
120 N

120 N

30

15 m

The force and the displacement


are in the same
F4/2/107
direction.
Work done, W = F s
= 120 15
= 1800 J

40

30

15 m

Work done, W = Fs cosF4/2/108

= 30 15 cos 40 = 345 J
125

Forces and Motion

CHAPTER

Work = Force Displacement (in the direction of the force)

F
O
R
M
4

SPM
08/P1

No work is done when


1 Force, F does not move
(a) James stands still for
30 minutes holding some
books of 20 N. What is the
work done on the books?

2 Force, F is at a right angle (perpendicular) to



the displacement, s
A waiter walks a distance of 5 m holding a tray of
food that weighs 10 N. How much work has the
waiter done to the tray?

F = 20 N

direction of motion
F = 10 N

(b) Nicholas pushes a concrete wall in his


classroom with a force of 20 N for 30 minutes.
How much work does he do on the wall?

weight = 10 N

wall
5m
20 N

F
O
R
M

Solution
The waiter exerts an upward force of 10 N while he
is holding the tray. When he walks forward a
distance of 5m, the tray is not displaced upwards or
downwards that is, the displacement in the direction
of the force is zero.
Work done, W=F 0=0

CHAPTER

CHAPTER

F
O
R
M

W = 20 N

Solution
4
Work done, W = F s
= 20 0
=0
Both James and Nicholas will get tired but there
is no work done on the books or the wall as the
force does not move.

Alternative method
Use the formula: W = Fs cos
F = 10 N, s = 5 m, = 90
W= Fs cos = 10 5 0 = 0
This means that there is no work done to the tray.
Work Done against the Force of Gravity

56
Auntie Sarimah mops
the floor by using a
force of 9 N at an angle
of 60 to the floor.
How much work has
she done after mopping
a total distance of 4 m?

1 An upward force must be applied to lift an object


of weight, mg newton, to a height of h metres.
F (= mg)

4m
60
9N

weight
= mg

Figure 2.97

Figure 2.98

Solution
The angle between the force and displacement is 60.
Work done, W= Fs cos
= 9 4 cos 60
= 18 J

Forces and Motion

2 The work done is the same as the force


multiplied by the distance, in the direction of
the forcethat is,
where F is equal to and
Work done = F h
opposite in direction to

W =
mg h
the weight, mg.
126

57
Solution
(a) Work done

Displacement in the
=Force 
direction of the force
W= mg h
= mgh

A carton of weight, mg newton, is lifted to the top of a


flight of stairs.
F

Work done mg or mg b because the


force, F (= mg) acts vertically over a height h.
The work done is not dependent on the path
taken i.e. or b, but on the vertical height
reached.

mg
b

Figure 2.99

(a) What is the work done? F4/2/111


(b) If the mass of the carton is 2 kg and the height of
the stairs is 60 cm, calculate the work done by the
worker. [Take g = 10 m s2]

(b) If m = 2 kg, g = 10 m s2, h = 60 cm = 0.6m


The work done, W= mgh
F
=2 10 0.6
= 12 J

mg

Energy

SPM
06/P1

1 Energy is defined as the capacity to do work.


2 The SI unit of energy is the joule, J. It has the same unit as work.
3 Energy is a scalar quantity.

F4/2/111

Elastic potential energy is


the energy stored in a
compressed/stretched spring
or elastic material.

Gravitational potential
energy stored in an object Kinetic energy is the
energy that a body has due
due to its position.
to its motion.

Heat energy is a type of


energy associated with
temperature.

FORMS OF ENERGY

Sound energy is the


energy created when an
object vibrates.

Chemical energy is a stored


energy which is released
during a chemical reaction.
Batteries and food store
chemical energy.

Nuclear energy is the


energy released by a
nuclear reaction.

127

Electrical energy is the


energy of electric charges
or electric currents.

Forces and Motion

Reminder

CHAPTER

F
O
R
M
4

Energy Transfer and Work Done


1 Energy is transferred from one object to another when work is done.
A weightlifter displaces a barbell from the ground to a
height above his head. Work is done to lift the barbell.

The work done transfers the chemical energy in the


F4/2/113
pitchers body to the baseball.
The chemical energy
is converted to kinetic energy due to the motion of
the ball.

The chemical energy (stored in food) in the


weightlifter is transferred to the barbell which
possesses energy due to its position, that is, the
gravitational potential energy.

F
R
M

Potential Energy

2 Work done is the medium forO transferring energy from one object to another.
CHAPTER

Chemical energy Kinetic energy


Chemical energy Potential energy

CHAPTER

F
O
R
M

A pitcher throws a baseball towards a batter.

1 The potential energy of an object is defined as the energy stored in the object because of its
position or its state.
2 There are two types of potential energy.
1 Gravitational potential energy
Example: A ball raised above the floor

2 Elastic potential energy

Example: A wound coil-spring in a toy car

final position
(a) After winding

(a) Before winding


ball
mass = m kg

Gravitational Potential Energy

Elastic potential energy will be discussed in


Section 2.12.

initial position

SPM
04/P1

1 A boy is lifting a box of mass m kg at a


constant velocity to a height, h metres above
the Earths surface.
2 Since the box is moving with constant
velocity, its acceleration is zero.
3 Thus, the net force acting on the box is zero.
This means that the upward force, F is equal to
the weight of the box (= mg).
F = mg (in magnitude)

mg
h

Figure 2.100
F4/2/114

Forces and Motion

128

4 Since the force moves a displacement of h to


lift the box, work is done by the force.
Displacement
Work done, W = Force in the direction

of force

58
A pole-vaulter of mass 55 kg clears the bar at 6.0m.
What is his potential energy at the highest point?
[Take g = 10 m s2]


= F s
= mg h
= mgh

Solution

5 What has happened to the work done?


It has changed into one form of energy which,
in this case, is the gravitational potential
energy, Ep stored in the box because of its
vertical position above the ground.

W Ep

Ep =W
= mgh

Potential energy, Ep= mgh


=55 10 6
= 3300 J

Ep =mgh

6 Figure 2.101 shows a ball of mass m displaced


from position B to C. The h represents the
vertical height.

Kinetic Energy
1 Kinetic energy, Ek, is the energy possessed by
an object due to its motion.
2 Consider a force F acting on a trolley of mass
mkg on a frictionless plane over a distance of s.
The force accelerates the trolley from an initial
velocity, u (= 0 m s1) to a velocity, v m s1.

m kg

C
F

CHAPTER

6.0 m

Figure 2.102

3 What happens to the work done, W = F s?


Since
m kg the trolley possesses kinetic energy, Ek
(because of its motion), which is acquired
because
of the work done, we can reasonably
C
conclude that the kinetic energy of the trolley
is transferred from the work done.

Figure 2.101

When the ball is displaced from position B to


C, work is done.
F
Work done, W = F s cos
s
=Fh

= mgh

is the angle betweenBthe force


vector and the displacement vector.
Vertical height, h = s cos

7 Therefore, an object raised to a height, h


possesses a gravitational potential energy,
Ep = mgh, which is independent of the path
taken by the object.
129

Ek = Work done
=Fs
F = ma
A= mas
(v 2 u2)
=m
2
1 2
u=0
= mv
2

Ek =

v 2 = u2+ 2as
(v 2 u2)
as =
2

1 2
mv
2

Forces and Motion

F
O
R
M
4

59
A pitcher throws a baseball of mass 145 g with a
speed of 20 m s1. What is its kinetic energy?

Solution

1
= mv 2
2

Kinetic
energy
145 g = 0.145 kg

Ek = 1 mv2
2

= 29 J

= 4 Ek

F
O
Work is equivalent to energy as they are
R interchangeable.
(a) Work done = Energy acquired, and
M

(b) Energy = Work done

CHAPTER

E = 1 m(v)2
2
1
2
= 2 m(2 v)

= 4 1 mv2
2

u=0

1
= 0.145 20 20
2
CHAPTER

v' = 2v

Momentum p = mv
p = mv
= 2 mv
= 2p

Kinetic energy, Ek

F
O
R
M

m
v

Principle of Conservation of Energy


3 As the coconut falls to the ground, the
gravitational potential energy, Ep decreases
while the kinetic energy, Ek increases due to its
increasing velocity.
4 However, the sum of kinetic and potential
energy remains constant as the coconut falls.
5 When the coconut reaches the ground, all its
initial potential energy has been changed into
kinetic energy.
6 This is a typical example of the conservation
of energy.
7 The Principle of conservation of energy
states that:

1 Figure 2.103 shows a coconut of mass, m kg


falling from a height of h metres to the
ground.

100% Ep

50% Ep
50% Ek

Energy cannot be created or


destroyed. It can be transformed
from one form to another, but the
total energy in a system is constant.

100% Ek

Figure 2.103

2
Before falling, the coconut possesses
gravitational potential energy, EP = mgh. Since
the coconut is at rest, its kinetic energy, Ek is zero.
Forces and Motion

8 Figure 2.104 shows another example of the


conservation of energy by an athlete during a
pole-vaulting event.
130

bar

Ep(gravitational)
O

M
P
Ep(elastic)

v
Ek(kinetic)
K

Ek of running athlete Elastic, EP of the bending pole Gravitational potential energy, EP Ek of athlete

Figure 2.104 Transformation of energy during a pole-vaulting event

CHAPTER

Ek

Solution

60

1
1
(a) Kinetic energy, Ek = mu2 = 2 102
2
2

= 100 J
(b) The kinetic energy is used to overcome the
frictional force.
Initial kinetic Work done to
=
energy overcome friction

Ek = F s
F = F rictional force

100 = F 5

F = 20 N
(c) Heat energy gained = Kinetic energy lost = 100 J

A durian falls from a height of 6 m. What is the


velocity of the durian just before it strikes the
ground? [Take g = 10 m s2]
Solution
According to the Principle of conservation of energy:
Kinetic energy gained = Potential energy lost
1
mv 2 = mgh
m is the
2

mass of
1
the durian.
v 2 = 10 6
2


v = 120
= 10.95 m s1
The velocity of the durian before it strikes the ground
is 10.95 m s1.

62
Figure 2.106 shows a
gymnast bouncing on a
trampoline. He moves
upwards at a velocity
of 8ms1.
How high will he reach?
[Take g = 10 m s2]

61
A moving 2 kg object has an initial velocity of
10ms1. It comes to a stop on a rough surface after
travelling a distance of 5 m.
10 m s1

stop

5m

h=?
u = 8 m s1

Figure 2.106

2 kg
frictional force

v=0

Solution
Potential energy gained = Kinetic energy lost
1

mgh = mv 2
2
1
10 h = 8 8

2

rough surface

Figure 2.105

Calculate
(a) the kinetic energy of the object,
(b) the frictional force acting on the object,
(c) the heat energy produced.


131

h = 3.2 m
Forces and Motion

F
O
R
M
4

63
At a childrens playground, Calvin of 30 kg mass
climbs up a concrete slide of 2.3 m height. He slides
down the slope that has a length of 5 m. At the end of
the slope, which is 0.3 m above the ground, his
velocity is 1 m s1.

Potential
Kinetic
Work done
+
energy lost = energy gained against friction
Figure 2.107

R
M
4

(a) What is his change in potential energy?


(b) What is his kinetic energy at the end of the slope?
(c) Find the average frictional F force against his
motion along the slope. [Take Og = 10 m s2]


600 = 15 + (F 5)
5F = 600 15

F = 585
5
= 117 N

F = F rictional force

CHAPTER


=
15 J
(c) Let F be the frictional force. According to the
Principle of conservation of energy:

CHAPTER

F
O
R
M

Solution
(a) Change in potential energy
= mgh
= 30 10 (2.3 0.3)
= 600 J
1
(b) Kinetic energy, Ek = mv 2
2
1
= 30 1 1

2

To pull or to lift?
The figure (a) shows a worker pulling a load of mass
120kg along a 3 m inclined wooden plane onto a lorry.
The tension in the rope is 600N.
worker
rope
load

(c) Explain why the work done in (a) is greater than the
work done in (b) even though the gain in gravitational
potential energy of the load is the same.
Solution

lorry

(a)

(a) Pulling:
Work done, Wa= F s
= 600 3
= 1800 J
(b) Lifting vertically:
Work done, Wb= mg h
= 120 10 0.9
= 1080 J
(c) The work done in situation (a) is greater than that in
situation (b) because extra work is required to
overcome the friction between the load and the
inclined plane. The extra work done changes into
heat energy which dissipates to the surroundings.

(b)

Extra info

00 N

3 m 0.9 m

road

0.9 m

Then, why does the worker choose method (a)?


Obviously, this is because he only needs to apply a lesser
force to pull the load (600 N < 1200 N).
It is easier to raise a heavy load by pulling it along an
inclined surface than lifting it vertically. This is the function
of an inclined plane.

(a) What is the work done by the worker to pull the load
onto the lorry?
(b) Instead of pulling, if the worker lifts the load vertically
up a height of 0.9 m onto the lorry as shown in figure
(b), how much work does he need to do?
[Take g = 10 m s1]

Forces and Motion

132

To show the principle of conservation of energy


Final velocity of the trolley and the weight just
before the weight hits the soft board, v
x
=
5

0.02
s

x
=
cm s1
0.1

Apparatus/Materials
Ticker-timer with tape and power supply, thread,
trolley, slotted weights with holder, smooth pulley
on a clamp, metre rule, triple beam balance, trolley
runway, soft board, and cellophane tape.

2 Calculation:
Mass of trolley = M
Mass of weight = m
Height of weight before its release = h
Final velocity of trolley and weight = v

Arrangement of apparatus
ticker-timer
trolley

thread
pulley

ticker
tape

(a) Loss of potential energy of the weight


=
mgh

ity 2.6

CHAPTER

Activ

F
O
R
M

(b) Gain in kinetic energy of the trolley and


1
the weight = (M + m) v2
2

weight
soft board

It is found that:
1
(M + m) v2 = mgh
2

Figure 2.108
F4/2/293

Procedure
Discussion

1 The mass of the trolley, M is determined using


the triple beam balance.
2 A 150 g weight (m) is connected by a thread,
which passes over a smooth pulley, to the front
of the trolley.
3 A friction-compensated runway is prepared and
the apparatus is set up as shown in Figure 2.108.
4 The height of the weight from the soft board, h
is measured with a metre rule.
5 The ticker-timer is switched on and the trolley is
released.
6 From the ticker tape obtained, the velocity of
the trolley and the weight just before the weight
hits the soft board is determined.

1 The loss of potential energy of the weight is not


converted to kinetic energy of the weight only.
Part of it is converted to kinetic energy of the
trolley.
2 As the trolley and the weight are moving with
the same speed, the total kinetic energy gained
1
= (M + m) v 2.
2
Conclusion

Results
1 Analysis of the ticker tape:
after the weight hits
the soft board, the
trolley moves at a
constant velocity

Note
In practice, you may find that
1
(M + m) v2 < mgh.
2

before hitting
the soft board, the
trolley moves with
increasing velocity
direction
of
motion

This is because part of the potential energy lost is


converted to sound energy and does work against
air resistance. The principle of conservation is still
true if these are included.

Figure 2.109

133

Forces and Motion

Activity 2.6

When the weight falls, the loss of its gravitational


potential energy is equal to the gain in kinetic
energy of the trolley and the weight. Energy is
therefore conserved. The principle of conservation
of energy is thus verified.

3 The larger units are the kilowatt (kW) and the


megawatt (MW).
1 kW = 1000 W or 103 W
1 MW = 1 000 000 W = 106 W
4 Power is a scalar quantity as both the work
done and energy are scalar quantities.
5 Another unit of power is the horsepower (hp),
which is commonly used in electrical appliances
such as air conditioners.

1 hp = 746 W

1 hp 3 kW
4

SPM
04/P1

Power

1 Power, P is the rate at which work is done, or


the rate at which energy is transformed.

2 The SI unit of power is the watt (W).


1 W = 1 J = 1 J s1
1 s

Power depends on 2 factors:


Work or energy
Time

P= W = E
t
t

1 If t is constant:
PW
Work done , Power
For example,
F
Two librarians A and B are transferring
books from
O
R
the floor to a tabletop. Librarian A carries
one book
to the table while librarian B carriesMtwo books.

Time taken , Power


For example,
Two cars A and B of the same mass accelerate until
80 km h1. As both cars have the same mass, both
cars do the same amount of work to accelerate to
the same velocity.

CHAPTER

2 If W or E is constant:
1
P t

CHAPTER

F
O
R
M

Power, P = Work done


Time taken
Energy transformed
or
P=
Time taken

t=5s
v = 80 km h1
A

A
B

B
t=0
stationary

Car A takes 5 s to reach the velocity while car B


takes 10 s to do so.
Car A takes a shorter time to complete the work.
Car A is more powerful.

In 10 s, librarian B does more work (lifting 20


books as compared to 10 books by librarian A).
Librarian B generates more power.

Activ

ity 2.7

t = 10 s
v = 80 km h1

To measure the rate at which work is done by a person

Apparatus/Materials

Activity 2.7

Weighing scales, stopwatch and ruler.


Arrangement of apparatus

(a) (b)
Figure 2.110
Forces and Motion

134

(c)

Procedure

Work done by the person in climbing the flight of stairs


= Weight Vertical height of stairs
= mgh
= mgnx
Work done
Power generated by the person =
Time taken
mgnx
=
t

1 The mass of the person, m is determined using a


weighing scale.
2 The time, t taken by the person to climb a flight
of stairs is measured with a stopwatch.
3 The number of steps, n is counted and the height
of each step, x (in metre) is measured using a
ruler.

Mass of person = m kg

Note:
The above method measures only the initial power
of the person. In the process of climbing up a flight
of stairs (say, 200 steps) without stopping, the time
taken to climb every 10 steps increases as the
person becomes tired and hence his/her power
progressively decreases.

Number of steps = n
Height of each step = x m
Time taken = t s
Vertical height of stairs, h = nx m
Weight of person = mg N, where g = 10 N kg1

64
A student who is undergoing National Service
training has a mass of 45 kg and takes 6s to climb a
flight of stairs of 36 steps. If each step is 16cm high,
how much power is generated by the student?
[Take g = 10 m s2]

Work done, W = mgh



= 45 10 5.76

= 2592 J
Power generated by the student, P
Work done
=
Time
taken

2592
=
6

Solution
First, find the vertical height, h of the stairs.
h = nx
= 36 0.16
n = number of stairs
= 5.76 m

= 432 W

Power and Velocity

65

Power, P can also be expressed in terms of force,


F and velocity, v.

A stuntman (mass 60 kg) in a movie climbs up onto


the roof of a building of 56 m high. If the power
generated by the stuntman is 2.1 kW, how long does
it take him to complete this task? [Take g = 10 m s2]

Power, P
= Work done
Time taken
= Force Displacement
Time

Solution
Power generated by the stuntman = Work done
Time taken
2100 = mg h

t
= 60 10 56
t


Change the unit of
kilowatt to watt.
33 600
t=
2.1 kW = 2100 W
2100

= Force Displacement
Time
= Force Velocity
P=Fv

Work done
= Force Displacement

Displacement
Velocity =
Time

t = 16 s
135

Forces and Motion

CHAPTER

Calculation

Conclusion
The above method can be used to measure or
compare the power outputs of two or more persons.
F
O
R
M
4

66
A car is moving at a constant velocity of 30 m s1.

Solution
Since the car is moving at constant velocity,

Engine thrust, F = Total resistance force


= 700 N

friction = 700 N

Figure 2.111

Power required, P= F v


= 700 30
= 21 000 W
= 21 kW

Efficiency of Various Devices

1 Heat engines such as a diesel or a petrol engine, and electric motors are
F
two common devices that convert
energy from one form into another,
usually mechanical energy. O
R
2 The pie chart in Table 2.12 shows
the relationship between the total
M
energy input and total energy output of an efficient device and an
inefficient device respectively.4
CHAPTER

If the total resistance force (road friction and air


resistance) acting on the car is 700 N, what is the
power needed by the car to maintain this velocity?

CHAPTER

F
O
R
M

Constant velocity
a=0

Fnet = ma = 0

engine
thrust

Table 2.12

Efficient device

Inefficient device
wasted energy
output

energy
input

useful energy
output

useful
energy output

output

input

energy
input

wasted
energy output
output

input

F4/2/115a

F4/2/115b

Energy input= Total energy output


= Useful energy output + Wasted energy output

In any device, not all


the energy input is
converted into useful
energy output or
useful work done.

Forces and Motion

A portion of the energy output is


converted into heat energy as a result
of friction or air resistance. This heat is
lost to the surroundings and cannot
be used to produce useful work.

136

3 Efficiency compares the useful energy output to the energy input.


The efficiency of a device:

If efficiency = 100%, then:


Useful energy output = Energy input
Perfect device

Useful power output


100%
Power input
P (useful)
= o
100%
Pi

Efficiency =

However, this perfect situation is not


achievable, that is, efficiency < 100%.

The efficiency of a device can also be calculated in


terms of power.
E (useful)
Efficiency = o
100%
Ei

Eo
The useful energy
output and the
t
energy input terms
100%
=
Ei
are each divided
by time.
t

CHAPTER

Useful energy output


100%
Energy input
Eo (useful)
=
100%
Ei

Efficiency =

4 Table 2.13 compares the efficiency of a petrol engine and efficiency of an electric motor.
Table 2.13

Less efficient device (Petrol engine)

Efficient device (Electric motor)

wasted energy
(thermal energy)
(75 J)
input
energy
100 J
petrol engine

efficiency = 25%

wasted energy
(20 J)
input
energy
100 J

useful energy
output
(25 J)

A large portion of the heat produced at high


temperature in the engine is not converted into
F4/2/116a
kinetic energy of the
car.

useful energy
output
(80 J)
electric motor

efficiency = 80%

The resultant heat which originates from friction or


Wasted energy
the heating of the coils in the motor is only
a small
(Internal energy
of the car and
portion of the energy input.
other energy)

Input energy
100 J

Useful output energy


(Kinetik energy of
the motion of the car

To increase the efficiency of a device, we need to reduce the wasted energy output.
Case

(I)

Energy input

100 J

Useful energy output

60 J

F4/2/116b

Wasted energy output


40 J

Efficiency
60
=
100%
100
= 60%

(II) 80 J

60 J

20 J

60
100%
80

= 75%

We need only 80 J to
produce the same
useful energy output.

The more efficient


device is able to
reduce the wasted
energy output.

Same useful
energy output

137

Efficiency
improved.

Forces and Motion

F
O
R
M
4

67

13

A petrol engine has a work output of 96 kJ per


minute. What is the power input if the efficiency of
the engine is 20%?

The diagram shows a weight which is connected to


a wooden block through a frictionless pulley with
an inelastic string.

96 000 J
Power output =

60 s
= 1600 W
P (useful)
Efficiency = o
100%
Pi


1600
20% =
100%
P

i
160 000
Power input, Pi =

20

68

Floor

Weight

What is the energy change in the system after the


weight is released?
A Potential energy of the wooden block
Kinetic Energy of the weight
B Potential energy of the weight Kinetic
energy of the wooden block
C Potential energy of the weight Kinetic
energy of the wooden block and weight
D Potential energy of the weight and wooden
block Kinetic energy of wooden block

A crane lifts a load of 500 kg to a4height of 120 m in


16 s.

Comment
When the weight moves downwards, its potential
energy is converted to the kinetic energy of both
the weight itself and the wooden block (since both
are moving)

120 m

Answer C
Figure 2.112

If the power input is 45 000 W, what is the efficiency


of the motor used in the crane?
[Take g = 10 N kg1]
Solution
Useful energy output = mgh
= 500 10 120
Power
= 600 000 J
Energy

Energy input = Power Time
=
Time
= 45 000 16
= 720 000 J
E (useful)
Efficiency = o
100%
Ei



= 600 000 100%
720 000
= 83.3%
Forces and Motion

Frictionless
pulley

Wooden block

=8000 W
F
O
R
M

Inelastic string

Frictionless table

CHAPTER

CHAPTER

Solution

F
O
R
M

SPM
Clone

10

138

14

SPM
Clone

11

A ball bearing P is released at the top of a smooth plane as shown in the figure.
Smooth plane

Ball bearing P
0.6 m

CHAPTER

What is its velocity at Q?


A 2.5 m s 1
B 3.5 m s 1
C 4.9 m s 1
D 5.8 m s 1
Comments
The potential energy of the ball bearing at P is converted to the kinetic energy at Q.
Solution
Kinetic energy at Q = Potential energy at P
1 2
mv = mgh where m is the mass of the ball bearing
2
v = 2gh
= 2 10 0.6
= 3. 5 m s1
Answer B

2.10
1 The figure shows a weightlifter lifting a barbell of 80 kg.
SPM

What is the work done on the barbell?
Clone
06

1.8 m
1.7

2 A waiter carries a tray full of dishes weighing 30 N from the kitchen to a customer who is 5 m away.
What is the work done on the tray and dishes?
3 What is the work done by a workman carrying a bag of cement of mass 30 kg up a flight of stairs with
20 steps, if each step is 15 cm in height?

139

Forces and Motion

F
O
R
M
4

4 A nurse pushes a patient in a wheelchair with a force


of 30 N for a distance of 25 m.

30 N

9 Saidin, a fireman, slides down a pole when the fire


alarm sounds. He reaches the ground at 5 m s1.
Given that his mass is 60 kg, find the friction that
acts against his motion as he slides down the pole.

30 N

3.0 m

25 m

5 The figure shows two trucks pulling a ship in a canal


of length 1.6 km. The tension in each cable is
5000N.
10 Simon rides his bicycle down a slope of 5 m vertical
height without pedalling his bicycle. The velocities of
the bicycle before and at the end of the slope are
2ms1 and 8ms1 respectively.
u = 2 m s1

F
O
R
Find the total work done on the M
ship.

v = 8 m s1

CHAPTER

CHAPTER

What is the work done by the nurse?


F
O
R
M

5m

6 A fireman of a mass of 70 kg climbs


a 10-m ladder
4
to rescue a girl.

Given that the mass of Simon and his bicycle is


70kg, find
(a) the total mechanical energy before Simon
moves down the slope,
(b) the work done by Simon against friction along
the slope.

30

11 A sprinter runs at a steady speed of 8 m s1 against a


resistance force of 35 N. What is his power?
12 In a hypermarket, a machine can lift a crate of
500 kg up onto a shelf as shown in the figure. The
time taken for the machine to do this work is
1minute.

If the ladder is inclined at an angle of 30 to the wall


F4/2/305
as shown in the figure, how much work has he done?
7 A 7 kg bowling ball is rolling at 3 m s1. What is its
kinetic energy?
8 The figure shows a girl on a swing. She is released
from a height of 1.5 m.

500 kg
initial
position

(a) What is the minimum power needed?


(b) If the machine F4/2/309
is operating at an efficiency of
80%, what is its power input?

v=?
1.5 m

13 A pump can force 300 kg of water to a height of


12m in 20s. Find its useful power. If the power
input is 2 kW, what is the efficiency of the pump?

0.3 m

Find her maximum velocity.


Forces and Motion

2m

140

2.11 Appreciating the Importance of Maximising the Efficiency of Devices

Appreciating the Importance of Maximising the Efficiency


of Devices

1 It has been explained in Section 2.10 that the


efficiency of diesel and petrol engines is low as
a large portion of energy is lost as heat or
sound energy.
2 The efficiency of an electric motor is high but
the efficiency of an electrical generator at a
power station is low (about 30%).
3 The main sources of energy of an electrical
generator in a power station are fossil fuels
such as coal, petroleum, and natural gas.
4 Similarly, vehicles (for transportation) derive
their energy from petrol and natural gas. Thus,
fossil fuels are of utmost importance to us.
5 It must, however, be noted that fossil fuels are
not renewable and these sources of energy will
be depleted some day.
6 Apart from looking for alternative sources of
energy or increasing the source of renewable
energy, efforts must be made to maximise the
efficiency of fuel-burning devices.
7 From the formula of efficiency:
Useful energy output
Efficiency =
100%
Energy input

If the useful energy output (or work done) is


constant, then:
1
Efficiency
Energy input

8
With higher efficiency, fuel-consuming
devices will require less fuel to produce the
same amount of work and hence, the
availability of existing sources of fossil fuels
will be prolonged.
9 The same reasoning is applicable to other
devices. Since less energy input is required to
produce the same amount of work or useful
energy output, the cost of operating the device
is reduced (we need to pay for energy
consumption).
10 Besides conserving resources, maximising the
efficiency can also reduce the heat output or
sound production which might be detrimental
to the environment.

Ways of Increasing the Efficiency of


Devices
Electrical Devices

life span is much longer since it does not


have a filament which can burn out when it
overheats.
3 Use a lamp with a reflector so that the
illumination can be directed to specific areas.
In this way, the required illumination can be
obtained with a smaller number of light bulbs.

Light Fittings
1 Replace filament light bulbs with fluorescent
lamps which have a higher efficiency, preferably
compact fluorescent bulbs.

Air Conditioners
1 Choose a brand/model with a high efficiency.
2 Generally, a higher capacity unit has a higher
efficiency but the choice of an air conditioner
is determined by the users requirement and
the size of the room. Central air conditioners
have high efficiencies.
3 Ensure that windows are closed and gaps
under doors are sealed when using an air
conditioner so that the temperature in the
room can be maintained.

Filament light bulb


(Efficiency: 2%)

Compact fluorescent bulbs


(Effeciency: 79%)
Figure 2.113

2 The illumination efficiency of a fluorescent


bulb is four times that of a filament bulb. Its
141

Forces and Motion

Efficiency , Energy input

CHAPTER

2.11

F
O
R
M
4

Operation of Electrical Devices


1 The management of electrical devices includes
ensuring that the devices are in good operating
condition.
2 If electrical devices function efficiently, it
increases their life span.
3 Devices must be inspected regularly and
serviced.
4 For example, the filter in an air conditioner
and the fins of the cooling coil of a refrigerator
must be periodically cleaned to ensure there
is good air flow and to maintain their efficiency.

Washing Machines
1 Use a front-loading machine,
which is
F
more economical in waterO and electricity
R
consumption.
2

2
When less water is being used, the
consumption of electricity to heat up the water
is correspondingly reduced.
3
A front-loading washing machine uses
less detergent compared to a top-loading
machine.

2.12

CHAPTER

CHAPTER

F
O
R
M

Refrigerators
1 Choose the capacity of a refrigerator according
to the size of the family.
2 The refrigerator must be installed away from
sources of heat and direct sunlight. The
distance from the wall must be more than
3cm to ensure there is good air flow.
3 The doors of the refrigerator must always be
shut tight.
4 Use a refrigerator with a manual defrost device
such a refrigerator can save as much as
3050% electrical energy when compared to a
refrigerator with an automatic defrost device.
However, defrosting must be carried out
periodically to maintain the efficiency of the
refrigerator.
5 In terms of energy efficiency, it is more
economical to use a large capacity refrigerator
than a smaller capacity unit.

SPM
08/P2(B)

4
Understanding
Elasticity

2.12 Understanding Elasticity

Meaning of Elasticity

1 Figure 2.114 shows that the shape and the size of an object change
when an external force is applied on it.
when when

force is applied
force is removed

when when

force is applied
force is removed

Figure 2.114

2 When the external force is removed, the object returns to its original
shape and size.
3 The property of an object that enables it to return to its original
shape and dimensions (sizes) when an applied external force is
removed is called elasticity.
4 Rubber and plastic are examples of elastic materials.
5 Soap or plasticine is deformed when an external force is applied on
them. The deformation is permanent even after the external force is
removed that is, these materials are non-elastic.
Forces and Motion

142

force of
repulsion
(positive)

Forces between Atoms


1 The property of elasticity is caused by the
existence of forces of repulsion and attraction
between molecules in solid materials.
2 For simplicity, we will discuss elasticity in
relation to metals only.
3 Forces of repulsion and attraction between
atoms are always present in metals in a solid
state.
4 In the absence of an applied external force
on a metal rod, atoms vibrate about their
equilibrium positions. The atoms are
separatedby a distance called the equilibrium
distance, xo. At this equilibrium distance, the
force of attraction is balanced by the force of
repulsion the resultant force is therefore
zero.
5 The graph in Figure 2.115 shows the
relationship between the resultant force and
the distance between two atoms, x.
6 When a compressive force is applied to the
two ends of the rod, the length of the rod is
reduced and hence the separation distance
between the two atoms is reducedthat is,
x < xo. The resultant force (or effective force)

force of repulsion
increases when
x decreases
equilibrium distance
distance of
separation between
2 atoms, x

xo

force of
attraction
(negative)

force of attraction
increases with
distance, x

force of attraction
decreases with distance, x

Equilibrium
F

Metal rod is compressed, length


of rod decreases.

rod under
MetalMetal
rod
in
equilibrium
no
external
force
condition.

F4/2/120f

x > xo

Arrangement of atoms

x < xo
F

Metal rod under compression:


1 Force of repulsion takes
F4/2/120e
effect.
2 When the compressive force
is removed, the force of
repulsion between the atoms
pushes the atoms back to
their equilibrium positions.

Metal rod is stretched,


length of rod increases.

F4/2/120b

F4/2/120d

xo
F

Equilibrium
condition:
F4/2/120c

1 
Force of repulsion
= Force of attraction
2 Resultant force = 0

143

Metal rod under tension:


1 Force of attraction takes
F4/2/120g
effect.
2 When the stretching force is
removed, the force of attraction
between the atoms pulls the
atoms back to their equilibrium
positions.

Forces and Motion

CHAPTER

acting between the two atoms is a force of


repulsion. When the external force is
removed, the repulsive force pushes the atoms
back to their original position and the metal
rod returns to its original length.
7 When a stretching force is applied to the two
ends of the metal rod, its length increases. As
a result, the separation distance between the
two atoms is increasedthat is, x > xo. The
resultant force acting between two atoms is a
force of attraction. When the external force is
removed, the force of attraction pulls the
atoms back to their original position and the
metal rod returns to its original length.

Figure 2.115
F
O
R
M
4

2.4
Relationship between force and extension of a spring
Situation
Azizi and Kamaruzzaman, who are 1 year old and
3 years old, are sleeping in their sarong cradles.
The springs of both cradles are identical and new.
Observe the babies and the springs in the Figure
2.116. What inference can you make?

Inference
CHAPTER

F
O
R
M

The extension of the spring depends on the weight or


force acting on it.

(a) Azizi (1 year old)

Hypothesis

(b) Kamaruzzaman
(3 years old)

Figure 2.116

The greater the stretching force, the longer the


extension of the spring.

F
O
R
To investigate the relationship between
a stretching force (weight) and the extension of a spring.
M

Variables

CHAPTER

Aim

(a) Manipulated : weight of load, F


(b) Responding : spring extension, x
(c) Fixed
: force constant of the spring, k
Apparatus/Materials
Spring, pin, slotted weight, weight holder, retort stand with clamp, metre rule, and plasticine.
Arrangement of apparatus
metre rule

zero mark
clamp

spring

retort
stand

pin as pointer
o

o

Experiment 2.4

plasticine

(a)
Figure 2.117

Forces and Motion

144

(b)

extension, x
weight holder
weight

Procedure
1
2
3
4
5

The metre rule is clamped vertically to the retort stand, with its 0 mark at the top.
The spring is suspended from the clamp.
The pin is attached to the bottom end of the spring with plasticine.
The initial position of the pin, o is noted.
A weight holder of mass 50 g is hooked onto the bottom of the spring. A slotted weight of mass 50g
is added to the weight holder. The total weight acting on the spring is noted and the new position of
the pin, is recorded.
6 The extension of the spring is calculated from the formula: x = o
7 Step 5 is repeated with the addition of 50 g each time until a maximum of 300 g.
All the respective readings are noted and recorded.
2

Tabulation of data

CHAPTER

Original position of the spring = o cm


Table 2.14

Load, Stretching Position


Spring
m (kg)
force,
of pin,
extension,

F = mg
(cm) x = ( o )

(N)

(cm)
0.10
0.15
0.20
0.25
0.30

F
O
R
M

1.0
1.5
2.0
2.5
3.0

Graph
The graph of spring extension, x against stretching force, F is drawn.
x (cm)

F (N)

Figure 2.118

Conclusion
The straight line in the x-F graph passes through the origin. This shows that the extension of the spring,
x is directly proportional to the stretching force, F.
The hypothesis is valid.

Experiment 2.4

Hookes Law
1 Hookes law states that the extension of a spring is directly
proportional to the stretching force acting on it provided the
elastic limit of the spring is not exceeded.
2 In Experiment 2.4, the spring returns to its original length when
the load is removed that is, the spring is elastic.
145

Forces and Motion

3 As the addition of weights continues (the stretching force increases), a situation will be reached
where there is permanent extension to the spring when the load is removed.
4 The elastic limit of a spring is the maximum stretching force which can be applied to the
spring before it ceases to be elastic, i.e. permanent extension or permanent stretching is
produced.
SPM
04/P2

Graph of Stretching Force, F against Spring Extension, x

SPM
09/P1

1 Figure 2.119 shows a graph of stretching force, F against the extension of a spring, x.
F

spring not
obeying Hookes law

O R

Figure 2.119

2 Section OE of the graph is a straight


line passing through the origin.
F
Thus, the stretching force is Odirectly proportional to the extension of the spring (that is, the
R
extension is directly proportional
to the stretching force) and Hookes law is obeyed.
M
3 Beyond point E (EQP), the graph takes the form of a curve that is, the stretching force does
not vary directly with the extension
and Hookes law is no longer applicable.
4
4 If the load is removed at point Q, the spring will contract but with a permanent extension,
represented by line OR.
CHAPTER

F = kx
spring obeying
Hookes law

CHAPTER

F
O
R
M

Q P
E

Force Constant or Spring Constant


1 Figure 2.120 shows the linear portion of a graph of the stretching force against the extension of
a spring.
F

Figure 2.120

2 The graph is a straight line passing through the origin. Hence:


F x
F=kx
where k is a constant known as the force constant or spring constant.
3 From the formula : F = kx
F (N)
k =
x
(m)

1

The unit of k is N m .

Forces and Motion

146

4 From Hookes law:


F = k x
Equation of straight line: y = m x, where m is the gradient.

k=m

k= b
a

= gradient of the F-x graph
5 The value of the force constant is a measure of the stiffness of the spring.

Force Constant and Stiffness of Spring

(a) A load of 2.5 N extends spring A by 20 cm.


Applying F = kx,
F

k =
x
2.5 N
=
0.2 m

(b) A load of 15 N extends spring B by 20 cm.


Applying F = kx,
F

k =
x
15 N
=
0.2 m

= 12.5 N m1

= 75 N m1

F (N)
75

stiff spring

in
pr

Spring A requires 12.5 N to be


extended by 1 metre whereas
spring B requires 75 N for the same
extension.
Spring B is stiffer than spring A.

ng A

spri

12.5
O

soft spring
x (m)

Force constant, k = Gradient of graph


1 A larger value of k indicates a stiffer spring.
2 A steeper graph indicates a stiffer spring.

Force and compression of spring


Hookes law is also applicable in the case of the
compression of a spring where the compression of the
spring (a decrease in length) is directly proportional to
the compressive force.

F

x = 0 

Physics Blog
147

Forces and Motion

F4/2/121

CHAPTER

To understand the meaning of the force constant, look at the following examples:

F
O
R
M
4

69

Solution
m = 300 g = 0.3 kg
Stretching force= mg
=0.3 10 = 3 N
Extension, x= 26 20
= 6 cm = 0.06 m
From the formula: F = kx

F
k =
x
3
=
0.06

When 200 g is added, load = 300 + 200


= 500 g
= 0.5 kg
Stretching force= mg
=0.5 10
= 5 N
From the formula: F = kx

Extension, x = F

k
5
=
50

= 0.1 m
= 10 cm

Length of the spring= 20 + 10


= 30 cm

O
R
M

= 50 N m1F

CHAPTER

CHAPTER

F
O
R
M

A spring has an original length of 20 cm. When a load


of mass 300 g attached to it, the length of the spring is
extended to 26 cm. Calculate the force constant, in
Nm1, of the spring. What is the length of the spring
when the load is increased by 200 g?
[Take g = 10 N kg1]

4
Application of Hookes Law on Springs

From Hookes Law, for a spring under two separate


forces F1 and F2 (or loads W1 and W2) with
corresponding extensions x1 and x2:

F1 = kx1
The stretching force is

F2 = kx2
proportional to the
F1 x1
extension of the spring.
Dividing,
=
F
x
2
2

Many questions do not need the calculation of the


force constant, k.
The following method explained in these examples
is suggested.

In the situations for loadextension:


W1 x1
=
W2 x2

m1 x1
=
Since W = mg, we obtain:
m2 x2

0
1

2

x1 = 1 0

Therefore:

x2 = 2 0
F1
F2

m1

or

Used when one
extension or both
extensions are given,
or easily determined
from the information
given.
Used more often.

m2

Figure 2.121

F4/2/122

0 = the original length of a spring


1 = length of spring for F1 and m1
2 = length of spring for F2 and m2

Forces and Motion

F1 x1
=
F2 x2
m1 x1
=
m2 x2

148

F1 1 0
=
F2
2 0
m1 1 0
=
m2
2 0

or

Used only when
needed to find the
original length or
reading on the scale
when no load is
attached (please refer
to Example 72).

71

70

A 30 cm spring is compressed to 25 cm when a load


of 300 g is placed on it. What additional load is
required to compress the spring to 22 cm?

The original length of a spring is 10 cm. With a load


of mass 300 g, the length of the spring is extended to
16 cm.
(a) What is the length of the spring with a load of
400g?
(b) What load is required to extend the spring to
19cm?

Solution

Solution

m1 x1
= ,
m2 x2

300 6
=
400 x2

6 400
x2 =
300


x1 = 16 10
= 6 cm


= 480 g
Additional load= 480 300 = 180 g


= 8 cm
Length of spring = 10 + 8
= 18 cm

72
Figure 2.122 shows the two positions of a pin when a
spring is loaded with two different weights.

Alternative methods
(i)

ruler

m 1 0
1 =
m2
0
2
300 16 10
=
400 2 10

6
3
=
4 2 10

300 g


32 30 = 6 4

32 = 54

2 = 18 cm
Length of spring = 18 cm
(ii)
300 g 6 cm
100 g 2 cm
400 g 4 2 cm
= 8 cm
Length of spring = 10 + 8
= 18 cm
(b) Applying

m x
1 = 1 ,
m2 x2

300 6
=
m2
9

9 300
m2 =
6

Compression:
x1 = 30 25 = 5 cm
x2 = 30 22 = 8 cm

CHAPTER

(a) Applying

m1 x1
= ,
m2 x2

300 5
=
m2
8

8 300
m2 =
5

Applying

10 cm

10 cm

15 cm

15 cm

20 cm

20 cm

25 cm

25 cm
500 g

(a)
(b)
Figure 2.122

What is the reading of the pin when the spring is


without a load?

Extension
= (16 10) cm
= 6 cm

Solution
Let 0 = reading of the pin when the spring has no
load.
m1 1 0

Applying
m2 = 2 0 ,


300 20
= 0
500 25 0

x1 = 16 10
= 6 cm
x2 = 19 10
= 9 cm

3 20
= 0
5 25 0


75 30 = 100 50

20 = 25

0 = 12.5 cm


= 450 g
A load of 450g is required.
149

Forces and Motion

F
O
R
M
4

Factors Affecting the Stiffness of a Spring

Activ

To investigate the factors that affect the elasticity of a spring

ity 2.8

Apparatus/Materials

Procedure

F
O
R
M

Results and Inferences

 1 The apparatus is set up as shown in Figure 2.117 (Experiment 2.4).


2 Steps 1 to 6 of Experiment 2.4 are repeated using the pair of springs of different lengths.
3 A force-extension (F-x) graph is plotted for each spring for comparison.
4 Steps 1 to 3 are repeated with springs of different thickness, springs with coils of different diameters and
springs of different type of materials. Please take note that in each case, all other physical factors remain
constant while one particular factor is being investigated.

CHAPTER

CHAPTER

F
O
R
M

A pair of steel springs, P and Q of different lengths, a pair of steel springs, R and S with wires of different
diameters (different thickness), a pair of steel springs, T and U with coils of different diameters, a steel spring, V
and a copper spring, W of the same dimension, pin, slotted weights, weight holder, retort stand with a clamp,
metre rule and plasticine.

4

Length of spring



Steel springs

longer

F (N)

F-x graph


F (N)

spring P

spring Q

Steel springs

Same dimensions

F (N)
spring R spring R


F (N)

spring S spring S

F (N)
spring T

spring U

larger

smaller

Activity 2.8

Type of material

smaller

larger
(thicker)

Coil diameter

shorter

Thickness of
spring wire
Steel springs

Factors
investigated

Table 2.15

steel
spring
F (N)
spring T

spring U

copper
spring
spring V

F (N)

spring W

O x (cm)
O
O
x (cm) O
x (cm)
x (cm) x (cm)
x (cm)

O
O


The shorter spring, P The spring with the
The spring
with the
The steel spring, V is
F4/2/123
Conclusion is stiffer.
wire of a larger
coil of a smaller
stiffer than the copper

diameter (thicker),
diameter, T is stiffer. spring.

R is stiffer.

Forces and Motion

150

spring T

sprin

System of Springs
0

Identical Springs

Note: For each case below, when load = W, spring extension = x (applied to
a spring alone).
Note: o = length of original spring

Figure 2.123

Table 2.16

Series arrangement

Parallel arrangement

0

0

1x
2

0

0

0

11
xx
22

0

CHAPTER

1x
2

The same load is applied to each spring.


Tension in each spring = W
Extension of each spring = x
Total extension = 2x
Length of spring system
= Total original length + Total extension
= 2o + 2x

The load is shared equally among the springs.


W
Tension in each spring =
2
x
Extension of each spring =
2
x
Length of spring system = o +
2

If n springs are used:

If n springs are used:

The total extension = nx

The total extension =

x
n

73
Solution
First, find the extension of a single spring (very
important).

Figure 2.124 shows a


series arrangement of
three identical springs.
The original length of
each spring is 16 cm.
With a load of 200 g,
the length of each
spring becomes 20 cm.
What is the length p
of the system shown
in the figure?

Extension, x1 = 20 16 = 4 cm
m1 x1

Applying
m2 = x2 ,


200 4
=
300 x2

4 300
x2 = = 6 cm
200

300 g

Total extension = 3 6 cm = 18 cm

Figure 2.124

p = Total original length + Total extension


= (3 16 cm) + 18 cm = 66 cm
151

Forces and Motion

F
O
R
M
4

74

y
Q

For spring Q or R :

x 4.5
Extension = 2 = 2

300 g

Extension = 2.25 cm

Figure 2.125

For spring S, T or U :
x 4.5
Extension = 3 = 3

Solution
First, find the extension of a single spring, x.
x
For 2 springs in parallel, extension =
2
F
x
O =
For 3 springs in parallel, extension
3
R

Extension = 1.5 cm

x1 = 15 12
= 3cm

m1 x1
=
m2 x2


200 3
=
300 x2


200x2 = 3 300

x2 = 4.5 cm

CHAPTER

CHAPTER

F
O
R
M

For spring P :

Figure 2.125 shows a system


of 6 identical springs.
The original length of each
spring is 12cm. When a load
of 200 g is added, the length
of each spring extends to
15cm.
What is the length y of the
spring system?

4
extension
= x
3

12 + 1.5

12 + 2.25

extension = x
2
P

12 + 4.5

extension = x
300 g

y= Total original length + Total extension


= 12 3 + (4.5 + 2.25 + 1.5)
= 36 + 8.25 = 44.25 cm

Length of spring system


= Total original length + Total extension

Non-identical Springs Arranged in Series

75
10 cm

12 cm

Springs A and B are two non-identical springs. Spring


A has an original length of 10 cm which stretches to
12cm with a load of 300 g. Spring B has an original
length of 15 cm and extends to 18 cm with a load of
500 g. What is the length y of the spring system with a
load of 800 g?

B
15 cm

18 cm

300 g

y
500 g

800 g


Figure 2.126
Forces and Motion

152

Solution

For spring B:
500
3
=
800 b

5 Work done, W
= Average force Displacement, x
= 0 + F x
2

Elastic Potential Energy


1 Elastic potential energy is the energy
stored in a spring when it is extended or
compressed.
2 The elastic potential energy is a result of the
work done to extend or compress the spring.
3 Figure 2.127(a) shows a spring with an
original length of o being extended and
compressed with a force that is increased from
zero to F. The extension and compression
have the same value x. Figure 2.127(b) shows
the graph of force against the extension/
compression of the spring.


= 1 Fx
2
=Area of the shaded triangle under the graph
of force against extension
Substitute F = kx into the formula.
Work done, W = 1 kx x = 1 kx2
2
2

Elastic potential energy, Ep is given as:
EP=1 kx2
2

o

76

x
F

spring extended

because
EP = W

A force is increased from 0 to 30 N and applied to a


spring, compressing it by 12cm as shown in Figure
2.128.
force, F (N)
F

30

(a)
F4/2/329(a)

force

compression (cm)

12

Figure 2.128

Find
(a) the force constant, k of the spring,
(b) the elastic potential energy stored.

Area = 1 Fx
2
= Work done
= Elastic Potential Energy

Solution
(a) k = gradient of F-s graph
30 N
1
=
0.12 m = 250 N m

(b) Ep = Area under the F-s graph
1
= 2 0.12 30 = 1.8 J

extension/ compression

(b)
Figure 2.127

4 The work done on the spring is not equal


to F x, because the force is not constant but is
increasing from zero to F.
153

Change the unit


of compression
to metres.
12 cm = 0.12 m

Forces and Motion

Extension, b= 800 3 = 4.8 cm


500
y = Total original length + Total extension
= (10 + 15) + (5.3 + 4.8)
=
35.1 cm

x1 = 12 10
= 2cm

Extension, a= 800 2 = 5.3 cm


300

spring
compressed

x1 = 18 15
= 3cm

CHAPTER

m1 x1

Applying
m = x ,
2 2
For spring A:
300
2
=
800 a

F
O
R
M
4

The graph in Figure 2.130 shows the relationship


between the applied force F, which increases nonuniformly with the extension, x of the elastic band of
a catapult. The mass of the stone is 25g.
(a) What is the work done to stretch the elastic band
of the catapult?
(b) What is the elastic potential energy stored in the
elastic band?
(c) If the stone is released, what is its velocity as it
leaves the catapult?

77
A spring with a 300 g sphere attached to it is
extended by 5 cm. The force constant of the spring is
200 N m1.

5 cm

300 g

smooth surface

Figure 2.129

What is the elastic potential energy stored in the


spring? What is the maximum velocity of the sphere
when the stretching force is released?
Solution
Extension, x = 5 cm = 0.05 m

1
Elastic potential energy, Ep = kx2
2 F
1 O
= R200 0.052
2 M

CHAPTER

CHAPTER

F
O
R
M

=
0.25 J
Maximum kinetic energy gained 4
= Elastic potential energy lost
1 2
Velocity, v is

mv = 0.25
maximum when
2
the spring is not
1
under tension or
0.3 v2 = 0.25
compression, i.e.,
2
when the original
length is restored.


2 0.25
v2 =
0.3

v = 1.666
= 1.29 m s1

v2 = 0.28 2
0.025

= 22.4

v = 4.7 m s1

79
A spring is compressed with a force of 300 N.
If the potential energy stored is 6 J,
(a) what is the compression of the spring?
(b) determine the force constant of the spring.

78
elastic band
stone

Solution

1
(a) Elastic potential energy, Ep = Fx
2
1

6 = 300 x
2

F
F (N)
3.0
2.5
2.0
1.5
1.0
0.5
2

Solution
(a) The number of squares covering the area under
the Fx graph are estimated and multiplied by the
value of each square. A partial square, whose
area is greater than half the area of a full square,
is counted as a complete square. Conversely, a
square that is less than half filled is not counted.
Number of squares = 28

Value of 1 square = 0.5 N 0.02 m = 0.01 J
Work done, W = 0.01 J 28 = 0.28 J
(b) Ep = Work done = 0.28 J
(c) Ek = Ep = 0.28 J
1
mv2 = 0.28
2

1
0.025 v2 = 0.28
2

10

12

14

16

Compression, x = 0.04 m = 4 cm
(b) Use the formula F = kx,
F

k =
x
300
=
= 7500 Nm1
0.04

x ( cm )

Figure 2.130
Forces and Motion

154

A weighing apparatus
uses the elastic property
of a spring. When a load
is being weighed, the
spring is either extended,
or compressed, in direct
proportion to the load
whose value is registered
on the linear scale of the
apparatus. If there is no
load, the spring in the
weighing apparatus
returns to its original
length which
corresponds to the zero
reading on the scale.

Spring shock
absorbers
Spring shock absorbers
are mounted on the
wheels of cars and
motorcycles to absorb
impact and reduce
vibrations resulting
from uneven road
surfaces.

Mattress and
cushion

The spring attached to


the baby hammock
enables it to bounce
vertically when the
hammock is displaced
downwards slightly.
The bounce helps babies
to fall asleep.

The springs in a
mattress or cushion
undergo many cycles of
compression during its
use yet the mattress or
cushion always returns
to its original shape.
This is due to the
elasticity of the
springs.

Uses of springs
in daily life

Voltmeter,ammeter,
galvanometer,spring
operated clocks and
watches
The potential energy
stored in the spiral
springs is converted into
kinetic energy which
turns the hands of the
clock and the pointer of
the meter until the
springs return to their
original state.

Baby hammock

CHAPTER

Weighing
apparatus

Uses of the Property of Elasticity

Tyre pressure
gauge

Door closer
This device has a spring
system and is mounted
on the door to close it
automatically. When
the door is open, the
elastic potential energy
stored in the compressed
spring closes the door.

When the nozzle of the


gauge is fixed on to the
valve of the tyre, the air
pressure in the tyre pushes
a piston which compresses
a spring inside the gauge.
This allows us to read the
amount of pressure in the
tyre.

Toys
The toys below move
using the power of
springs. The work done
in compressing the
spring is converted into
elastic potential energy,
which is then converted
into the kinetic energy of
the toys, thus making the
toys move.

door closer

155

Forces and Motion

F
O
R
M
4

The elasticity of a springboard


enables a diver to jump high in
the air before diving into the
pool.

As the bow bends, the elastic


twine of the bow is stretched to
store the elastic potential energy
which propels the arrow when it
is released.

15

F
O
R
M
4

Other elastic material

CHAPTER

CHAPTER

F
O
R
M

The material of trampoline has


an elastic property to help a
gymnast bounce upwards.

The pole in a pole-vaulting event


uses the property of elasticity to
help the pole-vaulter to leap over
the bar.

The elasticity of the strings of a


tennis or a badminton racket
allows a ball or shuttlecock to
bounce off them.

The double clips and clothes


pegs make use of the spring
between the clips to hold things
tightly.

SPM
Clone

11

Figure (a) and (b) shows the set up to study the elasticity of spring X and Y and the corresponding force-length graphs.
Force
X

Length of
spring

(a) (b)

Comment
As shown in the diagram,
for the same force
applied, spring X
extends less than spring
Y. Therefore, spring X
is stiffer than spring Y

Which comparism about spring X and Y is correct?


A X is stiffer than Y
B To stretch 1 cm, work done needed on X is less
than on Y
C The original length of X is longer than Y

Answer A
Forces and Motion

156

Force
X

Y
F
Length of
spring

2.12
1 A spring of length 20 cm is extended to 26 cm
when pulled by a force of 10 N. What is the length
of the spring when it is compressed by a force of
20N?

2 The figure below shows a spring which is compressed.

450 g

500 g

(b)

12 cm

What is the length of the spring system in Figure (b)?


All the springs are identical.

10 cm

6 The figures show a spring system comprising


identical springs.

CHAPTER

Calculate the value of m if the original length of the


spring is 15 cm.
3 The figure shows the pointer reading of a spring.
When an additional load of 200 g is placed in the
pan, the pointer reads 25 cm.

20 cm
25 cm

300 g

600 g

300 g

What is the reading of the pointer when the total


load is removed?

4 M and N are two non-identical springs each


measuring 12 cm. When subjected to loading, their
respective lengths are as shown in the figure (a)
below.

7 Figure (a) shows a spring of length 20 cm


compressed to a length of 15 cm by a load of 250g.

15 cm N

Each spring is of length 18 cm and extends to 22


cm when subjected to a load of 200 g. What is the
length of the spring system in the figure?

250 g

18 cm
y

300 g

15 cm

500 g

12 cm

600 g

(a)

(b)

(a)

What is the length y of the spring system if M and N


are arranged in series and the load applied is 600 g?

250 g

5 Figure (a) shows two springs arranged in parallel,


each of length 9 cm, subjected to a load of 600 g.

12 cm

15 cm

12 cm

(b)
600 g

What is the value of m in Figure (b) if three identical


springs are compressed to a length of 12cm?

(a)

157

Forces and Motion

F
O
R
M
4

9 The figure shows a sphere of mass 20 g pushed


against one end of a spring on a smooth surface.
The original length of the spring is 18 cm and its
force constant is 12.5 N m1.

8 The figure is a graph of extension, x against load, m


for a spring.
x (cm)
a
12 cm

12 cm

compression
force

m (g)

600 800

smooth surface

(a) What is the elastic potential energy stored in the


spring?
(b) What is the maximum velocity reached by the
sphere when the compressive force on the
spring is removed?

(a) What is the value of the force constant, k?


(b) What is the value of a?
(c) What is the potential energy stored when the
spring is extended by a cm?

F
O
R
1. Speed is the rate of change of distance.
M
Distance travelled

Speed =

Time taken

the collision is less than the kinetic energy before


the collision.
(b) In an elastic collision, momentum, total energy
and kinetic energy are conserved.
10. Acceleration is proportional to the applied force but
inversely proportional to the mass.
11. Impulse is the product of a force and the time
it acts.
Impulse = Ft
12. Impulse is equal to the change in momentum.
Ft = mv mu
13. The impulsive force is reduced by prolonging the
time of impact.
14. When forces are in equilibrium, the object will be at
rest or move with constant velocity.
15. A force can be resolved into vertical and horizontal
components.
16. Work is the product of a force and the distance
moved in the direction of the force.
17. Energy is the capacity to do work.
(a) Potential energy, Ep = mgh
1
(b) Kinetic energy, Ek = mv 2
2

CHAPTER

CHAPTER

F
O
R
M

2. Velocity is the rate of change of displacement.


Displacement
Velocity =
Time taken
3. Acceleration is the rate of change of velocity.
vu
a=
t
4. Equations of linear motion with constant acceleration:

v = u + at
1
s = (u + v)t
2

v 2 = u2 + 2as
1
s = ut + at 2
2


5. On a displacementtime graph, the gradient of the
graph represents the velocity of the object.
6. On a velocitytime graph,
(a) the gradient of the graph is equal to the
acceleration of the object.
(b) the area under the graph is numerically equal to
the distance travelled.
7. Inertia is the tendency for an object to maintain its
state of rest or uniform motion in a straight line.
8. Momentum of a moving object is the product of its
mass and its velocity.
9. The principle of conservation of momentum states
that the total momentum of a system is conserved,
if no external force acts on the system.
(a) For inelastic collision, momentum and total
energy are conserved but the kinetic energy after
Forces and Motion

20 g

18. Power is the rate at which work is done, or the rate at


which energy is transformed.
Work done
Power =
Time taken
19. Efficiency =

Useful power output


100%
Power input

20. Hookes Law, F = kx


1
Elastic potential energy, Ep = kx 2
2

158

SPM Exam Practice 2

2
Multiple-choice Questions

1 Puan Halimah pushes her trolley from P to U via Q, R, S, T and U as shown in


Diagram 1.

4 Which features of a graph is used


to determine acceleration?
A The slope of a displacement
time graph
B The slope of a velocitytime
graph
C The area under a
displacementtime graph
D The area under a velocitytime
graph

2.3

1.1

Significant Figure

Diagram 1

Find her displacement from P.


A 10 m
B 17 m
C 23 m
D 77 m

2.2

Motion Graphs

3 In Diagram 3, graphs X and Y are


the velocity-time graphs for car V
and car W respectively.

2 Cars P and Q start from rest


simultaneously and accelerates at
acceleration a and 3 m s2
respectively.

Diagram 2

After 2 seconds, P is ahead of Q


by 6 m. Find a.
A 4.5 m s2
B 5.0 m s2
C 5.5 m s2
D 6.0 m s2

Which of the following areas


represents the distance between
the two cars at time t?
A OPQ
B OQR
C OPQR
D OPR

159

F
O
R
M
4

Inertia

5 A body stays at rest, or continues


SPM
to move with a constant velocity,
Clone
09 unless an external force acts on it.
Which of the Newtons laws is
related to the above statement?
A Newtons first law of motion
B Newtons second law of
motion
C Newtons third law of motion
6 A loaded ocean tanker is more
difficult to stop than an empty one
because it
A possesses more kinetic energy
B sinks more into the water
C possesses more inertia
D encounters a smaller frictional
force
1.1

2.4

Diagram 3

Linear Motion

CHAPTER

2.1

Momentum and
Conservation of
Momentum

7 A golfer swings a club to hit a golf


ball. The club continues to swing
while the golf ball flies off with a
1.1 high
velocity.
Which of the
Signifi
cant Figure
following quantities is conserved?
A Kinetic energy
B Potential energy
C Momentum
D Impulse
Forces and Motion

8 Diagram 4 shows two trolleys


P (mass = 2 kg) and
Q (mass = 1 kg) placed together.
P and Q move in opposite
directions when the release pin is
struck.

C Speed of P
= 1 the speed of Q
2
D Velocity of P
= 2 times velocity of Q

F
O
R
M
4

2.5

The Effects of a Force

12 Diagram 6 shows Nazli catches an


incoming softball of mass 0.14 kg
by moving his hand backwards.

1.1

Significant Figure

Diagram 6

The velocity of the ball reaching


his glove is 10 m s1. If the time
taken to stop the ball is 1.2 s, what
is the force exerted on his hand?
A 0.9 N
C 2.0 N
B 1.2 N
D 8.6 N

Which of the following statements


is correct?
A Kinetic energy of P
= Kinetic energy of Q
B Momentum of P
= 1 Momentum of Q
2

Which of the following tape charts


can be expected if the student
repeats the experiment using a
falling weight of 200 g?

CHAPTER

Impulse and Impulsive


Force

11 An egg falls on a thick mattress. It


does not break because the mattress
A reduces the time of impact.
B prolongs the time of impact.
C reduces the kinetic energy of
the egg.
D reduces the eggs impulse.

Diagram 4

CHAPTER

F
O
R
M

2.6

9 The graph in Diagram 5 shows the


velocity-time graph of a car.

2.7

Safety Features in Vehicles

13 Which of the following is not a


safety feature of a car?
A Safety belts
B Spacious compartments
C Air bags
D Padded dashboard
Diagram 5

Which part of the graph indicates


that the resultant force on the car
is zero?
A PQ
C RS
B QR
D ST

2.8

Forces and Motion

15 The mass of an object in Kuala


Lumpur is 20 N. Compare its
mass and weight on Mount
Everest.
Mass Weight
A
2 kg
20 N
B
2 kg
20 N
C
2 kg
20 N
D
2 kg
20 N

Gravity

14 The tape chart shown in Diagram


7 is obtained in an experiment
carried out in a laboratory.
falling weight used is
1.1 The
Significant Figure
100 grams.
10 A force of 9 N pushes an object of
mass 3 kg on a floor. If the object
moves at a uniform velocity of
2 m s1, what is the magnitude of
the friction, in N?
A 1
3
B 3
C 6
D 9

Diagram 7

1
the
3
weight of his father. If the total
1.1
weight
theFigure
student and his
Signifiof
cant
father is 600 N, what is the mass,
in kg, of the student?
A 15
B 20
C 30
D 150
16 The weight of a student is

160
1.1

Significant Figure

17 FR1 and FR2 are the resultant forces


of the forces shown in Diagrams
8(a) and (b) respectively.

(a)

(b)

At which point of the path does


the stone possess minimum
kinetic energy?
A P
B Q
C R
D S
21 Diagram 11 shows a pulley
system with P and Q of masses
2 kg and 1 kg respectively.

2.12 Elasticity
24 A spring with an original length of
20 cm is stretched to 26 cm.
The force is F N and the elastic
potential energy stored is Ee. The
spring is then compressed so that
its length is now 17 cm. What is
the force and the elastic potential
energy stored in the spring?
Force Elastic
applied potential

energy stored
1
1
F
A
E
4 e
4

Diagram 8

What can be said about FR1 and FR2?


A FR1 FR2
B FR1 = FR2
C FR1 FR2
18 The following situations show
1.1 Significant Figure
equilibrium of forces except
SPM
A
a
coconut
falling
from
a
tree.
Clone
08 B a ship floating at rest in the sea.
Diagram 11
C a car moving up a hill at
constant speed.
D a lamp hanging at rest in a hall. What is the total kinetic energy of
P and Q when Q rises 2 m?
A 5 J
B 10 J
C 20 J
2.10 Work, Energy, Power and
D 40 J
Efficiency
19 A motor takes 5 s to lift a load of
6 kg to a height of 0.8 m.
SPM
Clone

04

motor

load

0.8 m

Diagram 9

What is the power of the motor?


A 9.6 W
C 240 W
B 96 W
D 375 W
20 A boy throws a stone in the
direction as shown in Diagram 10.

Diagram 10

1
F
2

1
E
4 e

1
F
2

1
E
2 e

Ee

25 Diagram 12 shows three identical


springs, P, Q and R, supporting a
load of 600 g. The original length
of each spring is 10 cm.

22 Pak Hashim with a mass of


50 kg climbs a coconut tree
to a height of 6 m to pick a
coconut in 1 minute. What
is his power?
A 5 W
B 25 W
C 50 W
D 300 W

2.11 The Importance of


1.1

Maximising the Efficiency


of cant
Devices
Signifi
Figure

23 The efficiency of a device can be


increased if
A the useful power output is
increased while the power
input is maintained.
B the useful power output is
maintained while the
power input is increased.
C the useful power output and
power input are increased.

Diagram 12

What is the total length, y, of the


spring system if P, Q and R are
arranged in series and a load of
300 g is attached to spring R?
A 27 cm
B 39 cm
C 48 cm
D 57 cm

161

Forces and Motion

1.1

Significant Figure

Forces in Equilibrium

CHAPTER

2.9

F
O
R
M
4

1.1

Structured Questions
3 (a) Bakri carries a pail of sand up a flight of stairs of
1.2 m as shown in Diagram 3. The weight of the
pail of sand is 100 N.

1 A sprinter runs along a 100-m track as shown in


Diagram 1.
100 m
t=5s
24 m

Diagram 3


t=5s

100 mis the work done on the pail of sand by


What
Bakri? [2 marks]
(b) Two elephants pulled the same pail of sand to the
same height using two ropes and two pulleys as
shown in Diagram 4.

(a)
The sprinter reaches his highest velocity
5seconds after starting from rest. He covers a
displacement of 24 m in the 5 seconds. What is
his average acceleration?
[2 marks]
(b) What is the average net force that is needed to
achieve the acceleration in (a) if the mass of the
sprinter is 60 kg?
[2 marks]
(c) Find the total time it takes the sprinter to
complete 100 m if he maintains
his
highest
F
24 m
O
velocity.
R
[
2 marks]
M

CHAPTER

Diagram 1
CHAPTER

F
O
R
M

2 A boy scout of a mass of 45 kg runs at a speed


of 6ms1 before grabbing a 4rope and swinging
upward.

F4/2/145

Diagram 4

Diagram 2

(i) How much work did the two elephants do


on the pail of sand?
[1 mark]
(ii) If the angle between the two ropes is 178,
determine the tension in each rope.
[2 marks]
(iii) Is it possible to pull the ropes such that they
are horizontal to the floor? Explain your
answer.
[3 marks]

4 Diagram 5 shows two blocks, P and Q, of masses 2m


and 3m respectively tied with a string. The spring
which is attached to block P is compressed to 12 cm.

(a) What is the change in energy?


[1 mark]
(b) How high will the boy swing upward?  [2 marks]
[Take g = 9.8 m s2]
(c) The boy releases his hold on the rope at the
highest point of the swing.

(i) How does the boy fall? Tick your answer.
Vertically
In a parabolic path

[1 mark]

(ii) State his velocity on landing.
[1 mark]

(iii) The boy bends his knees and takes 1.2s to
stop his fall. Find the impulsive force on his
legs.
Forces and Motion

Diagram 5

(a) The force constant, k, for the spring is 5000 N m1.


Explain the meaning of 5000 N m1.[1 mark]
(b) Given the original length of the spring is 20 cm,
find the elastic potential energy stored in the
spring. [2 marks]

162

(c) The string is burnt and P and Q are pushed apart from each other.
(i) Name the principle of physics in this motion.
[1 mark]
(ii) If Q moves at a speed of 6 m s1, determine the speed of P.[2 marks]
(d) Determine the value of m.[2 marks]

6 (a) What is meant by force constant of a spring?



[1 mark]
(b) Diagram 8 shows an increasing force applied to
the right against two springs.

Diagram 6


(a) (i) Define momentum.
[1 mark]

(ii) Relate the outcome for each material to the
change in momentum. Then, deduce a
relevant physical concept.
[5 marks]
(b) Explain the changes in energy of the unbroken
egg from the moment it is released until it
touches the towel.
[4 marks]

Diagram 8

Draw a force-distance graph to show how the


force varies with the distance moved. The force
starts from zero and increases as it moves to the
right.
[2 marks]

Explain the shape of your graph.


[2 marks]
(c) Mr Arsenal applies a stretching force of 380 N to
pull a chest expander (consists of 2 springs) from
30 cm to 40 cm.

deck

truck wheel

Diagram 7

(c) Diagram 7 shows a boy in action on his


stakeboard.
You are asked to investigate and suggest how the
skater can improve his performance by
considering the following aspects:
Skaters attire
Structure of the skateboard (deck and truck)
Wheels of the skateboard
Bearings fitted to the wheel
[10 marks]

Diagram 9

(i) Find the force constant of each spring.


(ii) What is the stretching force needed to pull
another chest expander of three springs
[identical to the springs in (i)] to 52 cm?
(c) Table 1 shows 4 toy pistols with different
specifications. You are required to determine the
most suitable toy pistol that can shoot the dart the
furthest.

163

Forces and Motion

CHAPTER

5 Diagram 6 shows two eggs of the same mass


released from the same height. One egg falls on a
concrete floor and breaks while the other one falls on
a thick folded towel and remains unbroken.

Essay Questions
F
O
R
M
4

Table 1

Characteristics of the dart:


Rough body
Density = 1.6 g cm3

= 1.6 g cm3



(a)



(a)

(b)
Characteristics of the dart:
Rough body
Density = 1.2 g cm3

F
O
R
M



(a)
4

CHAPTER

Characteristics of the dart:


Smooth body
Density = 0.8 g cm3

CHAPTER

F
O
R
M

(b)

(b)
Characteristics of the dart:
Smooth body
Density = 2.0 g cm3

S


(a)




(b)

Study the specifications of all four toy pistols from the following aspects:
The force constant of the spring, k
The density of the plastic dart
Compression of the spring
The body of the dart
Explain the suitability of each aspect.
Justify your choice.

[10 marks]

Experiment
1 A student carries out an experiment to study the relationship between the speed, v, of a trolley and the distance of
compression, e, of a spring that will push the trolley down the track after the compression of the spring is released.
A friction-compensated runway and a ticker-timer with its accessories are set up as shown in Diagram 1. The trolley is
pushed back to compress the spring with a compression, e, of 1.0 cm. The trolley is released. It moves at a constant
velocity, v, down the runway, after it is pushed by the compressed spring.

a.c. power supply

Diagram 1
Forces and Motion

164

The procedure is repeated with compression values of e, 2.0 cm, 3.0 cm, 4.0 cm and 5.0 cm.
The ticker tapes are shown in Diagrams 3(a), (b), (c), (d) and (e) respectively.
Diagram 2 shows the parts of the actual size ticker tape obtained in the experiment.
The velocity, v, of each compression can be calculated by using the formula:
x
v=
cm s1
0.2




in which x is the distance covered in an interval of 10 ticks. An example is shown in Diagram 2.

CHAPTER

Diagram 2 Part of a ticker tape at a constant velocity


(a)
e = 1.0 cm


(b)
e = 2.0 cm


(c)
e = 3.0 cm


(d)
e = 4.0 cm


(e)
e = 5.0 cm
Diagram 3

(a) In the experiment described above, identify



(i) the manipulated variable,
[1 mark]

(ii) the responding variable,
[1 mark]

(iii) the constant variable.
[1 mark]

(b) (i) Using the method shown in Diagram 2, measure x of every ticker tape in Diagram 3 with a ruler.
x
, calculate the value of v for every ticker tape.

(ii) Using the formula v =
0.2
Tabulate your results for e, x and v.
(c) On the graph paper provided, draw a graph of v against e.
(d) Based on your graph, state the relationship between v and e.

COMPANION WEBSITE

Online Tests

165

[7 marks]
[5 marks]
[1 mark]

Forces and Motion

F
O
R
M
4

Das könnte Ihnen auch gefallen